You are on page 1of 254

System

Fundamentals
Workbook

Raj Aggarwal
BEng PhD CEng FIEE SMIEE
Brian Bolton
BSc MSc PhD CEng MIEE
Richard Daniels
BSc MSc CEng MIEE
Phil Moore
BEng ACGI PhD CEng MIEE

Acknowledgements
Electrical Power Systems by Distance Learning (EPS) has been
developed and produced at the University of Bath. This EPS
text forms part of a University of Bath programme of study
and is published by the Distance Learning Unit within the
Faculty of Engineering and Design. EPS programmes of study
are supervised by the Department of Electronic and Electrical
Engineering.

Programme Team
Department of Electronic and Electrical Engineering
Head of Department
Adrian Evans
Production
Educational Technologist Tracey Madden
Graphic Design
Nicola Robinson

Published by the Distance Learning Unit


Faculty of Engineering and Design
University of Bath, Claverton Down, Bath BA2 7AY

Extracts in this publication are reproduced by permission


of the relevant copyright holders.
Copyright 2009 The University of Bath

All rights reserved; no part of this publication may be reproduced,


stored in a retrieval system, or transmitted in any form or by any
means, electronic, mechanical, photocopying, recording, or
otherwise without the prior written permission of the publishers.
Typeset by Tradespools, Frome, Somerset.
Printed by the University of Bath Imaging, Design and Print
Services.
V0909

Unit Components
The System Fundamentals unit is an integrated learning package
comprising the following elements:
Workbook

The textbook: Power system analysis and design


(Glover & Sarma)

SAQ answers booklet

You will find guidelines for the use of all these materials in the
student handbook, available through Moodle.
Self-assessment questions (SAQs) are used throughout the workbook
to make sure that you understand all the concepts. Worked
solutions are provided in a further booklet but do not turn to them
too early. Test yourself!
In addition there are numerous exercises in each section which
help the flow of the argument and for which answers usually
appear in the text that follows. Occasionally an exercise might ask
you to consider an aspect from your own experience or working
environment. In these cases the answer you give will be specific
and personalised and for this reason a solution as such cannot be
provided. Like the SAQs it is a good idea to do the exercises as you
work through the workbook. Use the answer space in the exercise
box for your workings or notes, or use your own paper as necessary.
Try not to skip any. Some of them may seem elementary and it can
be tempting to hurry on to the next paragraph, but they all have a
purpose.
It is a good idea to make your own notes as you go along; these will
be useful when you are revising for the examination. To help you
identify important concepts you will find that each section starts
with a list of learning objectives. Make sure you understand all of
them before moving on to the next section.
Good luck with your studies!

Section 1

Introduction
1.0 Introduction ............................................. page 1-1

Section 2

Basic considerations
2.0
2.1
2.2
2.3
2.4
2.5
2.6
2.7
2.8

Section 3

Objectives ................................................. page 2-1


Introduction ............................................. page 2-1
Phasors ..................................................... page 2-2
Power in ac circuits ................................. page 2-10
Balanced three-phase systems ................. page 2-19
The single-line equivalent circuit ............. page 2-28
Per unit notation .................................... page 2-31
Network calculations .............................. page 2-35
Matrix analysis of electric circuits ............ page 2-39
Self-assessment questions ........................ page 2-58

Transformers and switchgear


3.0 Objectives ................................................. page 3-1
3.1 The transformer ........................................ page 3-1
3.2 Switchgear ............................................. page 3-38
Self-assessment question ......................... page 3-48

Section 4

Generation
4.0
4.1
4.2
4.3

Objectives ................................................. page 4-1


Introduction ............................................. page 4-1
Steady-state operation .............................. page 4-3
The load chart for a synchronous generator
feeding an infinite busbar ......................... page 4-8
4.4 Power transfer between synchronous
systems ................................................... page 4-11
4.5 Transient operation ................................. page 4-13
4.6 Fault conditions and the equal area
criterion .................................................. page 4-26
Self-assessment question ......................... page 4-31
Section 5

Load flow analysis


5.0
5.1
5.2
5.3
5.4

Objectives ................................................. page 5-1


Introduction ............................................. page 5-1
The load flow problem ............................. page 5-2
The Gauss iterative method ...................... page 5-4
The Newton-Raphson method ................ page 5-12
Self-assessment question ......................... page 5-16

Section 6

Unbalanced systems
6.0
6.1
6.2
6.3
6.4
6.5

Section 7

Fault calculations
7.0
7.1
7.2
7.3

Section 8

Objectives ................................................. page 6-1


Introduction ............................................. page 6-1
Symmetrical components ......................... page 6-1
The operator ............................................. page 6-3
Sequence components of impedance ....... page 6-6
Sequence impedances of transformer
connections ............................................ page 6-11
Self-assessment questions ........................ page 6-14

Objectives ................................................. page 7-1


Introduction ............................................. page 7-1
Fault analysis ............................................ page 7-3
More complicated networks ...................... page 7-9
Self-assessment questions ........................ page 7-13

Transmission line characteristics


8.0 Objectives ................................................. page 8-1
8.1 Introduction ............................................. page 8-1
8.2 The series inductance of a two-conductor
line ........................................................... page 8-2
8.3 Mutual inductance and electromagnetic
interference .............................................. page 8-5
8.4 The self-inductance of a co-axial cable ...... page 8-8
8.5 The shunt capacitance of a two-conductor
line ........................................................... page 8-9
8.6 The concentric cable ............................... page 8-12
8.7 Simple line models ................................. page 8-14
8.8 Line performance chart ........................... page 8-19
Self-assessment questions ........................ page 8-25

Section 9

Principles of protection
9.0
9.1
9.2
9.3

Objectives ................................................. page 9-1


Introduction ............................................. page 9-1
Types of protection ................................... page 9-2
Protection settings for a ring distribution
system ...................................................... page 9-8
9.4 Distance protection .................................. page 9-9
9.5 Unit protection ...................................... page 9-13
Self-assessment questions ...................... page 9-16

Section 1
INTRODUCTION

Welcome to the foundation module of the distance learning


masters course in electrical power systems. Welcome also to the
debate on what constitutes the foundations or fundamentals of
such a complex and multi-disciplinary subject.
A brief survey of different national power supply companies shows
a range of perceptions of what a power system may be and what
might be seen as fundamental to it. Many years ago, the power
system of England and Wales showed a very clear division between
the distribution of electricity and the generation and transmission
of electricity. For many years the Central Electricity Generating
Board was responsible for generation and transmission in England
and Wales and within CEGB there was a strong nuclear lobby. The
concerns and interests of such an organisation were bound to be
quite different from those of the distribution companies. Within
the distribution companies themselves there was little interest in
generation or system stability and it sometimes seemed that the
two sectors had no real concern for, or knowledge of, each others
problems. In the modern English and Welsh system we see
distribution engineers planning and operating generating stations,
transmission engineers wrestling with the complexities of
marketing and generators dealing directly with the consumers of
electricity.
In Scotland, of course, things were always different and
generation, transmission and distribution were tied together in a
vertically integrated industry which many would argue is the only
sensible way to operate a power system. So power engineers in
Scotland would instinctively have had a more holistic view of
power system engineering than would their equivalents in England
and Wales. It is a view that the French might share because they
too have a vertically integrated system, although nuclear power
dominates the French scene, and there is that strange
amalgamation of electrical distribution and gas distribution in
EDF/GDF.
The view from the USA is different again and the Americans have
always been more ready to look upon electricity as a commodity
to be bought and sold; a view which is now prevalent in the
privatised industry of England and Wales.
INTRODUCTION

1-1

Module 1

So what is fundamental the generation, transmission or


distribution of electricity, the network, the system or the
commercial base? Or should we look further and recognise the
new technologies which will underpin the operation of any
modern, high-tech power system, such as information processing,
communication, pattern recognition, the application of neural
networks to control and operation, and so on?
For this course we have chosen to take a systemic view of the
industry, but we have opted to begin with a fairly traditional
introduction. We have set aside the actual generation of energy as
being a rather specialist subject and have chosen to see the
generator as a system component which plays an essential part in
the dynamic and transient operation of the electricity supply
system, but we will not look in any detail at how a coal, nuclear or
gas-fired power station actually works. The network is of course
central to any system and understanding its operation in various
modes is fundamental to an understanding of the system, but to
do so one needs to understand the way such a network can be
modeled in overall terms. The components that make up the
network and its interconnections also have to be modeled and this
leads to a fundamental need to be able to describe all parts of the
system mathematically in a form which enables accurate and valid
representation.
The system has to be operated safely, reliably and economically
and this requires the accurate and valid generation, transmission
and processing of information supported by robust control systems
and an operational policy which has a strategic view of the whole,
integrated system.
Overall therefore we have a vision of an electrical power system as
a fairly unified model, which for advanced operation needs to be
understood in mathematical as well as system terms and which
can only be operated and analysed efficiently using advanced
computer systems. The various modules of this course lead you in
easy stages to this overview and the skills and capabilities
developed will be of value to any ambitious power system engineer
in whichever particular field he or she chooses to work.
This foundation module is the first step on that path and it is
intended to pull together and consolidate a number of issues
which may be familiar to you, new to you or perhaps halfremembered from past courses. Where the material is familiar
there will be a temptation to move on quickly, but we would
recommend that you make a point of completing each exercise in
each section to satisfy yourself that you are indeed on top of the
subject.
The course team wish you every success and look forward to
corresponding with you and meeting you at the first residential
week.
1-2

SYSTEMS FUNDAMENTALS

Section 2
BASIC CONSIDERATIONS

2.0 Objectives
When you have completed this section you should be able to:

Calculate the magnitude and direction of the flow of power


in simple three-phase networks

Establish single-line equivalent circuits to represent threephase circuits with all parameters expressed in per unit
values

Carry out simple network calculations involving network


reduction.

2.1 Introduction
The purpose of this section is to revise some of the fundamentals
of power system analysis. Most of it will be familiar to you, but it
may be some time since you have used the theory and a quick
revision now will help you to avoid making mistakes in later
chapters.
The whole point of power system analysis is to model a complex
system so that we can analyse its performance and make
predictions about its future behaviour. We need to be certain
therefore that the basic building blocks are firmly in place.
If you are feeling very confident, then you can jump to the selfassessment questions (SAQs) at the end of this section. If you can
complete all of them correctly and with ease then by all means
jump ahead to Section 3.

BASIC CONSIDERATIONS

2-1

Module 1

2.2 Phasors

2.2.1 Introduction
There is a section on this topic in the second chapter of the
reference text Power System Analysis and Design, so you can get two
slightly different views of the same material.
With an ac power system in steady-state operation we are usually
dealing with voltages and currents which are varying sinusoidally
with time. We can represent the voltage of such a system as:

v V max cosot
where v is the instantaneous value of the voltage, V max is the
maximum value of the voltage, o 2pf is the angular frequency
of the supply voltage and f is the frequency in Hertz (or cycles per
second).
It is usual in power systems work to use root-mean-square (r.m.s)
values of voltage and current and the symbols that we will use for
these respectively are:

V and I
For a sinusoidal waveform:

V max
I max
V p and I p
2
2
All values for voltage and current given in this text will be r.m.s.
values and the supply frequency will be 50 Hz unless otherwise
stated.
Now let us apply a voltage V 220 V across an inductance of
value L 0:732 H.
We have:

vL

di
dt

so that

V max cosot L

2-2

SYSTEMS FUNDAMENTALS

di
dt

Section 2

or

di V max
cosot

L
dt
so that

V max
cosot dt
L

giving

V max
sinot
oL

The maximum value of i is

I max

V max
oL

and in r.m.s notation

I V=oL

Exercise
Sketch the curves for v and for i
on the axes given in Figure 2.1

Figure 2.1

The cosine curve for the voltage should present no problem.


Remember that we quoted an r.m.s. value so that our peak value
of voltage on our sketch should be:

V max

p
26220 311 V

The frequency is 50 Hz, the period is 1=50 20 ms and the


wave completes one full cycle in 20 ms.
BASIC CONSIDERATIONS

2-3

Module 1

The maximum value of the current in the inductor is:

I max

V max V max

1:35 A
oL
2pfL

The r.m.s. value of i is

1:35
I p 0:96 A
2
The current wave also repeats in 20 ms, but it is a sine wave. It
will appear as shown in Figure 2.2.

Figure 2.2
The current wave reaches its maximum value 5 ms later than the
voltage wave. It is said to lag the voltage by 5 ms. This time is
equivalent to p=2 radians or 90 electrical degrees.
So we have:

v V max cosot
and

i I max sinot
We can rewrite the equation for current as:

i I max cosot  p=2


The p=2 term demonstrates that the waveform is lagging a
reference cosine wave by p=2 radians.

2-4

SYSTEMS FUNDAMENTALS

Section Two

2.2.2 R-L Circuits


If we now include a resistor R in series with the inductor then the
equation for voltage becomes:

and if we solve for the current we obtain for the steady-state part
of the solution:

where is the angle tan1(L/R).


The current still lags the voltage, but by an angle which will be
less than /2 but greater than zero. The case is illustrated in
Figure 2.3. Note that the horizontal scale in this figure is in
radians. This has been done so that we can show directly
instead of having to convert it into an equivalent time.

Figure 2.3

The angle is known as the power factor angle and it will always
be negative when the current is lagging the voltage.

BASIC CONSIDERATIONS

2-5

Module 1

2.2.3 Capacitive Circuits


Exercise
Take the example of a capacitor
of value C 14:5 mF with a
voltage of 220 V across it and
repeat the above derivation to
show that the current in the
capacitor is given by:

i I max cosot p=2

In this case

1
v
C

i dt

so that, if

v V max cosot
then

d
d
Cv CV max cosot
dt
dt

i oCV max sinot


and since

sinot cosot p=2

2-6

SYSTEMS FUNDAMENTALS

Section 2

we have:

i oCV max cosot p=2


or

I max oCV max

V max
Xc

where

Xc

1
oC

In this case the current leads the voltage by p=2 radians or 90


electrical degrees. If we include a resistance in series with the
capacitance then the angle by which the current leads the voltage
is reduced to a general power factor angle f which is greater than
zero but less than p=2. The equation for current is then:

i I max cosot f

2.2.4 Polar and Rectangular Notation


Now, having reminded ourselves of the principle of lagging and
leading currents, let us look at two other ways of expressing the
relationships between sinusoidal quantities.
The first is the polar method.
This is based on the idea that we can generate a sinusoidal
waveform by the vertical projection of a rotating phasor. We start
with a horizontal phasor of magnitude V , as shown in Figure
2.4(a). We then allow the phasor to rotate anticlockwise at an
angular frequency o. After a time t 1 the phasor has progressed to
the position shown in Figure 2.4(b) and the projection of the
phasor on to the vertical axis is given by Vsinot 1 . Thus the
value of Vsinot at any time t 1 can be represented by a phasor V
at an angle ot 1 to the horizontal.

BASIC CONSIDERATIONS

2-7

Module 1

Figure 2.4
What is particularly useful to us is that a second sinusoidal
quantity which is delayed by an angle f, for example the phasor
Isinot  f, can be represented by a phasor I at an angle
ot 1  f as shown in Figure 2.4(c).
We make this the basis of our polar representation, by choosing
one phasor as our reference and referring all others to it.
Let us select V as our reference and represent it as V0 . We
imply that it is sinusoidally varying at an angular frequency o, but
the zero tells us that we have chosen it as our reference phasor.


We now describe the other phasor as I  f . This tells us that


the phasor has a magnitude I , but that it lags the reference phasor
by f degrees (f could be given in radians, but it is usual to give
the angle in degrees when using this format).
Finally we can switch to rectangular co-ordinates.
The phasor V shown in Figure 2.5 can be described at a particular
instant in time as either Va or as a phasor made up of two
components:
a horizontal component Vcosa,
and a vertical component Vsina.
To make it clear which is which we use the method of complex
numbers. The horizontal component is defined as the real
component and the vertical component is defined as the
imaginary component and is preceded by the operator j.
Thus: Va Vcosa jVsina

2-8

SYSTEMS FUNDAMENTALS

Section 2

Figure 2.5
Exercise
Add together the two vectors
5030 and 50  60 .

5030 50  60 50cos30 j50sin30 50cos60 j50sin60


43:3 j25 25  j43:3
68:3  j18:3
H68:32 18:32 angle tan1 18:3=68:3
70:7  15
It is useful to be able to switch from polar to rectangular coordinates and back again with ease. In calculations involving
addition and subtraction we should always use the rectangular
form, because:

a jb c jd a c jb d
and

a jb  c jd a  c jb  d
When we need to multiply and divide however, the polar form is
best because:

aa 6b b a6b a b
and

aa 7bb a7b a  b


BASIC CONSIDERATIONS

2-9

Module 1

The angles are simply added for multiplication and subtracted for
division.
You can of course convert a and b to rectangular form and
multiply through as follows:

aa a1 ja2
bb b1 jb2

aa6bb a1 ja2 b1 jb2


a1 b1  a2 b2 ja2 b1 a1 b2
the magnitude of this is:

q
ja6bj a1 b1  a2 b2 2 a2 b1 a1 b2 2
and the angle is:
1

a  b tan

a2 b1 a1 b2
a1 b1  a2 b2

You may find the polar method quicker!


This would be a good time to try SAQ 1

2.3 Power in ac circuits

2.3.1 Introduction
The power in an ac circuit is the product of the instantaneous
voltage across the circuit and the instantaneous current in the
circuit. The unit of power is the Watt.
We need to consider for a moment the problem of sign
convention. It is easy to talk about power from a generator or
power absorbed by a load and in simple circuits we will have no
problems with signs because it is obvious where the power is
coming from and where it is going to. Thus in Section 2.2 of the
reference textbook it is assumed that the power absorbed by a load
is positive.

2-10

SYSTEMS FUNDAMENTALS

Section 2

In more complex situations we need to be more careful about


signs and we usually adopt a standard sign convention. Consider
Figure 2.6

Figure 2.6
The power in the generator is defined as Vg i and is assumed
positive because the current flows through the unit in the
direction of the increasing potential difference.
The power in the load is defined as VL i and is assumed negative
because the current flows through the unit against the direction of
increasing potential difference.
In a power system, this is important because we often move power
across ac links in different directions depending on where we have
a power requirement and where we have a generating capacity. So
a more likely reference diagram for us might be that of Figure 2.7.

Figure 2.7
You might assume that the power flow is from left to right, but we
cannot be sure until we know the relative magnitudes and phase
of the voltages at either end of the link. We shall return to this
later

2.3.2 Power absorbed in a resistor


For now, let us look at the power absorbed by a resistor R when a
voltage v V max cosot is applied across it.

BASIC CONSIDERATIONS

2-11

Module 1

The current through the resistor is:

V max
cosot I max cosot
R

The instantaneous power is:

p V max I max cos2 ot


V max I max
1 cos2ot

2
VI 1 cos2ot
So we have a power of average value VI with a double frequency
component VIcos2ot . This double frequency component
averages out to zero and the instantaneous value of power never
goes negative.

2.3.3 Power in an ac circuit with reactance


We will now apply a voltage v V max cosot across an
impedance to obtain a current flow:

i I max cosot f
This is a general case, with f taken as positive. The circuit is
therefore capacitive, but that does not matter.
The instantaneous power is:

p V max I max cosotcosot f


p

V max I max
cosf cos2ot f
2

p VI cosf cos2otcosf  sin2otsinf


p VIcosf1 cos2ot  VIsinfsin2ot
We can recognize the first term as the power in a resistor when a
current Icosf flows through it. Thus our circuit clearly has a
resistive component and the power absorbed by the resistance is
the power that would be absorbed by a current Icosf flowing
through a circuit that had a voltage v V max cosot across it. We
have previously defined the angle f as the power factor angle and
cosf is defined as the power factor.

2-12

SYSTEMS FUNDAMENTALS

Section 2

The component of power, VIcosf, we define as the average


power or the real power or the active power. You will meet all
three definitions in power systems work.
Now consider what else there is in the power equation. If we
remove the average power absorbed in the resistance then we are
left with:

VIsinfsin2ot
This is a double-frequency component and has an average value of
zero. The component clearly alternates between positive and
negative values and so represents energy which is pulsating into
and out of the load. This is the energy required to establish the
magnetic field of the current and the electric field of the voltage.
As these two quantities vary sinusoidally the energy into and out
of the field oscillates at double frequency, but no power is
consumed. Because the average power of this component is zero it
would be nice if we could ignore it altogether, but we cannot
because it has a component of current Isinf associated with it.
This current does cause losses in the system. Imagine for example
that the load is wholly reactive so that no power is consumed by
the load. The current flow would still cause heating in the
generators and transformers that provided the supply. It is quite
possible to have a situation where there is no average power being
delivered to the load, but the generator and transformer are on
overload because the heating effect has raised their operating
temperatures above the normal working level. This component of
current is therefore vitally important to us as power engineers
because it limits the capacity of the system to deliver real power.
Consider again the equation for this double-frequency component
of power:

VIsinfsin2ot
The amplitude of the oscillation is:

VIsinf
We define this as the reactive power Q , so that

Q VIsinf
It has the same units as real power, but to make it clear that this
component has no real value we use the unit var, or Volt-Ampere
reactive, for it.

BASIC CONSIDERATIONS

2-13

Module 1

Exercise
Calculate
(a)

the real power in each


branch of the circuit
shown in Figure 2.8

(b)

the reactive power in


each branch of the
circuit

(c)

the total power taken


from the supply.

Figure 2.8
Let us take the supply voltage in Figure 2.8 as our reference
phasor, then the current in the 50 O resistor is given by:

I1

2400
4:80 A
50

The real power absorbed by the resistor is then:

P1 VI 1 cosf 24064:86cos0 1152 W


The reactive power absorbed by the resistor is:

Q 1 VI 1 sinf 24064:86sin0 0 var


The current in the second branch of the circuit is:

I2

2400
2400

4:8  53
30 j40 5053

The real power absorbed by the branch is:

2-14

SYSTEMS FUNDAMENTALS

Section 2

P2 VI 2 cosf 24064:86cos53 693:3 W


The reactive power absorbed by the second branch is:

Q 2 VI 2 sinf 24064:86sin53 920 var


The total power taken from the supply is the sum of these four
components i.e.

Ps 1152 693:3 1845 W


Q s 0 920 920 var

2.3.4 Complex Power


Let us look again at the total power absorbed by the second
branch in Figure 2.8. It has two components:

P2 VI 2 cosf 24064:86cos53 693:3 W


and

Q 2 VI 2 sinf 24064:86sin53 920 var


We can see that these are the rectangular co-ordinates of a phasor:

S 115253
We define S as the complex power. Its units are VA (voltamperes).
We then use the rectangular co-ordinates and the j operator to
define, in general:

Sa Scosa jSsina
P jQ
Now we have to be careful!
The complex power S is clearly the product of a voltage and a
current, but look what happens when we simply multiply the
voltage across the second branch of the circuit in Figure 2.8 by the
current through it:

V 2400
I 2 4:8  53

BASIC CONSIDERATIONS

2-15

Module 1

V6I 2 24064:8  53 1152  53


1152cos53 j1152sin53
693:3  j920
THIS IS NOT THE COMPLEX POWER IN THE BRANCH!!!!!!!
Notice the error in the sign of the reactive component
So to get the correct answer for the complex power using phasors
we have to define S as follows:
When a voltage V0 is measured across a circuit in which a
current If flows, the complex power absorbed by that circuit is
defined as:

S VI *
where I * is the conjugate of I , i.e. if I If then I * I  f
Now we can obtain the complex power in the second branch of
the circuit as follows:

V 2400
I 2 4:8  53
I * 4:8 53
2

S VI *2 115253
S 693:3 j920
. . .which is correct.
This explains another odd feature of power calculations in power
systems which often confuses the newcomer. Current into an
inductive circuit (e.g. the second branch in Figure 2.8) lags the
voltage and is given in general by

I  f Icosf  jIsinf
but the complex power in an inductive circuit is given by:

S VI * VI f
S VIcosf jVIsinf
Notice the difference in sign between the two reactive terms.

2-16

SYSTEMS FUNDAMENTALS

Section 2

Similarly in a capacitive circuit, the current leads the voltage, so:

V V0
I I f
I * I  f
S VI  f
I Icosf jIsinf
S VIcosf  jVIsinf

Exercise
Calculate the power consumed
in each of the branches of the
circuit illustrated in Figure 2.9
and the total power taken from
the supply.

Figure 2.9
In the first branch we have a reactive impedance 30  j40 O i.e.
the branch is capacitive.
BASIC CONSIDERATIONS

2-17

Module 1

The current in the branch is:

2400
2400

I1
4:853 A

30  j40 50  53
The conjugate of this current is 4:8  53 A
The complex power is:

S1 VI *1 2400 64:8  53 1152  53


S1 693  j920 VA
In the second branch we have a reactive impedance 40 j30
i.e. the branch is inductive.
The current in the branch is:

I2

2400
2400

4:8  37 A
40 j30 5037

The conjugate of this current is 4:837 and the complex power


in the branch is:

S2 VI *2 2400 64:837 115237


S2 920 j693 VA
The total complex power from the generator is the sum of these
two components, so

ST 693 920 j920 693 1613  j227 VA


or

PT 1613 W and Q T 227 var:


This introduces another terminology that often causes confusion,
but should be clear in the context of the above example. This is
the notion of generating and absorbing var.
The inductive branch in Figure 2.9 absorbs 693 var.
The capacitive branch absorbs 920 var.
Absorbing something negative is equivalent to generating
something positive, so it is common to refer to a capacitive circuit
as generating vars.

2-18

SYSTEMS FUNDAMENTALS

Section 2

In our example the capacitive branch generates 920 var. The


total var requirement from the system is obviously affected by this.
The inductive circuit demands 693 var, just as it demands 920 W
and it doesnt matter where the var comes from.
In our example, the capacitive circuit generates 920 var of which
693 can go to the inductive branch. This leaves 227 var which
the generator has to absorb.
This means that the electrical conditions in the supply must be
such that it can absorb 227 var.
Overall the sum of complex powers in a closed system must be
zero. The real power components must sum to zero i.e. generation
must equal absorption, and the reactive components must sum to
zero, i.e. var absorbed by inductive elements must be balanced by
var generated by capacitive elements. The a.c. generator can
generate or absorb var depending on its excitation, but more of
that later.

2.4 Balanced three-phase systems

2.4.1 The three-phase supply


We assume that our supply comes from a three-phase a.c.
generator which is designed to produce the following voltages
from three separate windings:

V a1 a2 V0
V b1 b2 V  120
V c1 c2 V  240
The windings are represented diagrammatically in Figure 2.10a
with an angular separation of 120 . In Figure 2.10b the windings
are connected together and the common connection is called the
neutral point, n. This connection is known as the Star or Wye
connection.

BASIC CONSIDERATIONS

2-19

Module 1

Figure 2.10
We can redefine the voltages as:

V a1 a2 V an
V b1 b2 V bn
V c1 c2 V cn
The voltage phasors are represented in Figure 2.11 with a common
reference point at n. The reference phasor is V0 .

Figure 2.11
The voltages between each generator terminal and the neutral
point all have the same magnitude V , but they have different
phase relationships. The magnitude of the voltages V an ; V bn and
V cn are known as Phase voltages because they represent the
voltage across the phase. The voltages between the terminals,
V ac ; V cb and V ba are known as the Line voltages because they
are the voltages between the power lines that leave the generator.
We can determine the relationship between the line voltages and
the phase voltages for a star-connected system as follows:

2-20

SYSTEMS FUNDAMENTALS

Section 2

the voltage V ac is the voltage of point a with respect to point c.


This is the voltage of point a with respect to point n minus the
voltage of point c with respect to point n, i.e.

V ac V an  V cn
hence,

V ac V0  V  240
V ac Vcos0 jVsin0  Vcos240  jVsin240
V ac V j0 0:5 V  j0:866 V
V ac 1:5 V  j0:866 V


p
1 0:866
2
2
V
V ac 1:5 0:866 tan
1:5
V ac 1:732  30 V
p
V ac 3  30 V
We see that we have a simple magnitude relationship between
phase and line voltages. The line voltages are always H3 times the
phase voltages in balanced three-phase systems.

2.4.2 Balanced Three-Phase Star-Connected Loads


In Figure 2.12 we illustrate a balanced three-phase load connected
to the balanced three-phase supply of the previous section. Each
load impedance is identical in magnitude and phase. The load is
connected in star with a common point at s. Both n and s are
earthed to provide a return connection from load to supply.

BASIC CONSIDERATIONS

2-21

Module 1

Figure 2.12
The current in the a phase of the generator can be determined
simply as:

ia

V an V 
0
Z
Z

similarly

V
 120
Z
V
ic  240
Z
ib

Exercise
Carry out the summation of
ia ; ib and ic .

2-22

SYSTEMS FUNDAMENTALS

Section 2

V
[cos0 + jsin0 + cos( 120) + jsin( 120) + cos( 240) + jsin( 240)]
Z
V
= [1 + j0 0.5 j0.866 0.5 + j0.866]
Z
=0

ia + ib + ic =

So, in a simple balanced load fed from a balanced three-phase


supply the three load currents sum to zero. For this simple case,
the neutral connection between s and n is redundant. Notice also
from the circuit of Figure 2.12 that the currents in the phases of
the generators and the load are the same as the currents in the
lines.
The total power taken by the load must equal the sum of the
powers taken in each branch. We can therefore deduce that the
total complex power taken by the load is:

S = 3VI*
for Va

= V0
ia =

V
V
=
= I
Z

The total complex power is then:

S = 3VI = 3(VIcos jVIsin)


It is worth looking quickly through the full derivation of this
result in Section 2.6 (pages 5771) of the reference text because
the full analysis reveals an important fact. This is that the
instantaneous power delivered is constant over time. In other
words the pulsating component of real power which we found in
the single-phase case has gone! This is very important because it
means that there are no pulsating torques on the drive shaft of
the generator.
We have therefore S

= 3VI* in a three-phase balanced system.

We can rewrite this in terms of the line voltages, VL, so that S

3VLI*

The three-phase power in a balanced system can be defined either


as:

S = 3VPI*P

BASIC CONSIDERATIONS

2-23

Module 1

or

S H3V L I *L
where: V P is the phase voltage,

V L is the line voltage,


I P is the phase current, and
I L is the line current.
When system values are quoted, it is usual to quote three-phase
and line quantities. Thus a 400 kV system delivering 3000 MVA
at a power factor of 0:8 lagging is a three-phase system with a line
voltage of 400 kV which is delivering 1000 MVA to each phase of
the system.
The current per phase is:

p
30006106
3
6
IP
4330 A
4006103
3
The power factor is 0.8 lagging (i.e. 36:9 ), so:

I P 3464  j2600 A

Exercise
A 381 V, three-phase, starconnected, a.c. supply delivers
250 kW at a power factor of
0:707 lagging to a three-phase,
star-connected load. Determine
the current in each phase of the
load.

The total real power delivered is P 250 kW.

2-24

SYSTEMS FUNDAMENTALS

Section 2

The total VA delivered is:

P
2506103
35445 kVA

0:707
cosf
p
but S 3V L I L*
S

3546103
;I L* p
45 53645 A
36381
;I P I L 536  45 A

Exercise
We wish to raise the power
factor of the supply to 0:85
while maintaining the real
power delivered constant. To do
this we connect three equal
capacitors in a star connection
in parallel with the load.
Calculate the reactance of each
capacitor.

We begin with S 35445 250 j250 kVA


We maintain the power at 250 kW and raise the power factor to
0.85. Then, S 2506103 =0:85 29431:8 kVA
or, S 250 j155 kVA
The reduction in var is 250  155 95 kvar and this must be
generated by the capacitors, so each phase bank must provide
95=3 31:7 kvar.
Therefore,

XC

V 2P
48400
1:526 O:

VAP 31700

2.4.3 Balanced Three-Phase Delta connections.


The three generator windings represented in Figure 2.10a could
theoretically be connected in a delta arrangement as shown in
Figure 2.13. The line connections would be made at the junctions
of a1 c2 ; c1 b2 and b1 a2 . the voltages across the lines are now the

BASIC CONSIDERATIONS

2-25

Module 1

voltages across the phase windings, so that in a delta-connected


supply we have VL = VP.

Figure 2.13

The phase relationship depends on the exact nature of the phase


connections and we will deal with this when we look at
transformer connections.
For the moment let us just accept that we could have a deltaconnected supply if we so wished.
A three-phase load may also be connected in a delta arrangement.
A possible arrangement is represented in Figure 2.14 in which the
delta-connected load is fed from a star-connected supply.

Figure 2.14

Van = V 0
Vbn = V 120
Vcn = V 240
From Section 2.4.1 we know that the line voltage

Vac = 3 V 30
so that the voltage

2-26

SYSTEM FUNDAMENTALS

Section 2

V a0 c0 V ac H3 V  30
The current in the ac branch of the load is then:

I a0 c0

V a0 c0 p V
3  30  f
Zf
Z

V ba V bn  V an V  120  V0

p
3 V  150

The current in the ba branch of the load is:

I b0 a0

V b0 a0 p V
3 150  f
Zf
Z

The current in line a is:

I a I a0 c0  I b0 a0
p
p
3 V30  f
3 V150  f
Ia

Z
Z
p
3 V p
Ia
6 3  f
Z
which is H3 times the phase current I a0 c0 but 308 in advance of it.
So in general, the line current to a delta connection has a
magnitude of H3 times the current in each phase of the delta, or

I L H3 I P

Exercise
A 381 V, three-phase, starconnected, ac supply delivers
250 kW at a power factor of
0:707 lagging to a three-phase,
delta-connected load. Determine
the current in each phase of the
load and in each line.

The total real power delivered is P 250 kW.


The total VA delivered is

S 2506103 =0:707 35445 kVA:


BASIC CONSIDERATIONS

2-27

Module 1

S = 3 VL I L*
354 103
45D = 53645D A
3 381
I L = 536 45D A
536
( 45 30)D
IP =
3
I L* =

Remember that the reference phasor here is the phase voltage


V0 in the supply.

2.5 The single-line equivalent circuit


2.5.1 Introduction
When dealing with balanced three-phase circuits it is not
necessary to work with the full complexity of three-phase
connection. A single-line equivalent circuit is all that is needed.
Figure 2.15 illustrates the case.

Figure 2.15

The supply voltage V0 is the reference phase voltage, Zs


represents any series impedance (per phase) and ZL is the phase
impedance, assuming a balanced three-phase, star-connected
load.
Power is calculated per phase and the three-phase power is simply
3 V cos. The neutral return is a zero impedance
connection.

2-28

SYSTEM FUNDAMENTALS

Section 2

2.5.2 Delta-Star Transformations


Three delta-connected impedances as represented in Figure 2.16a
can always be replaced by the star connection of Figure 2.16b,
where:

ZAB 6ZCA
ZAB ZBC ZCA
ZBC 6ZAB
ZB
ZAB ZBC ZCA
ZCA 6ZBC
ZC
ZAB ZBC ZCA
ZA

Figure 2.16
The reverse transform, from star to delta, gives the following
relationships:

ZA ZB ZB ZC ZC ZA
ZC
ZA ZB ZB ZC ZC ZA
ZBC
ZA
ZA ZB ZB ZC ZC ZA
ZCA
ZB
ZAB

BASIC CONSIDERATIONS

2-29

Module 1

Exercise
A balanced three-phase load
consists of three impedances
each of value 4 j3 O
connected in a delta
arrangement. The load is
supplied from a 381 V, threephase, balanced supply. The
lines connecting the load to the
supply each have an impedance
of 1 j4 O.
Calculate:
(a)

the line current drawn


from the supply

(b)

the current in each


impedance of the load

(c)

the total complex power


provided by the supply.

We should first convert the delta-connected load into a star


equivalent. Using the formula given above:

537 6537
1:6737 O
34 j3
ZA 1:33 j1:01 O
ZA

The total series impedance in the line equivalent circuit is:

1:33 j1:01 1 j4 2:33 j5:01 5:5365


The line current drawn from the supply is then:

IA

VA
220

39:8  65 A
ZS 5:5365

This is also the current in each equivalent star connected


impedance.
The current in each actual impedance is:

IL
IP p 23 A
3
The phase angle of the line current, with respect to the reference
2-30

SYSTEMS FUNDAMENTALS

Section Two

voltage 2200, will be 65. The phase angle of the actual


phase current in the load will be 65 30 or 95 with
respect to the reference voltage 2200, but 65 with respect to
the line voltage across the actual impedance.
The complex power provided by the supply per phase is:

S = VI*
S = 2200 39.865
S = 875665 VA
S = 3700 W + j7936 var
The total complex power provided by the supply is therefore
11.1 kW + j23.8 kVAr

2.6 Per unit notation


2.6.1 Per unit notation
Per unit notation expresses values as a fraction, or per unit, of a
reference base. Thus if we select a voltage of 1000 V as our base,
or reference, value then any other voltage v can be expressed as:

v
p.u.
1000
A voltage of 500 V is 0.5 p.u. to a 1000 V base and a voltage of
1500 V is 1.5 p.u. to a 1000 V base.

2.6.2 Base quantities


It is only necessary to select two independent base quantities in
power system work because all other base quantities can be
defined in terms of the chosen two.
It is usual to select voltage (V) and complex power (VA) as the
independent base quantities and there will be less confusion if we
use single-phase, or line-to-neutral values throughout. The actual
base values are chosen to lie close to the nominal values of the
system being represented.
Thus for a single-phase transformer nominally rated at 6.4 kV
primary voltage and 100 MVA the selected base quantities would
be voltage and volt-amperes and their values would be:

BASIC CONSIDERATIONS

2-31

Module 1

base voltage; V b

6:4 kV

and
base volt-amps;VAb

100 MVA

From these two quantities we can define other base values as


follows:

VAb 1006106

15:6 kA
Vb
6:46103
Vb
V 2b
6:42 6106

0:41 O
base impedance; Zb
Ib
VAb 1006106

base current; Ib

Note that these values have magnitude, but no phase.


The conversion from real values to per unit values is
straightforward once the base values have been determined.
Suppose, for example, that the transformer specified above
supplies a load of 60 MVA at 0:866 power factor lagging. We can
express this either as:

S 6030 MVA
or

S 51:96 j30 MVA;


or

P 51:96 MW and Q 30 Mvar


In per unit notation these values become:

60
30 0:630 p:u:;
100

or


S

51:96
30
j
100
100


0:52 j0:3 p:u:;

or

P 0:52 p:u: and Q 0:3 p:u:

2-32

SYSTEMS FUNDAMENTALS

Section 2

Exercise
A 500 MVA, 11 kV, threephase synchronous generator has
a synchronous reactance of
0.43 per phase and supplies
300 MVA at a lagging power
factor of 0.8. Use the singlephase nominal values of VA and
voltage to convert the impedance
and the load value into per-unit
notation. Also determine the perunit value of the load current.

Hence

It then follows that the synchronous reactance is:

The load is:


p.u. at a power factor of 0.8 lagging
The magnitude of the load current is given by:

The base current is:

and the load current in per-unit notation is the ratio of IL to Ib, or,

IL = 0.6 p.u.

BASIC CONSIDERATIONS

2-33

Module 1

2.6.3 Transformers and the per unit notation


An enormous advantage of using the per unit notation in complex
power networks is that the per unit values of voltage, current,
impedance and power do not change when they are referred from
one side of a transformer to the other, provided that the base
values are carefully selected. The ideal transformer winding thus
disappears from the system network diagram and so simplifies
considerably the network analysis.
To achieve this we select the voltage bases on either side of the
transformer to have the same ratio as the nominal transformer
voltages.
Example
A single-phase, two-winding, transformer is rated at 50 kVA,
240:120 V. The equivalent leakage impedance, referred to the
240 V winding is 0:280 O. Determine the values of the per
unit leakage impedance when referred to the primary winding and
to the secondary winding using the nominal primary and
secondary voltages as base values.
The primary solution
Base values are:

V b 240 V;
VAb 50 kVA;
Zb 1:152 O
The primary equivalent leakage impedance is then given by:

Zprimary

0:2
80 0:17480 p:u:
1:152

The secondary solution


Base values are:

V b 120 V;
VAb 50 kVA;
Zb 0:288 O
To obtain the secondary equivalent leakage impedance we have to
refer the primary value to the secondary winding. This is done by
multiplying by the square of the turns ratio. Thus the secondary
referred value of the leakage impedance is given by:
2-34

SYSTEMS FUNDAMENTALS

Section 2


Z2 Z1

N2
N1

2

0:280

120
240

2

0:0580 O

In per unit notation this becomes:

Zsecondary

0:05
80 0:17480 p:u:
0:288

We see therefore that as we refer per unit values across a


transformer there is no change in value provided that the base
voltages either side of the transformer have the same ratio as the
nominal transformer ratio.

2.7 Network calculations

2.7.1 An Example
We have already seen that we can remove some complexity from a
circuit by replacing delta connections with star equivalents and
then working with a single-line equivalent circuit. The following
analysis is an example of how circuit reduction is used to calculate
currents in a complex circuit.

2.7.2 Calculating the Fault Current in a Symmetrical


Network.

Figure 2.17

BASIC CONSIDERATIONS

2-35

Module 1

The diagram of Figure 2.17 represents a 33 kV, interconnected,


three-phase system. Generators G1 and G2 are both rated at 33 kV,
120 MVA and have series reactances, Xs , of 0:1 p.u.. The
connecting busbars are AB, BC, CD and DA and each has a
reactance of 10 O. A balanced three-phase short circuit occurs at
feeder point D. Feeder point B remains unconnected.
To calculate the current feeding into the fault:
The first thing we have to do is to sort out the parameters. We
have Xs in per unit, which will be to the equipment base i.e.
33 kV (line) and 120 MVA (three-phase). The busbar reactances
are in ohms, the voltage is in kV and the rating is in MVA.
It will be better to work in per unit throughout, so let us select the
following base values:

336103
p 19:16103 V 1:0 p:u:
3
1206106
406106 VA 1:0 p:u:
VAb
3
19:12 6106
Zb
9:12 1:0 p:u:
406106
Vb

The busbar reactance then becomes

10
1:1 p:u:
9:12
The generator reactances remain at 0:1 p.u.
We then rearrange the network diagram as shown in Figure 2.18.
Reactances are shown in p.u.

2-36

SYSTEMS FUNDAMENTALS

Section 2

Figure 2.18
The mesh ACD forms a delta network and this can be converted to
a star arrangement as shown in Figures 2.19a and 2.19b.

Figure 2.19

2:261:1
0:55 p:u:
4:4
2:261:1
ZC
0:55 p:u:
4:4
1:161:1
0:275 p:u:
ZD
4:4

ZA

Replacing the mesh ACD by the star ACD gives Figure 2.20a.

BASIC CONSIDERATIONS

2-37

Module 1

Figure 2.20
We now note that the terminal emfs of the generators are both
held at 1:0 p.u. volts throughout and since the terminals are
equipotentials they can be joined together to give the circuit of
Figure 2.20b.
The two parallel branches can now be combined to give a
reactance:

0:6560:65
0:325 p:u:
0:65 0:65

We then have the final single-line diagram shown in Figure 2.21.

2-38

SYSTEMS FUNDAMENTALS

Section 2

Figure 2.21
The current flow in this circuit is 1:0=0:6 1:67 p:u:
The base value of current is:

Ib

VAb
406106

2:16103 A
Vb
19:16103

So the fault current, per phase, is:

I F 2:16103 61:67 3:56103 A


The short circuit VA of the above circuit at the point D is defined
as:

V6I F 1:06

1:0
1:67 p:u:
0:6

or

1:676406106 66:8 MVA per phase


or

3666:8 200 MVA; three phase

2.8 Matrix analysis of electric circuits

2.8.1 Introduction
If you have used matrix algebra to solve problems in electric
circuits before and if you feel confident using it, then try the
exercises in this section and move on quickly.
BASIC CONSIDERATIONS

2-39

Module 1

The purpose of this section is to introduce you to the admittance


matrix of a simple a.c. network and show you how simple matrices
are manipulated to provide solutions to problems involving sets of
simultaneous equations.
Computers are always used to solve complex problems, but the
only way to understand what is happening is to solve a few simple
problems by hand. This is what we are about to do.

2.8.2 A simple matrix


We begin with two equations:

Y1 a1 X1 a2 X2
Y2 b1 X1 b2 X2
In matrix form these are written as:

Y1
Y2

a1

b1

a2
b2



X1
X2

Exercise
Write the given equations in
matrix form.

I1 3V1 4V2
I2 5V1 6V2


I1
I2

The two variables on the righthand side of the equations are V1


and V2 and the matrix format is

2-40

I1
I2

SYSTEMS FUNDAMENTALS

3 4

5 6



V1
V2

Section 2

Exercise

3 2
Y1
I1
4 I2 5 4
I3

32

Y5

3
V1
54 V2 5
Y9
V3

Complete the matrix form of the


equations shown.

I1 Y1 V1 Y2 V2 Y3 V3
I2 Y4 V1 Y5 V2 Y6 V3
I3 Y7 V1 Y8 V2 Y9 V3

The complete matrix format is:

3 2
Y1
I1
4 I2 5 4 Y4
I3
Y7

Y2
Y5
Y8

32 3
Y3
V1
Y 6 54 V2 5
Y9
V3

2.8.3 Circuit equations in matrix form


The circuit equations that we use are the ones obtained from a
nodal analysis of the circuit and the three parameters involved are:
Current I
Voltage V and
Admittance Y
The basic equation relating them is:

I VY
All three terms may be complex, but to begin with let us restrict
ourselves to resistive circuits and direct currents.

Figure 2.22
We begin then with the circuit in Figure 2.22 in which all
parameters are given as admittances. This satisfies the first rule of

BASIC CONSIDERATIONS

2-41

Module 1

nodal analysis which is that all impedances in the circuit should


be shown as admittances.
The second rule is to replace all voltage sources with equivalent
current sources.
This is how it is done.
A voltage source Va in series with an admittance Ya, as shown in
Figure 2.23a, can be represented by a current source Ia, of value
Ia = VaYa, in parallel with an admittance Ya as shown in
Figure 2.23b.

Figure 2.23

We can show that Figures 2.23a and 2.23b are equivalent as


follows:
1. Calculate the voltage at terminal 1 of Figure 2.23b when the
load circuit is not connected

but, we define

Ia as VaYa so,

2. Calculate the admittance seen by the outside circuit looking in


at node 1 in Figure 2.23b

Y = Ya
(the internal impedance of a current source is in.nite)
As far as the external circuit is concerned, the terminals marked 1
in Figures 2.23a and 2.23b are identical because they present the
same open circuit voltage Va and offer the same internal
admittance Ya.

2-42

SYSTEM FUNDAMENTALS

Section 2

Exercise
Calculate the values of I a and
Y a to make the following
circuits equivalent.

Figure 2.24
We convert the resistance r in the first circuit to an admittance Y
where:

1
0:1 S
10

(the unit of admittance is the siemens unit symbol S)

Ia is defined as V a Y a
so that I a 10060:1 10 A
and Y a Y 0:1 S
Let us now return to Figure 2.22 and carry out two conversions:
1. Convert Va and the admittance Y01 into a current source I1 in
parallel with with an admittance Y 1 .
2. Convert V b and the admittance Y32 into a current source I2 in
parallel with an admittance Y2 .
These are shown in the new equivalent circuit of Figure 2.25

BASIC CONSIDERATIONS

2-43

Module 1

Figure 2.25
There are four nodes marked on the circuit and numbered 0, 1, 2
and 3. The zero node is taken as reference.
Voltage values of node 1, 2 and 3 with respect to the reference are
written as:

V1
V2
V3
The voltage of node 2 with respect to node 1 is written as:

V21
and

V21 V2  V1

Exercise
Write down the voltages of node
2 with respect to node 3, and of
node 3 with respect to node 1.

V23 V2  V3
V31 V3  V1

2-44

SYSTEMS FUNDAMENTALS

Section 2

Exercise
Write down the equation to
describe the current flow from
node 2 to node 1.

I21 V21 Y12


Similarly,

I30 V3 Y03
We must now proceed with the full nodal analysis of the circuit in
Figure 2.25. To do so we apply Kirchoffs current law at each node,
except the reference, to obtain three equations.
Exercise
At node 1.
(a)

How many currents enter


or leave node 1?

(b)

Define these in terms of


given circuit values.

We have three currents at node 1.


Current I1 can be defined as entering the node.
A current equal to V1 Y1 can be defined as leaving node 1 and
flowing to the reference point.
A current equal to V12 Y12 can be defined as leaving node 1 and
flowing to node 2.

BASIC CONSIDERATIONS

2-45

Module 1

Thus:

I1 V1 Y1 V12 Y12
or
or

I1 V1 Y1 V1  V2 Y12
I1 V1 Y1 Y12  V2 Y12

For the sake of completeness we shall add on an extra term to


remind us that there are three voltages of interest to us, namely
V1 ; V2 and V3 .

So; I1 V1 Y1 Y12  V2 Y12  V3 0

Equation 2.1

I2 V1 Y12 V2 Y2 Y12  V3 Y2

Equation 2.2

Exercise
Obtain the similar equations for
the currents at node 2.

The equation at node 3 is:

 I2 V1 0  V2 Y2 V3 Y03 Y2

Equation 2.3

Collect these equations together as shown:

I1 V1 Y1 Y12  V2 Y12  V3 0
I2 7V1 Y12 V2 Y2 Y12  V3 Y2
I2 7V1 0  V2 Y2 V3 Y03 Y2

2-46

SYSTEMS FUNDAMENTALS

Equation 2.4
Equation 2.5
Equation 2.6

Section 2

Exercise
Write the equations in matrix
format.

3 2
Y1 Y12
I1
4 I2 5 4 Y12
I2
0

Y12
Y2 Y12
Y2

32 3
0
V1
5
4
Y2
V2 5Equation 2:7
Y03 Y2
V3

2.8.4 The admittance matrix


Equation 2.7 is a correct representation of the conditions found in
Figure 2.25, but it is a particular case.
If we are to proceed to a general analysis of power system
networks for load flow or fault conditions, then we need a general
form of Equation 2.7 which we can apply to all circuits.
This is given below:

3 2
Y11
I1
4 I2 5 4 Y12
I3
Y13

Y12
Y22
Y23

32 3
Y13
V1
5
4
Y23
V2 5
Y33
V3

Equation 2:8

Note that the symbols now have a different meaning from those
defined in Figure 2.25 and that the negative signs are now
contained in the terms Y12, Y13 and Y23.
Thus the new term Y12, when referred to Equation 2.1, is given by

Y12new Y12

BASIC CONSIDERATIONS

2-47

Module 1

and

Y11new Y1 Y12
Y13new 0
Y22new Y2 Y12
Y23new Y2
Y33new Y03 Y2
The admittance term in Equation 2.8 is known as the Admittance
matrix.

2.8.5 Writing the admittance matrix


We can make some interesting deductions from the form of the
matrix equations which we derived in the last section.
Firstly, note the term in the row 1, column 1, position of the big
matrix. This is:

Y1 Y12
It is obviously linked with V1 and I1 both of which occur at node
1 in Figure 2.24, so look at node 1 in Figure 2.24 and you will see
that Y1 and Y12 are the admittances connected to node 1.
It would seem therefore that this could be a useful rule:
Put into position 1; 1 in the admittance matrix the sum of
all the admittances connected to node 1. This is the new
term Y11 of Equation 2.8.
Look now at position 2; 2 in the admittance matrix and see there
the sum of all the admittances connected to node 2. So we have
another rule:
Put into position 2; 2 in the admittance matrix the sum of
all the admittances connected to node 2. This becomes the
new term Y22 .
The rule works for node 3 and position 3; 3 as well, so we can
now use this general rule to write in the diagonal values in any
admittance matrix.
We move on to consider the term at the intersection of row 1,
column 2, and we see there the admittance connected between
node 1 and node 2, but with a minus sign before it. This value,
including the minus sign becomes Y12 .
2-48

SYSTEMS FUNDAMENTALS

Section 2

At the intersection of row 2, column 1, we have the admittance


connecting node 2 and node 1, but with a minus sign before it.
This becomes the new Y12 .
The general rule seems to be that the term at the intersection of
row K, column L is the admittance connected directly between
node K and node L, with a minus sign before it. This admittance,
including the minus sign, is represented by YKL .
Exercise
Prepare a brief summary of the
steps to be taken to obtain the
matrix equations for a network.

Summary: To obtain the matrix equation for the nodal analysis of


any circuit proceed as follows:
1

Convert all impedances to admittances.

Convert each voltage source and its associated series


admittance into a current source with equivalent parallel
admittance.

Number each node using zero for the chosen reference node.

Write down a current matrix containing the current-source


inputs at each node.

Write down an admittance matrix using the rules developed


above.

Write down a voltage matrix containing the node voltages

V1 ; V2 ; . . . Vn1
7

Form the matrix equation I Y V .

BASIC CONSIDERATIONS

2-49

Module 1

Exercise
Write down the matrix equation
for the circuit of Figure 2.26.

Figure 2.26

3 2
2
8
4 5 5 4 5
5
2

5
9
4

32 3
2
V1
4 54 V2 5
V3
9

2.8.6 Simple matrix manipulation


Matrix equations may be manipulated using the rules of matrix
algebra. For any but the most simple cases we would turn to a
computer for a solution, but it is useful to know the basic rules
and these are given below.
Addition and subtraction
This is easily demonstrated and the rules are clear:

 
a b
e

c d
g

 
f
ae

h
cg

bf
dh

and

2-50

k
m

SYSTEMS FUNDAMENTALS

 
l
p

n
r

 
q
kp

s
mr

lq
ns

Section 2

Multiplication
The order in which matrices are multiplied together is very
important. e.g.

A 6B
is NOT the same as

B 6A
We must accept another rule which says that:
two matrices can only be multiplied together when
the number of columns of the left-hand matrix equals
the number of rows of the right-hand matrix.
i.e. we can multiply the following:

a
4b
c

3

d
k
e 56
p
f

l
q

m
r

but we cannot multiply:

k
p

a
l
4
6 b
q
c

3
d
e5
f

The rule for multiplication is simply demonstrated as follows:

a
d

b
e

2

k
c
4
6 l
f
m

3


p
ak bl cm ap bq cr
5
q
dk el fm dp eq fr
r

BASIC CONSIDERATIONS

2-51

Module 1

Exercise
Carry out the following
multiplications.

2 4

6
a 4 3
5

6
b
9

7 6
2 1 564 4 2
5 4
3 3

 
6 4
4
6
9 6
6

7
15
2

a
2
2  16 25
6
4 3 8  5

6 8 20
9  4  4

3
8  4 10
7
12 2  2 5

5 12  15 15  6  12 20 3  6
2
3
11 34 14
6
7
4 6
1 12 5
2 3 17

2-52

SYSTEMS FUNDAMENTALS

Section 2

b

 

666  469 664 466
0 0

0
966  669 964  666
0 0

This is an interesting example of the difference between matrix


algebra and ordinary algebra. We would normally only expect a
product to be zero if one of the components were zero. In matrix
algebra it remains true that multiplying a matrix by the zero
matrix gives zero, but it is also possible to obtain zero by
multiplying together two matrices, neither of which is zero.
Inversion and division
To divide by a matrix, we multiply by the inverse of the matrix,
e.g.

R 6I V
becomes

R 1 6R 6I R 1 6V
or

I R 1 6V
1

where R

is the inverse of R

To obtain the inverse of a matrix we need to use the


DETERMINANT of the matrix, so we now need to define it. We
shall do so for the particular case of a 262 matrix.
The matrix is:

a
c

b
d

the determinant is written as:


a

c


b 
d

and its value is: ad  bc

BASIC CONSIDERATIONS

2-53

Module 1

Exercise
Find the value of the
determinant of the following
matrices.


a

5 4

3 2

5 4
3 2


7 6


b
c

4

The answers are:


(a)

562  463 2

(b)

10  12 2

(c)

35  24 59

The determinants of matrices of higher orders can be calculated,


but it is a tedious and drawn-out exercise which is best left to
computers.
The general procedure for inverting a 262 matrix then proceeds
as follows:
start with:

a b
c d

Interchange rows and columns to obtain:

a
b

c
d

Then replace each element with its minor, to obtain:

d
c

b
a

Change the signs of alternate elements row by row, starting the


first row with a positive and the second row with a negative, thus:

2-54

SYSTEMS FUNDAMENTALS

Section 2

d b
c
a

And finally, divide by the determinant of the original matrix to


obtain:

a
c

b
d

1



1
d b

a
ad  bc c

You will appreciate from this that inverting large matrices does
become an involved process so that we again resort to computer
solutions.
Further details of matrix algebra can be found in standard
textbooks on engineering mathematics (see Chapter 5 of Modern
Engineering Mathematics by Glyn James).
Exercise
Show that

5
7

6
8

1

4:0

3:5

Following the procedures given above, we have:

5
7

6
8

1


1
8

2 7

 
4:0
6

3:5
5

3:0
2:5

BASIC CONSIDERATIONS

2-55

3:0
2:5

Module 1

2.8.7 Solving a network equation


Let us assume that we have written the matrix equation for a
network in the form I Y V and assume that the values for I
are known (remember that these come from knowing the source
voltages and the series admittances).

Exercise
List the matrix operations that
we have to do to solve for the
nodal voltages.

We begin with the equation

I Y V
and we want to end up with

Y 1 I V
The matrix operations that we have to do are:

2-56

obtain the inverse of Y

multiply Y

expand the result to obtain the nodal voltages.

SYSTEMS FUNDAMENTALS

1

and I together, and

Section 2

Exercise
The matrix equation for a circuit
is written as

j1:4

0
j8

j3


j3
j2



V1
V2

Solve for the nodal voltages V1


and V2

We can rewrite the equation in the form:

j8
j3

j3
j2

1 

  
j1:4
V1

V2
0

Then calculate the value of the determinant


 j8

 j3


j3 
j8j2  j3j3 16 9 7
j2 

Then the inverse is arrived at as follows:

j8

j3

j3

j2

Starting with the original matrix, we begin by keeping constant


the elements on the diagonal passing from bottom left to upper
right and interchanging the remaining elements i.e.

j2
j3

j3
j8

The next step is to reverse the sign of the elements on the


diagonal passing through the bottom left and upper right; in this
case, the elements are both positive and so they become negative
i.e.

j2
j3

j3
j8

BASIC CONSIDERATIONS

2-57

Module 1

The inverse is found by scaling this matrix by the reciprocal of the


determinant i.e.

j8
j3

j3
j2

1


1 j2

7 j3

j3
j8

We now have:


1 j2
7 j3

j3
j8



  
j1:4
V1

V2
0

or,


  
V1
1 j26j1:4

7 j36j1:4
V2

  
j26j0:2
V1

j36j0:2
V2

  
0:4
V1

0:6
V2
or,

V1 0:4 v
V2 0:6 v
You might like to take this exercise further and convert the
original admittance matrix back to a network using the ideas of
subsection 2.8.3 and then solve, using standard methods, for the
voltages at the nodes.
The circuit should come out as follows:

Figure 2.27

2-58

SYSTEMS FUNDAMENTALS

Section 2

? Self-assessment question 2.1


(a)

Add together the voltages 311sinot , 311sinot  120


and 311sinot  240.

(b)

Calculate the voltage across the impedance


Z 10 j20O when a current I 5 j10 A flows
through it.

(c)

An impedance of 5 j7O is connected in parallel with


an impedance of 6  j9O. The combination is in series
with a third impedance of value 9 j12O. Calculate the
total effective impedance.

(d)

If an a.c. voltage of 240 V is applied across the


combination, calculate the current in each of the parallel
branches.

? Self-assessment question 2.2


The following loads are supplied from one substation busbar:

4000 kW at 0:6 power factor lagging


3000 kW at 0:9 power factor lagging
2000 kW at 0:8 power factor lagging, and
3000 kW at 0:9 power factor leading.
Determine the power factor of the busbar current.

? Self-assessment question 2.3


A balanced three-phase source supplies two loads in parallel. The
first consists of three equal impedances of 3 j6O connected in
a delta arrangement. The second consists of three equal
impedances of 1 j2O connected in a star arrangement.
One phase of the three-phase source goes open circuit.
Determine the value of the impedance that now exists between
the two remaining phases.

BASIC CONSIDERATIONS

2-59

Module 1

? Self-assessment question 2.4


A 50 kW induction motor operates at full load with an efficiency
of 0:9 and a power factor of 0:9 lagging. At half load it operates
with an efficiency of 0:75 and a power factor of 0:6 lagging.
Capacitors are used to improve the power factor to 0:8 at half full
load. Determine the power factor of the supply at full load if the
capacitors are kept in circuit.

? Self-assessment question 2.5


A three-phase 600 kW load has a power factor of 0:6 lagging. The
supply power factor has to be raised to 0:8 so a three-phase
transformer is connected across the supply. A three-phase bank of
capacitors is then connected across the secondary winding of the
transformer in a delta arrangement. The secondary of the
transformer and the capacitors are rated at 3:3 kV.
Find the capacitance value of each capacitor necessary to raise the
supply power factor to 0:8 lagging and the VA rating of the
transformer.

? Self-assessment question 2.6


Figure 2.28 represents four generators interconnected by four
busbars. Each generator is rated at 11 kV, 300 MW and each has a
series reactance, Xs of 0:1 p.u.. The interconnecting busbars each
have a reactance of 0:4 O.
Calculate the short circuit MVA for a balanced three-phase fault
on the busbar fed directly by G4.

2-60

SYSTEMS FUNDAMENTALS

Section 2

Figure 2.28

? Self-assessment question 2.7


Figure 2.29 shows a single-phase equivalent circuit for a
distribution network fed from both ends. Voltages are given as per
unit values and circuit parameters are shown as per unit
admittances.
Obtain the nodal admittance matrix for the network.

Figure 2.29

BASIC CONSIDERATIONS

2-61

Module 1

2-62

SYSTEMS FUNDAMENTALS

Section 3
TRANSFORMERS AND SWITCHGEAR

3.0 Objectives
At the end of this Section you should be able to:

use the equivalent circuit of a transformer to represent the


effect of a transformer connected to a larger network

explain the effects of the transformers magnetic circuit on


transformer parameters and transformer behaviour

estimate ratio and phase errors in current transformers

develop approximate models for three-winding and for auto


transformers

explain the principles of arc control in switchgear

determine the approximate values of restriking voltage and


rate of rise of restriking voltage for switches clearing fault
currents at current zero.

3.1 The transformer

3.1.1 Introduction
At first sight the transformer appears to be such a straightforward
piece of equipment that it hardly deserves detailed study. It is, for
example, such a very efficient device that it is tempting to assume
that it is lossless, but on a large transformer the power loss can
amount to 5 MW which presents the designer with a problem of
how to dissipate the associated energy which occurs in the heart
of a thermally insulated mass. The transfer characteristics are also
inherently non-linear because at the centre of the transformer is a
steel core which shows magnetic hysteresis and saturation. Under
TRANSFORMERS AND SWITCHGEAR

3-1

Module 1

normal operating conditions these phenomena will cause a small


distortion of the waveforms of current and voltage and contribute
to the general losses of the device, but a move away from normal
operating conditions can take the device into magnetic saturation
with very noticeable effects on the operating characteristics.
The purpose of this section is to review the basic theory of the
transformer and to use it to explain some of the less obvious
effects which can be such a nuisance to the electrical power
engineer.

3.1.2 Symbols and conventions


In order to avoid confusion over signs and phase shifts we shall
define our terminology very carefully. In general we shall use
lower case letters to represent instantaneous values and capital
letters to represent r.m.s. and maximum values.

v1
e1
v2
e2
i0
F
V1
V1M
V2
V2M
E

instantaneous voltage applied to primary winding


instantaneous induced e.m.f. in primary winding
instantaneous voltage across secondary terminals
instantaneous e.m.f. induced in secondary winding
instantaneous magnetising current in primary
instantaneous mutual flux linking primary and secondary.
r.m.s. voltage applied to primary
maximum voltage applied to primary
r.m.s. voltage across secondary terminals
maximum voltage across secondary terminals
r.m.s. value of e.m.f.

The conventions that we shall adopt for the assumed directions of


positive voltage, current, flux and e.m.f. are clearly shown in
Figure 3.1.

Figure 3.1

3-2

SYSTEM FUNDAMENTALS

Section Three

3.1.3 The transformer effect


If we apply a time-varying voltage v1 to the primary winding of
the circuit shown in Figure 3.1 a current i0 will flow in the N1
turns of the winding to establish a flux in the magnetic circuit.
The flux is known as the mutual flux because it links both the
primary and the secondary coils. There will be an induced e.m.f.
e1 in the primary winding and an induced e.m.f. e2 in the
secondary winding and both will be proportional to the rate of
change of flux.
If v1 is rising positive then e1 and e2 will also be rising positive in
the sense indicated in Figure 3.1. Neglecting resistance, we can
write:

v1 e1 N1

dF
dt

and

e2 N2

dF
dt

The e.m.f. e2 appears across the secondary terminals as the


terminal voltage v2 and it follows that:

v1 N1

v2 N2
Expressed differently, this tells us that the voltage per turn is the
same for each winding i.e.

v1
v2
dF

N1 N2
dt
The electrical stress between adjacent turns is therefore similar on
the high and low-voltage windings.

3.1.4 The transformer e.m.f. equation


If we assume that the flux in the core is sinusoidal and can be
represented by:

F FM sinot
then

TRANSFORMERS AND SWITCHGEAR

3-3

Module 1

v1 e1 N1

dF
N1 oFM cosot
dt

and

v2 e2 N2 oFM cosot
We see that v1 and v2 are in time-phase and lead the flux phasor
by p/2 rad as shown in Figure 3.2.
From the two previous equations we can write:

1
V1 p N1 2pf FM 4:44f FM N1
2
and

V2 4:44f FM N2
We note in particular that the voltage is directly proportional to
the frequency and to the maximum value of the mutual flux. For a
given transformer, in which N1 and N2 are fixed, the level of flux
in the core is determined by the applied voltage and by the
frequency. It is not determined by current flow.

Figure 3.2

3-4

SYSTEM FUNDAMENTALS

Section Three

Exercise
A 275 kV:132 kV transformer
has a tap change mechanism at
the top end of the high-voltage
(primary) winding which can
increase or decrease the number of
turns by 10% from the nominal
value. The tap is set at its lowest
value and the transformer is
connected on the high-voltage side
to a supply of 275 kV. The tap is
then set to its highest value, but
the supply is maintained at
275 kV.
a) For each tap setting +10% and
10%, calculate the low-voltage
output.

a) Case 1 (nominal)

b) Calculate the percentage change


in the maximum value of the core
flux. Assume zero winding losses.

V1 N1
(see page 3-3)
=
V2 N2
275 N1

=
= 2.083
132 N2
Case 2 (10% tap)

275 0.9N1
=
V2
N2
275
i.e.
= 0.9 2.083
V2
V2 = 146.7 kv
Case 3 (+10% tap)

275 1.1N1
=
V2
N2
275
i.e.
= 1.1 2.083
V2
V2 = 120.0 kv
b) The change at the low-voltage terminal is from 146.7 kV to
120 kV.

TRANSFORMERS AND SWITCHGEAR

3-5

Module 1

or

26:7
6100 18:2%
146:7

The core flux will also decrease by 18:2% provided that all
relationships remain linear, i.e. no magnetic saturation or
hysteresis.

3.1.5 Power transfer across a transformer


If a circuit is connected across the terminals of the secondary
winding as shown in Figure 3.3 then a current i2 will flow as
shown. The magnetomotive force (m.m.f.) produced by i2 flowing
in the secondary turns N2 is balanced by a current i1 flowing in
the primary turns N1 , so that:

i1 N1 i2 N2
The mutual flux remains at the original value determined by the
voltage and frequency.
The total primary current consists of i1 and i0 , where i0 is the
magnetising current.
The current i1 is determined by the above equation and i2 is
determined by the secondary voltage v2 and the secondary load
impedance Z2 i.e.

i2

v2
Z2

Since

i1 N1 i2 N2
it follows that

v1 i1 v2 i2
or, the volt-ampere input equals the volt-ampere output.

3-6

SYSTEM FUNDAMENTALS

Section Three

Figure 3.3
This relationship can be extended to a multi-winding transformer
to give:

v 1 i1 v2 i2 v 3 i3 . . . v n in :

Exercise
A third winding is wound on the
transformer core shown in Figure
3.3 to produce a voltage v3 .
Calculate the power supplied by
the source at v1 for the
following conditions:
Winding 2 supplies a load of
600 MW at 230 kV and power
factor 0:9 lagging;
Winding 3 supplies a load of
150 MW at 13 kV and power
factor 0:8 lagging.
Neglect the magnetising current.
The secondary load is 600 j290 MVA
The tertiary load is 150 j112 MVA
The total VA required from the source is therefore:

750 j402 MVA


or

750 MW at 0:88 power factor lagging

TRANSFORMERS AND SWITCHGEAR

3-7

Module 1

3.1.6 Impedance transfer across a transformer


For the arrangement shown in Figure 3.3, the secondary current is
given by:

i2

v2
v2
or
Z2 :
Z2
i2

If we convert the voltage and current values to primary values,


then we obtain:

 2
v1
N2
v1
N2
6N 2 6
6
Z2
N1
i1 N 1 i1
N1
or

where

v1
Z2 6N 2
i1
N1
N
N2

We write Z2 N 2 as Z20 , or Z21 and it is the effective impedance


that the primary winding sees. We can therefore place the load
either on the secondary side of the equivalent circuit as an
impedance Z2 or on the primary side of the equivalent circuit as
an impedance Z2 N 2 .

3.1.7 The transformer as a mutually coupled circuit.


So far we have ignored the effect of winding resistance and the
non-linearity of the magnetic characteristic.
If we continue to do so then we can show that Figure 3.4 will
serve as a simple equivalent circuit for a transformer. In this figure,
L1 and L2 are self inductances and M is the mutual inductance
between coils.

Figure 3.4
The two dots shown above the windings in Figure 3.4 are there for
a purpose. It is conventionally assumed that when a current enters
3-8

SYSTEM FUNDAMENTALS

Section Three

a terminal marked with a dot in a mutually coupled circuit it


produces an e.m.f. in the coupled circuit which tends to drive a
current out of the dot on the coupled circuit.
We can now write down the voltage equation for the primary
winding of Figure 3.4 as follows:

d
d
i1 i0  M i2
dt
dt


di0
di1
di2
L1
M
L1
dt
dt
dt

v1 L1

Exercise
Using the definitions of
inductance,

l1
and
i1
l2
M
i1

L1

(where l is the flux linkage),


and the known ratio of the
primary and secondary currents,
show that

di1
di2
M
0
L1
dt
dt

The flux linkage l1 is equal to N1 F


The flux linkage l2 is equal to N2 F so that the expression in
brackets becomes:

N1 F di1 N2 F di2

i1 dt
i1 dt
However:

N1 i1 N2 i2
so that:

N1

di1
di2
N2
dt
dt
TRANSFORMERS AND SWITCHGEAR

3-9

Module 1

giving:

N2 F di2 N2 F di2

0
i1 dt
i1 dt
We can therefore accept that:

v1 L1

di0
dt

This is useful because we can argue that the primary voltage v1 is


balanced by a primary e.m.f. e1 produced by the magnetising
current i0 flowing into the self-inductance L1 of the primary
winding. We can then separate the currents i0 and i1 and show
them as flowing in separate paths in our equivalent circuit, as
illustrated in Figure 3.5.

Figure 3.5

3.1.8 The full equivalent circuit


This is shown in Figure 3.6. The effect of winding resistance is
easily represented by resistors R1 and R2 and losses in the
magnetic core are represented by Rm . Because flux coupling
between the two windings is not perfect, due to leakage fluxes
from primary and secondary coils, leakage inductances LL1 and
LL2 are included in primary and secondary circuits respectively.
The e.m.f. e1 is shown appearing across an inductance Lm1 which
we call the magnetising inductance.
It can be shown that these inductances can be related to the selfinductances L11 and L22 of the primary and secondary windings
and to the mutual inductance L12 as follows:

3-10

SYSTEM FUNDAMENTALS

Section Three

LL1 L11  Lm1


LL2 L22 
and

Lm1

N22
Lm1
N12

N1
L12
N2

It is more usual to work in terms of reactances and the above two


equations are then written as:

X1 X11  Xm
N1
Xm
2pfM or
N2

N1
2pfL12
N2

This leads to the equivalent circuit of Figure 3.7 where the


magnetising branch has been moved to the left of the primary
circuit components. The advantage of this in the simple
equivalent circuit is that we can assume that all magnetising
current is now supplied directly from the supply.

Figure 3.6

Figure 3.7

TRANSFORMERS AND SWITCHGEAR

3-11

Module 1

Exercise
Two windings on a simple
toroidal core are shown to have
self inductances of 80 H and
20 H with a mutual inductance
of 35 H. Draw an equivalent
circuit and attach values for
components assuming that the
80 H coil is the primary,
N1 1 000 turns and
N2 500 turns.

Figure 3.8
The magnetising inductance

Lm1 2635 70 H
The primary leakage inductance

LL1 80  70 10 H
The secondary leakage inductance

LL2 20 

70
2:5 H
4

The power engineer tends to be concerned with networks having a


number of transformers in circuit and it is much easier to handle
the analysis of these networks using per unit notation. This is
explained in the previous section. The benefit of using per unit
values for transformer components is that they may be transferred
from side to side without change and the ideal transformer of the
equivalent circuit becomes a 1:1 transformer.

3-12

SYSTEM FUNDAMENTALS

Section Three

Exercise
A single-phase two-winding
transformer has a primary
impedance of 0:017 j0:07
p.u. and a secondary impedance
of 0:02 j0:08 p.u. The
magnetising impedance is
sufficiently large to be ignored.
Draw the equivalent circuit.

Figure 3.9
Open- and short-circuit tests
The impedance values to be inserted into the equivalent circuit
can be obtained from tests.
If we open circuit the secondary winding of the transformer
represented in Figure 3.7 and apply a voltage V 1 to the primary
then the only current that can flow will be the magnetising
current I m . The values of Rm and Xm can be obtained from test
measurements.
For example, a single-phase transformer has its rated voltage,

6:325 kV, applied to the primary winding. The secondary winding


is open circuit. A current of magnitude 10 A is recorded on the
primary side and the power input is measured as 4 kW.
The power consumed by the magnetic circuit is

V2
4 kW:
Rm
So,

Rm

6:3252 6106
10:0 kO
46103

The total current into the parallel branch is 10 A and this flows
into the parallel combination of Xm and Rm .

TRANSFORMERS AND SWITCHGEAR

3-13

Module 1

Thus:

I O = 10 =

IR + Im

V2 V2
+
Rm2 Xm2

From which

I o2
1
1
=
2
2
Xm
V
Rm
Substitution then gives

1
= 1.58 10 3 s
Xm
Xm = 0.633 103
If we now short circuit the secondary winding and raise the
voltage on the primary winding until full load current flows in
the secondary, then we can calculate the series equivalent
impedances. In this case the voltage applied will be very low
because of the short-circuit condition so that the current flowing
into the parallel circuit will also be very small and so much less
than the full-load current that it can be safely ignored. All input
power to the circuit will therefore be consumed in the series
impedances.
For example, the single-phase transformer with a rated primary
voltage of 6:325 kV provides the following test results on short
circuit:

We assume that the real power loss is entirely in the series


resistance of the windings and we assume that this can be
represented by one equivalent resistance R1 in the primary
equivalent circuit.
Then we have:

The whole of the applied voltage V1 is dropped across the primary


equivalent impedance R1 + X1, so that:

3-14

SYSTEM FUNDAMENTALS

Section Three

q
V 1 I 1 6 R21 X21
s s
 2

2
V1
0:66103
2
X1
R1
0:172 2:0 O
I1
300
The full equivalent circuit for the transformer is shown in Figure
3.10. All impedance values are shown as effective primary values.

Figure 3.10

3.1.9 Non-linear effects of the magnetic core


Magnetic phenomena
There are four strange phenomena associated with magnetic
materials that are important to the power engineer. The first and
most obvious is ferromagnetism itself by which an applied
magnetising force is enhanced to produce magnetic flux densities
in the material which would not occur if the space were occupied
by any non-ferromagnetic material.
Can you recall the other three phenomena?

Hysteresis

Saturation

Magnetostriction.

Let us deal with magnetostriction first - and briefly!


A ferromagnetic material increases its length in the direction of an
applied magnetic field. The extension is not large, perhaps 5 mm
in a metre length, but it means that a transformer core is in
continuous movement when excited by a time-varying field. If the
laminations in the core are clamped to restrict longitudinal
TRANSFORMERS AND SWITCHGEAR

3-15

Module 1

changes in length then transverse vibrations are established. The


most noticeable result of this is the generation of audio noise with
a fundamental frequency of twice the exciting frequency and
significant harmonics up to about 800 Hz.
Hysteresis and saturation are usually represented together on the
typical BH characteristic for a ferromagnetic material. A typical
example is shown in Figure 3.11.

Figure 3.11
The magnetising force H is proportional to the magnetising
current.
The flux density B determines the flux level and since, in a
transformer, the e.m.f.s produced are proportional to the rate of
change of flux we see that B is closely related to the applied
voltage.
It is usual to represent the BH loop by the curve OC shown in
Figure 3.11 and known as the magnetisation characteristic. The
area of the BH loop is proportional to the energy loss per cycle of
magnetisation and it makes sense to use transformer steels which
have small BH loops and so restrict magnetisation losses.
Effects on current and voltage waveforms
The magnetisation characteristic is non-linear and a typical curve
for a grain-oriented transformer steel is shown in Figure 3.12.

3-16

SYSTEM FUNDAMENTALS

Section Three

Figure 3.12
The effect of this on the current waveform can be easily
demonstrated with a simple graphical construction.
We assume a sinusoidal voltage source

v1 V1M sinot
applied to the primary winding of a transformer made of steel of
the type represented by Figure 3.12.
The flux density in the transformer must be given by:

F
1
B
A N1 A

v1 dt 

V1M
cosot
AoN1

or

B B1M cosot
Let us assume that conditions give us a maximum value of

B1M 1:6 T
Figure 3.13 shows how we can map across from the B1M curve to
the i0 curve using the magnetisation curve as the function relating
B and i.
The distortion of the magnetisation current is very apparent and it
is also clear that the distortion is significantly increased by
allowing the flux density to run into magnetic saturation levels.
The magnetisation current can be analysed using Fourier analysis
and will be shown to contain a large 3rd harmonic component.

TRANSFORMERS AND SWITCHGEAR

3-17

Module 1

This raises an interesting question. From where does the 3rd


harmonic current come?

Figure 3.13
A voltage source of single frequency cannot support a current of
any other frequency. Take for example the simple circuit of Figure
3.14 and write down the equation for current.

Figure 3.14

v
sinot
R

i is clearly sinusoidal at the angular frequency o and cannot have


a third-harmonic component unless the voltage also has a thirdharmonic component.
So the transformer, which demands a third-harmonic current, has
to find a means of generating a third-harmonic voltage if no thirdharmonic is available in the supply voltage.
What happens is that the magnetising current which must be
sinusoidal initially, forces the magnetising force H into a
3-18

SYSTEM FUNDAMENTALS

Section Three

sinusoidal excitation. The flux density is therefore driven to be


non-sinusoidal so that a primary e.m.f. is produced which is itself
non-sinusoidal.
There is then an unbalance between the applied voltage
V1M sinot and the distorted e.m.f. E1 sinot E3 sin3ot (say).
The induced e.m.f. E3 sin3ot now produces the third-harmonic
current that the transformer needs and i0 settles back almost to
where it should be. All this happens virtually instantaneously and
causes no upset to the operation of the transformer, but what we
learn from it is that there will always be a slight distortion of the
e.m.f.s produced by any transformer. A pure sinusoid of current or
of voltage does not exist in an iron-cored transformer.
Magnetic inrush
This is a common problem which occurs when transformers are
switched on and which can lead to maloperation of protective
gear unless the protection has been designed to recognise, and
respond to, harmonic inrush currents.
We can explain the effect as follows:
We know that if the voltage and flux waveforms in a transformer
are assumed to be sinusoidal then the voltage phasor leads the flux
phasor by p=2 rad as shown in Figure 3.15a. Let us examine
conditions at ot p=2. The voltage is zero, going negative. The
flux it produces must have an instantaneous rate of change equal
to zero and growing in magnitude, but negative in sign. Note that
it is the change of flux with time that matters and that the
absolute magnitude of the flux is irrelevant. The flux variation
shown in Figure 3.15b would satisfy the requirement as effectively
as the flux of Figure 3.15a.
If we move our time origin in Figure 3.15 to

ot p=2
then in Figure 3.15a

F FM cosot
and in Figure 3.15b

F FM cosot  1
Differentiating either expression with respect to time yields the
same answer for v1 =N1 .

TRANSFORMERS AND SWITCHGEAR

3-19

Module 1

Figures 3.15a and 3.15b


Imagine then that we were to switch the transformer on at the
instant when the supply voltage was zero, going negative. The flux
must immediately try to follow the pattern defined in Figure 3.15,
but if the transformer core is initially demagnetised then the flux
must start at zero and only the flux change of Figure 3.15b is
possible. A well-designed transformer would be just entering the
core saturation region when the flux reached its normal maximum
value, so the flux excursion of Figure 3.15b would push the core
well into saturation.
The effect on the magnetising current is illustrated in Figure 3.16.
The normal operating flux Fm would be reached about mid-way
between times t1 and t2 and by tracking across to the H , B
curve and onto the io curve you should be able to show that the
value of io is about one-fifth the value of the peak io , or
alternatively the peak current is five times the value of the normal
magnetising current. This is known as magnetic inrush current.

3-20

SYSTEM FUNDAMENTALS

Section Three

Figure 3.16
It is quite possible for the peak value of the inrush current to
exceed the peak value of the normal load current although it will
usually decay to its normal value of about 2% of the normal load
current within 10 cycles of a.c. The problem with it is not so
much the magnitude as the fact that it occurs only on the primary
side of the transformer or more precisely on the excited side so
that protection equipment will record an unbalance condition and
interpret it as an internal fault.

3.1.10 The current transformer


The principle of operation of the current transformer, or CT, is
exactly the same as for any other type of transformer. The reason
it is subject to a different form of analysis is due entirely to the
way it is connected into the system.
The typical power transformer is connected across a voltage source
and operates in parallel with the system. It is therefore voltage
driven and conditions in the core are determined by the applied
voltage and the supply frequency. Load current, or secondary
current, is determined by the secondary load.
The current transformer is by contrast connected in series with the
supply system so that its primary current is determined by the
current in the supply lines. The secondary current is then
determined by the turns ratio and this current is driven into the
secondary load, often known as the burden, irrespective of the size
of that burden.
Since a current transformer is intended to provide a secondary
current which is a reduced reflection of the primary current, the
turns ratios are high and N 2 : N 1 can be in excess of 75000 : 1.
TRANSFORMERS AND SWITCHGEAR

3-21

Module 1

For example, suppose we have a CT which is designed to produce


5 A in the secondary winding when the normal line current is
flowing through the primary. A secondary burden of 0:6 O will
have a voltage of 3 V developed across it and the secondary rating
will be 3 6 5 15 VA.
If we change the burden to 1 O, then the VA rating increases to
25 VA, but the secondary current remains at 5 A.
These values are typical for CTs and with such low burdens it is
clear that the resistances of windings and leads become critically
important and cannot be ignored as in the power transformer.
Voltage drops on the primary side are of no real significance to the
operation of the CT and we can assume an ideal primary winding
and allow the magnetising component of current to be drawn
from the secondary. This leads to the equivalent circuit of Figure
3.17.

Figure 3.17
The secondary e.m.f. e2 provides the magnetising current im , with
an in-phase component to provide core losses and a quadrature
component to produce the core flux.
The current

i02 i2  im
so that the current into the burden, i02 , may be limited and
distorted if the core goes into saturation and im increases. The
equivalent phasor diagram is built up as in Figure 3.18, starting
with E2 as the reference phasor.

3-22

SYSTEM FUNDAMENTALS

Section Three

Figure 3.18
The effect on the secondary current of im can be seen clearly. It
changes the magnitude from an apparent i2 to an actual i02 and

changes the phase of the current by y .
CTs used for power system protection purposes should have small
errors, low magnetising current and should not saturate over the
intended range of load and fault current.
There are two possible sources of errors which can be seen from
Figure 3.18. One is the change in effective turns ratio from i1 =i2
to i1 =i02 and the other is a phase shift in the secondary current
from a2 y to a2 .

Figure 3.19
Referring to Figure 3.19, provided y is small, the magnitude of the
difference between i02 and i2 is im cosam  a2 so that the
current ratio error can be defined as:

Current error

nominal ratio  actual ratio


6100%
actual ratio

N  Ni2 =i02
Ni2 =i02

i02  i2
i2

im cosam  a2
i2
TRANSFORMERS AND SWITCHGEAR

3-23

Module 1

Again from Figure 3.19, provided y is small:

y siny

im sinam  a2
rad
i2

Exercise
A current transformer operates
with 1 000 A in the primary
and a nominal turns ratio of
1 : 200. The magnetising
current required on the
secondary side is 0:20:45 A
and the impedance of the burden
and the secondary winding
together is 0:7 j0:4 O.
Estimate the ratio error and the
phase angle error.

Solution:

im 0:245 A
i2 5a2 y A
a2 is defined by the total secondary impedance
0:7 j0:4 0:8129:7 O

Ratio error

0:2
cos45  29:7 6100% 3:86%
5

Phase error

0:2
sin45  29:7 0:010 6 rad
5

or

0:6

3.1.11 Three-phase transformers


We examined three-phase systems in Section 2.4 and showed that
as we connect supplies in star and delta arrangements we create
phase shifts between line and phase voltages and between line and
phase currents. These phase-shifts are critically important when we
interconnect three-phase transformers and supply systems.
We can demonstrate the problem by examining the winding
arrangements of the two star/star transformers represented in
Figure 3.20.
3-24

SYSTEM FUNDAMENTALS

Section Three

Figure 3.20a
In Figure 3.20a the secondary windings are not connected and the
voltage Va1a2 in the secondary a winding illustrates the direction
of the voltage rise in the winding at a particular instant.
We can connect the three windings in star format as shown in
Figure 3.20c with the winding ends a2 ; b2 and c2 connected to
ground. The terminal a1 in Figure 3.20c is the secondary output
terminal for the a phase.

TRANSFORMERS AND SWITCHGEAR

3-25

Module 1

Figure 3.20b
Figure 3.20b illustrates an identical three-phase transformer with
its secondary windings unconnected and the secondary voltage
V a1a2 is again shown for a particular instant.

Figure 3.20c
In Figure 3.20d the secondary winding is connected in a star
arrangement, but this time we connect terminals a1 ; b1 and c1 to
common ground and the terminal a2 becomes the secondary
output terminal for the a phase.

3-26

SYSTEM FUNDAMENTALS

Section Three

Figure 3.20d
To the uninitiated these two transformers are identical because
each is wound in a star/star formation, but from the work that we
did in Section 2.4 we know that if the primary windings are
connected in parallel to the same supply then at the instant that
terminal a1 in Figure 3.20c is at Va1a2 above neutral, terminal
a2 in Figure 3.20d is at Va1a2 below neutral. In other words the
two terminals have a voltage of 26V between them. If we
connect these two terminals together we create a short circuit
across the windings with only the winding impedance to limit the
current. Figure 3.21 illustrates the problem.

Figure 3.21
It is vitally important therefore that when we connect
transformers in parallel we check not only the voltage levels at the
terminals, but also the phase displacement between the voltages.
The two star/star transformers in Figure 3.22 can be safely
connected in parallel by joining a1 to a1 ; b1 to b1 and c1 to c1 .
The transformers in Figure 3.23 must not be connected in parallel.

TRANSFORMERS AND SWITCHGEAR

3-27

Module 1

Figure 3.22

3-28

SYSTEM FUNDAMENTALS

Section Three

Figure 3.23
We avoid confusion by using a designation, which shows the
phase shift that a particular winding configuration generates. A
star/star/0, or Yy0, transformer is connected in star on primary and
secondary sides and produces no phase shift from primary phase
to secondary equivalent. This would apply to both transformers of
Figure 3.22.
A star/star/6, or Yy6, transformer is connected in star on the
primary and secondary sides and produces a phase shift of 1808
from primary phase to secondary equivalent. We imagine the
secondary phasor in the a phase at 6 Oclock (1808) when the
primary equivalent phasor is at 0.
We will get phase shifts in star/delta and delta/star transformer
connections and these will be +30 depending on the
connection. The common arrangements are illustrated in Figure
3.24.

TRANSFORMERS AND SWITCHGEAR

3-29

Module 1

Figure 3.24

Exercise
The phase windings of a threephase transformer are
represented diagrammatically in
Figure 3.25. Make the necessary
connections between terminals to
produce a Dy1 arrangement.

Figure 3.25
We need to create the phasor diagram of Figure 3.26. To do this
we make the connections of Figure 3.27.

3-30

SYSTEM FUNDAMENTALS

Section Three

Figure 3.26

Figure 3.27

3.1.12 Parallel operation of transformers


We saw in the previous section that before we connect two threephase transformers in parallel they must have the same phase shift
otherwise we create short-circuit currents around the connections.
It is also obvious that we must have the same voltage ratios.
One further condition needs to be considered and that is the
relative value of the transformer impedances. The point is best
illustrated by an example.
Let two single-phase transformers share a load of 2 MVA at a
power factor of 0:9 lagging. Transformer A is rated at 2 MVA and
has an impedance of 0:01 j0:05 p:u. Transformer B is rated
at 1 MVA and has an impedance of 0:008 5 j0:045 p:u. If
the transformers have equal open-circuit voltage ratios, find out
the load supplied by each and its power factor.
We convert all values to a common MVA base and take 2 MVA as
the reference value.

TRANSFORMERS AND SWITCHGEAR

3-31

Module 1

Then

ZA 0:01 j0:05
ZB 0:017 j0:09
MVA 1:025:8
If we use a single line equivalent circuit then the transformers
appear as two parallel impedances in series with the supply. Figure
3.28 illustrates the case.

Figure 3.28
The voltage across the parallel combination is:

I L6

ZA ZB
ZA ZB

The current IA is then:

IA IL6

ZB
ZA ZB

I B I L6

ZA
ZA ZB

and

We see immediately that the current, and therefore the load, is


shared by the transformers in the inverse ratio of their
impedances.

I A MVAA ZB

I B MVAB ZA
So the load is not shared equally!

3-32

SYSTEM FUNDAMENTALS

Section Three

The load taken by transformer A is:

ZB
ZA ZB
0:091 6
25:8 79:3  79:08
1:06
0:142 6
0:64226:02 p:u:

MVAA 1:025:8 6

The load taken by transformer B is:

ZA
ZA ZB
0:051
1:06
25:8 78:7  79:08
0:142 6
0:35825:42 p:u:

MVAB 1:025:8 6

So,

MVAA 1:154 MW 0:563 Mvar


MVAB 0:647 MW 0:307 Mvar
and the total is:

MVA 1:801 MW 0:87 Mvar


2:0025:78 MVA

3.1.13 Three-winding transformers


These may be used specifically to interconnect three circuits at
different voltages or a third winding, which is often referred to as
a tertiary winding, may be introduced primarily to suppress
harmonic voltages or to limit unbalance in three-phase starconnected transformers. In this second case, the rating of the
tertiary winding is determined by the need to accommodate large
currents that might flow under unbalanced fault conditions and it
will usually exceed the rating level required for harmonic
suppression. There is therefore an excess capacity which can be
used to provide an auxiliary supply and in many instances it is
standard practice to design the third winding as an auxiliary
supply and to accept the benefits of harmonic suppression and
limitation of voltage unbalance as a bonus.
The simplest equivalent circuit is arranged as a three-phase starconnected equivalent and only one phase is shown as in Figure
3.29.
TRANSFORMERS AND SWITCHGEAR

3-33

Module 1

Figure 3.29
For convenience, all values are given in per-unit. The base values
for voltages are the nominal rated values for each winding and a
common base volt-ampere rating is selected.
The impedances Z1 , Z2 and Z3 are not the resistance and leakage
reactance of each winding. They are theoretical values computed
from the separate values of each transformer and are determined
as follows.
Standard open-and short-circuit tests on each two-winding
combination will give values for the impedances Z12 , Z23 and
Z13 . These are the values that would go into any equivalent circuit
for a two-winding transformer made up of the windings 1 and 2,
2 and 3, and 1 and 3 respectively.
However, when the three windings are connected as a threewinding transformer we use the equivalent circuit of Figure 3.29
and it can be seen that the effective impedance between terminals
1 and 2 with 3 open circuit is Z1 Z2 .
Thus Z12 Z1 Z2 and similarly

Z23 Z2 Z3
Z13 Z1 Z3
Rearranging these equations gives:

1
Z12 Z13  Z23
2
1
Z2 Z12 Z23  Z13
2
1
Z3 Z13 Z23  Z12
2
Z1

3-34

SYSTEM FUNDAMENTALS

Section Three

Exercise
A three-phase, three-winding
transformer is rated as follows:
Primary: 300 MVA, 11 kV
Secondary: 300 MVA, 132 kV
Tertiary: 50 MVA, 11 kV.
Short circuit tests provide the
following data:
X12 0:1 p:u:
on a 300 MVA base
X13 0:16 p:u:
on a 50 MVA base
X23 0:14 p:u:
on a 50 MVA base
Determine the values of Z1 , Z2
and Z3 for insertion in an
equivalent circuit using a base
MVA of 300.

All values need to be to a common VA base, so X13 and X23 are


converted to give:

X12 0:1 p:u:


300
0:96 p:u:
50
300
0:84 p:u:
X23 0:146
50
1
Then : Z1 0:1 0:96  0:84 0:11 p:u:
2
1
Z2 0:1 0:84  0:96 0:01 p:u:
2
1
Z3 0:96 0:84  0:1 0:85 p:u:
2
X13 0:166

Note from the above that the equivalent circuit can have negative
impedances in it.

TRANSFORMERS AND SWITCHGEAR

3-35

Module 1

3.1.14 The autotransformer

Figure 3.30
The autotransformer is represented by the equivalent circuit of
Figure 3.30. It is significantly different from the two-winding
transformer in that a direct electrical connection is maintained
between primary and secondary circuits so that part of the energy
transfer is by current flowing from primary to secondary through
the upper part of the winding. This arrangement reduces the
leakage impedance of the transformer compared to that of the
two-winding equivalent which results in a reduced series voltage
drop, but causes an increase in the short circuit current.
Using the symbols defined in Figure 3.30 we can write:

VL V2
VH V2 V1
and, since V1 and V2 are linked by the core flux:

V1 V2

N1 N2
Thus

VH V2

N1 N2
N1 N2
VL
N2
N2

The nominal transformer ratio is therefore:

VH N1 N2

VL
N2
3-36

SYSTEM FUNDAMENTALS

Section Three

To a good approximation we also have:

iL N1 N2

iH
N2

Exercise
A two-winding transformer rated
at 100 kVA, 10 kV:5 kV is
connected as an autotransformer
to provide 10 kV from a 15 kV
supply. Calculate the new rated
kVA output.

On rated load:

100
20 A
5
100
10 A
i2
10

iH i1

The rated load current is therefore:

iL i1 i2 30 A
kVA output 30610 300 kVA
This demonstrates the gains that can be made using
autotransformers, although in practice we would have to uprate
the insulation on the 5 kV winding in order to make the above
connection. Generally an autotransformer is smaller, cheaper,
more efficient and has a lower regulation than the equivalent twowinding transformer.

TRANSFORMERS AND SWITCHGEAR

3-37

Module 1

3.2 Switchgear

3.2.1 The arc


When we open a switch in an electric circuit what we are doing is
inserting a high impedance into the circuit. Any current that was
flowing before opening the circuit will tend to continue flowing
and two effects will be noticed. The first effect will be the heating
of the high impedance path as a consequence of i2 R losses. The
second effect will be the generation of an induced e.m.f. across the
impedance, partly because of the iR drop, but mainly because the
current will be rapidly reducing and there will be an induced
e.m.f., L di/dt , which will appear across the high impedance. This
e.m.f. will cause an electric stress in the region.
The opening of the switch has therefore produced high thermal
stress and high electric stress in the medium between the displaced
contacts. This medium is likely to be oil or gas: it could be vacuum
in some modern equipment.
Both forms of stress cause ionisation of the medium which results
in large quantities of free charge carriers being made available. The
surge of current through this ionised region produces a conducting
plasma, or arc, across the switch contacts.
What has happened is that the high impedance that was initially
placed in the circuit has been replaced by a very low impedance
and we are almost back where we started.
Almost, but not quite, because the arc has in it the seeds of its
own destruction. It is in fact a variable resistance.
The art of switchgear design is to engineer the device in such a
way that the resistance of the arc increases from its original low
value to a value at which the current in the arc is reduced to
approximately zero at the instant of natural current zero in the
a.c. or transient waveform. At this point the arc is extinguished
and current ceases to flow across the break in the circuit.
The faster the current is reduced the greater is the induced e.m.f.
in the circuit inductance and the more likely it is that a high
electric stress will cause further ionisation of the medium. Hence
the need to control the change of resistance in the arc. It is also
helpful if, at the instant of current zero, the ionised medium
around the breaker contacts can be replaced by fresh material
which has none of the byproducts of ionisation in it and which is
therefore better able to withstand the transient electric stresses
caused by the rate of change of current.
3-38

SYSTEM FUNDAMENTALS

Section Three

Exercise
Give reasons why the above
explanation would not describe
fully the switching of a direct
current.

Direct current does not pass through current zero at regular


intervals in the way alternating currents do. The system current
therefore has to be reduced to zero by the action of the arc
resistance alone. If the rate of change of resistance, and therefore
current, is too high then excessive induced e.m.f.s are produced
and the arc restrikes and the current will be maintained by the
system voltages. If, however, the rate of change of resistance is too
low then the current level persists for some time causing excessive
heating of the arc by i2 R loss and the arc is maintained. Switching
direct currents is therefore fundamentally more difficult than
switching a.c.

3.2.2 Arc control


The resistance of the arc can be controlled by increasing the
length of the arc and decreasing its cross-sectional area.
The simplest way of increasing arc length is to increase the
separation between the switch contacts, but although this works
well with very low voltages, such as a 230 V a.c. mains supply, it
is a wholly impracticable solution at high voltages because the
separations needed would be enormous and the time taken to
achieve these separations would be unacceptable.
In medium and high voltage switchgear therefore, other methods
are adopted.
When an arc occurs in oil a large volume of gas is rapidly
produced. The high pressures that are created can be used to move
the gas, and surrounding oil, through baffles, across grids and into
regions of cool, clean oil. It can also be used to force cool clean oil
across the path of the arc. Thus the arc length can be increased
and rapid cooling helps to reduce the radius of the arc column. An
advantage of this form of circuit breaker is that when the arc is
extinguished a volume of cool, clean oil is left between the
contacts.

TRANSFORMERS AND SWITCHGEAR

3-39

Module 1

The air-blast circuit breaker uses gas under high pressure to cool
and stretch the arc, but the arc is created in air, which is the main
insulant, and the gas used to blow across the arc is air maintained
under pressure and released as the contacts separate. Sulphur
hexafluoride SF6 is now used extensively in gas circuit breakers
in place of air. It is an electronegative gas, which means that it has
an affinity for electrons. It is therefore less easy to ionise than air
and offers much better insulating characteristics. The gas is not
released to atmosphere, but contained within the switching
equipment where it is cooled and repressurised for continual use.
Vacuum circuit breakers offer an ideal solution to the problem of
current interruption because in the perfect vacuum there is no gas
to ionise and an arc cannot be maintained. The reality is different
because establishing and maintaining a sufficiently high vacuum is
difficult and costly.
To begin with, all materials within the vacuum chamber have to
be degassed to remove any gas molecules that might have been
absorbed by the materials. Then we have to accept that as two
current-carrying contacts separate in a vacuum an arc is produced
and maintained in ionised metal vapour that is boiled off the
surface of the contacts. There is however an important difference
between this vapour and the ionised gas of an arc in gas or oil. It
is that at normal current zero the vapour rapidly condenses
leaving no significant ionisation between the contacts. As the
voltage across the contacts rises there is no conducting path to
allow a restrike.

3.2.3 The single frequency switching transient


When the arc is extinguished it is important that the voltage
across the switch contacts does not rise more rapidly than the
ability of the switch to withstand the voltage. The voltage across
the contacts will often be a transient and it is defined as the
Restriking Voltage.
The rate of rise of restriking voltage (RRRV) then determines
whether the arc remains extinguished or restrikes. We can
examine the simple case of a single-frequency transient by
representing the system by the equivalent circuit of Figure 3.31.

3-40

SYSTEM FUNDAMENTALS

Section 3

Figure 3.31
This is a worst-case condition in which all resistance is ignored
and the supply voltage is a sinusoid:

Under fault conditions, with the fault close to the switch, the
fault current if is given by:

if =

1
2V
sin t
v. dt =
L
L
= 2 I f sin t

where only the steady-state part of the integral is considered and


V and If are r.m.s. values.
The current will go to zero at t
begins.

= 0 at which point the RRRV

A standard way of calculating the effect of opening the switch at


current zero is to imagine a current if to be injected into the
circuit across the circuit breaker contacts. The total current is then
zero for all time.
The voltage across the breaker, Vb, is then given by:

where the subscript (s) tells us that the variables are written as
Laplace transforms, and

The transient is likely to occur for only a small time compared to


the period of the main supply voltage so we can approximate

to

TRANSFORMERS AND SWITCHGEAR

3-41

Module 1

p
2If ot
over this short time.
The Laplace form of

p
2If ot
is

p o
2If 2
s
so that:

Vbs

p
2If oL

1
s1 s2 LC

or:

Vbs



p
1
sLC

2If oL
s 1 s2 LC

The inverse Laplace form of this can be read from standard tables
and gives:

Vbt



p
t
2If oL 1  cos p
LC

The voltage transient thus starts at zero when t 0, i.e. at current


zero and oscillates at a frequency fT given by:

1
fT
2p

r
1
LC

The first peak has a value of

p
2 2If oL
and occurs after a time

1
2fT
so that a good approximation for the RRRV is:

3-42

SYSTEM FUNDAMENTALS

Section 3

Any resistance in the circuit will damp this oscillation and in


practice the transient will normally decay after several cycles
leaving only the power frequency voltage across the contacts of
the switch.

3.2.4 The double frequency switching transient

Figure 3.32

The equivalent circuit for this condition is shown in Figure 3.32


in which impedances on the line side of the switch are
represented by lumped reactances L2 and C2.

as before and the fault current (ignoring currents in C1 and C2) is


given by:

if =

1
v. dt
( L1 + L2 )

2V
sin t
( L1 + L2 )

= 2 I f sin t
We again assume that the current goes through zero at t = 0, at
which time the arc is extinguished, and we determine the effect
of this by imagining a current if being injected across the switch
terminals. Provided t is small, we can write

as

and the Laplace transform of this is, as before:

The impedance seen by the injected current is:

TRANSFORMERS AND SWITCHGEAR

3-43

Module 1

Zs

sL1
sL2

2
1 s L1 C1 1 s2 L2 C2

The voltage across the switch is then given by:

Vbs



p o
sL1
sL2
2If 2

s 1 s2 L1 C1 1 s2 L2 C2

In the time domain, this becomes:

Vbt

 



p
t
t
2If o L1 1  cos p L2 1  cos p
L1 C 1
L2 C2

We now have two transient frequency components of natural


frequency f1 and f2 where:

s
1
1
f1
2p L1 C1
and

1
f2
2p

s
1
L2 C 2

The frequency of oscillation on the line side, f2 , can be an order of


magnitude greater than f1 .
This can give rise to a very high RRRV.
The presence of system resistance will act to damp these
oscillations and they may well decay to zero within a millisecond,
but this is quite sufficient to cause ionisation of the medium
between the switch contacts, and the main system is there to
supply energy to the arc should it restrike.

3.2.5 Resistance switching


Resistors can be used to limit the severity of the switching
transient.
If we revert to the simple case of a fault close to the circuit breaker
then the equivalent circuit of Figure 3.33 can be used and a
resistor R is shown here in place across the switch contact.

3-44

SYSTEM FUNDAMENTALS

Section Three

Figure 3.33
The analysis proceeds as in Sections 3.2.3 and 3.2.4, but the
impedance seen by the injected current is now:

s
Zs


s
1
2
C s RC LC

;Vbs

p
2If o
1
h
i

C s s a2 b2

where

1
2RC

b2

1
1
 2 2
LC 4R C

so that

Vbs

p
2I f o

"

1
s 2a


Ca2 b2 s s a2 b2

Transforming back into the time domain gives:

Vbt



p
at
at a
sinbt
2If oL 1  e cosbt  e
b

The transient is critically damped if

r
1 L
R
2 C
i:e: if b 0;
then:

Vbt

p
2If oL1  eat

Remember that we have assumed t to be so small that


If sin ot If ot .
TRANSFORMERS AND SWITCHGEAR

3-45

Module 1

Over this interval Vcosot will effectively be V and the transient


voltage will settle down to this value.
Figure 3.34 shows the transient waveforms predicted for the
conditions R 100 O, and R 500 O, with the undamped
oscillation added.
The critically damped response is shown in Figure 3.35 together
with the first cycle of the undamped transient.

Figure 3.34

Figure 3.35

3.2.6 Current chopping


A gas-blast breaker can force a current to zero at an instant other
than a natural current zero. The force of the blast literally blows
3-46

SYSTEM FUNDAMENTALS

Section Three

the arc out. This can create problems because a sudden collapse of
current in an inductive circuit will induce a large voltage. If this
voltage is sufficiently large then it will restrike the arc. This cycle
of extinction and re-ignition could happen several times before
the arc is finally extinguished.
Referring back to the equivalent circuit of Figure 3.31 we see that
the fault current is defined by

if

p
26I f sinot

and we solved for the voltage across the contacts by assuming that
the time of interest was small compared to that of the period of
the main supply, so that

p
if % 26I f 6ot
Let us assume that the arc is blown out at some time to after the
start of the fault where to does not correspond to a natural current
zero. The current at this instant, io , can be calculated from the
equation for if .
If we repeat the analysis of Section 3.2.3, allowing for the sudden
extinction of io , then we can consider injecting io into the
circuit, in addition to the fault current if . In this case io appears
as a step function so that the Laplace form of io is io =s. It
divides into two components as illustrated in Figure 3.36.

Figure 3.36
We then have:

io vcs

vcs sC
s
sL

So that:

TRANSFORMERS AND SWITCHGEAR

3-47

Module 1

io
vcs


1
s sL sC
io
vcs


1
C LC s2
Converting this back into a time equation, using standard Laplace
tables, we have:

vct

r
L

6io 6sinon t
C

where:

1
on p
LC
This transient voltage adds to the transient created by the initial
opening of the circuit breaker contacts.

? Self-assessment question 3.1


The emf wave in a transformer fed from a sinusoidal supply could
be distorted by restricting the flow of third-harmonic currents.
Describe ways in which you think the third harmonic could be
restricted.

? Self-assessment question 3.2


The B-H loop for a magnetic circuit shows that when H is zero B is
finite. When an a.c. circuit is switched open the break will tend to
occur at a current zero on the a.c. waveform rather than at a
voltage zero.
A transformer on no load would only have magnetising current
flowing into it and if it were then switched off, the circuit would
open-circuit when I o 0, i.e. when H O, leaving a residual
flux density in the core.
What could be the effect of this on the magnetising current if the
transformer were switched back on line while this residual flux
density remained in its core?

3-48

SYSTEM FUNDAMENTALS

Section 3

Self-assessment question 3.3

A three-phase, three-winding transformer is defined as follows:


Primary winding: 132 kV
Secondary winding: 33 kV
Tertiary winding: 6.6 kV

Z12 = 0.09 p.u. to a 30 MVA base, resistance neglected


Z13 = 0.15 p.u. to a 30 MVA base, resistance neglected
Z23 = 0.08 p.u. to a 30 MVA base, resistance neglected.
There is a balanced load of 2 000 A at 0.8 lagging power factor
on the tertiary winding and a star-connected inductive reactance
of j50 per phase is connected to the secondary winding.
Determine the voltage required at the primary winding terminals
to maintain 6.6 kV at the tertiary terminals.

Self-assessment question 3.4

A three-winding, three-phase transformer is connected in a Yyd


arrangement. The rating of the three windings and their
equivalent impedances are given below:

Y connection:

11 kV, 12 MVA

Z1 = 0.06 p.u. to a 12 MVA, base

y connection:

66 kV, 12 MVA

Z2 = 0.01 p.u. to a 12 MVA, base

d connection:

3.3 kV, 4 MVA

Z3 = 0.10 p.u. to a 12 MVA, base

The transformer is connected in parallel with a 15 MVA,


11/66 kV, Yy transformer which has a reactance of 0.05 p.u. on
its own base rating. A balanced three-phase short circuit occurs
across the terminals of the delta-connected winding. Calculate
the short-circuit current fed from the 11 kV busbar. Assume that
the 66 kV busbar is not connected to a supply.

Self-assessment question 3.5

Two similar three-phase transformers are operated in parallel to


supply a star-connected load of 0.130 per phase. Each
TRANSFORMERS AND SWITCHGEAR

3-49

Module 1

transformer is rated at 1 MVA, 11 000/420 V and has a series


impedance of (0:02 j0:10) p.u.
The combination is fed from an 11 kV busbar, but one
transformer has a 5% tapping on the low voltage side, i.e. its
secondary open-circuit voltage is 1:056420 V.
Calculate the load taken by each transformer from the main
supply.

? Self-assessment question 3.6


A meter is connected to the secondary of a current transformer
(CT) and is assumed to be resistive and to present a burden of 3 VA
when carrying a current of 5 A.
The CT has a turns ratio of 20 : 1 and should ideally produce a
meter current of 5 A when the primary current is 100 A. The
magnetising reactance, referred to the primary is 44:56104 O.
Determine the magnitude and phase errors that occur on full load.
Neglect winding resistances and iron losses.

? Self-assessment question 3.7


A circuit breaker can be represented by the equivalent circuit of
Figure 3.31 in which L 3 mH; C 1mF and the fault current I f
has an r.m.s. value of 50 kA.
Determine:
(a)

the natural frequency of oscillation of the restriking


transient; and

(b)

the RRRV

Sketch the waveform of the recovery voltage assuming that


resistance effects cause the transient to decay within a few cycles.

? Self-assessment question 3.8


A three-phase fault occurs on the line side of a circuit breaker,
some distance from the breaker, and the section of short-circuited
3-50

SYSTEM FUNDAMENTALS

Section Three

line can be represented by a series inductance of 1:6 mH and a


shunt capacitance of 0:02 mF. On the supply side, the breaker is
fed from a 132 kV, 50 Hz source with a source impedance
equivalent to a series inductance of 8:0 mH and a shunt
capacitance of 0:08 mF.
Assuming that the breaker interrupts the fault at current zero,
determine:
(a)

the peak value of the transient voltage on each side of the


break; and

(b)

the frequency of each transient.

? Self-assessment question 3.9


A circuit breaker with a damping resistor across its contacts can be
represented by the equivalent circuit of Figure 3.33. The fault
current if is current chopped when it has a finite value io .
Show that the voltage appearing across the contacts as a result of
current chopping is given by:

vc

io
6eat 6sinbt
Cb

where:

1
2RC
s


1
1
b

LC 4R2 C2
a

TRANSFORMERS AND SWITCHGEAR

3-51

Module 1

3-52

SYSTEM FUNDAMENTALS

Section 4
GENERATION

4.0 Objectives
On successful completion of this section you should be able to:

carry out calculations, or use load charts, to determine the


performance parameters of generators operating in the steady
state

use similar analysis techniques to evaluate conditions of


power transfer between synchronous systems

develop the swing equation for a generator and solve it for


simple conditions

use the equal area criterion to investigate the stability of a


generator connected to an infinite busbar under transient
conditions

use the equal area criterion to determine the critical


clearance angle for a machine under fault conditions.

4.1 Introduction
This section deals with the characteristics of the major power
generating elements within a power system and describes their
performance in some of the different modes of operation of the
system.
What are these different modes?
The most obvious is the steady-state mode, except that in a real
system a true steady-state mode is rarely encountered. The power
and the voltage magnitude will be controlled by feedback
mechanisms in a generator. One controller is the governor, the
other is the voltage regulator and each of these will be exercising
small, but essential, changes continuously. This mode of operation
is known as Dynamic Operation. Despite this, we will spend
GENERATION

4-1

Module 1

some time examining the steady-state operating condition because


until this is understood, the operation of the machine in the
dynamic mode is difficult to follow.
Large, sudden changes are not welcomed by system operators and
these usually occur as a consequence of activities beyond their
control, such as lightning strikes on overhead lines. These are
known as transient effects and are usually subdivided into two
sections. Firstly, quite slow electromechanical transients where all
the associated transient voltage and current waveforms are
assumed to be sinusoids. Secondly, the fast electromagnetic
transient effects over a few cycles after the disturbance which
result in heavily distorted waveforms. Each of these modes can
best be dealt with using different approaches to modelling the
generator.
Synchronous a.c. generators consist mechanically of two elements,
the stationary member known as the stator and the rotating
member known as the rotor. These two elements are physically
separated by an air-gap. Both carry windings made up of copper
coils, one is a direct-current field winding, the other is a polyphase
alternating-current armature winding. A rotating-field machine
carries the field winding on its rotor and this form is common
with large generators where the armature rating is in hundreds of
megawatts.

Exercise
Consider what advantage this
form of construction offers by
examining how electrical
connections are made to the
rotor.

Rotor connections to an a.c. machine are made through brushes


that make sliding contact with continuous rings of conducting
metal (slip rings). The lower the power carried through this slipring/brush combination the better. It should be obvious that the
power taken by the field circuit is much less than in the armature
circuit.
If the machine operates at synchronous speed in the steady state
mode, how then do loading changes occur?

4-2

SYSTEM FUNDAMENTALS

Section Four

In the case of a generator with the field on the rotor, an increase


in output power is obtained by increasing the input power to the
prime mover. The machine will then accelerate and run at a speed
above synchronous speed until the rotating magnetic field
produced by the rotor moves ahead of the rotating magnetic field
produced by the stator by an amount necessary to meet the new
power balance. The machine will then slow down again to its
synchronous speed but with a different angle between the two
components. This will be explained fully, later in the section. All
synchronous machines must move away from their synchronous
speed in order to respond to a change in load demand. This speed
change will not exceed 4% under normal conditions.

4.2 Steady-state operation


We will make the following simplifying assumptions:

all parts of the magnetic circuit have constant permeability


so that the field and armature fluxes can be treated
separately as proportional to their respective currents and
their effects superimposed

the field flux is sinusoidally distributed in the air gap

the armature flux is sinusoidally distributed and rotates at


synchronous speed with constant magnitude harmonics are
neglected.

So we begin with a three-phase generator rotating at an angular


speed os the synchronous speed and generating three-phase
alternating current at the system frequency f.
On no-load, the field current produces a field flux Ff that
generates the open-circuit terminal emf Ef . Ef lags the flux Ff by
90 and this simple relationship can be represented by the phasor
diagram of Figure 4.1.

GENERATION

4-3

Module 1

Figure 4.1
We now allow a lagging current I L to flow into a load. As it flows
through the armature windings this current produces its own
magnetic flux Fa . This flux Fa will be in phase with I L . The
resultant flux, Fr , in the air gap of the machine will be the sum of
Ff and Fa and the summation is illustrated in Figure 4.2. The flux
Fa will also induce an emf Ea which will lag the flux by 90 . This
will have to be added to the emf Ef to produce the effective emf
Et generated by the machine. The effect of I L can be represented
more easily by considering Ea as a voltage drop across an
imaginary reactance Xa so that jEa j jI L 6Xa j and
E f E t I L Xa .

Figure 4.2
We now have two more effects to consider. The first is leakage
flux. Despite our simplifying assumptions, superimposing one flux
on another is not straightforward and some flux is forced into
leakage paths. By this we mean paths that do not pass through the
main windings. The flux-coupling with the windings is not 100%
and we find that the emf Et is reduced. The reduction is
proportional to the load current and we can represent it as a
voltage drop I L Xl where Xl is defined as a leakage reactance. The
final effect is winding resistance Rs which causes a further drop
I L Rs in our terminal voltage leaving us with the output voltage on
load defined as V in Figure 4.3. We write this as
Et V IRs jXl .

4-4

SYSTEM FUNDAMENTALS

Section Four

Figure 4.3
A single-phase equivalent circuit can be drawn with the reactances
Xa and Xl representing the effects of armature reaction and
leakage flux. This is shown in Figure 4.4.

Figure 4.4
Figure 4.5 shows a further reduction of the circuit with Xa and Xl
combined to produce the synchronous reactance XS and with RS
removed on the assumption that losses will be relatively small.
The corresponding phasor diagram is also shown. The angles
marked in the Figure are the power factor angle f and the load
angle d. A typical value for XS would be 2:8 for a modern
machine.

Figure 4.5
Now let us return to the no-load condition with the emf E and the
voltage V set to be equal and held at these values. E can be held
by excitation controls and V can be maintained by connecting the
generator to a large system with a system voltage V . The system is
assumed to be so big that the behaviour of the one generator is
not likely to effect either the system voltage or the system
frequency. Such a system is known as an infinite busbar.

GENERATION

4-5

Module 1

Exercise
Figure 4.6 represents a generator
connected to an in.nite busbar
operating with the excitation
emf E equal to the terminal
voltage V on no load.
Deduce what would happen to
the various parameters in the
phasor diagram if the input
power were increased and V and
E were maintained constant.

Figure 4.6

An increase in input power must cause an increase in output


power so we must create a load current. A load current will cause
a voltage drop ILXS and the voltage and emf phasors must move
apart so that:

E = V + ILjXS
E must follow a circular locus of radius E because it is held
constant in magnitude. So we have the phasor diagram of
Figure 4.7.

4-6

SYSTEM FUNDAMENTALS

Section Four

Figure 4.7
The current I L has to have a component in phase with V and the
only possible solution is the one shown in which d has increased
and the current has a leading power factor.
Let us go back to the no load condition and this time let E be
greater than V . Now increase the input power and we move from
Figure 4.8a to Figure 4.8b.

Figure 4.8
Notice that in this case we can actually have a current at unity
power factor, but I L XS is looking a bit large and we would not
really want a voltage drop of the same magnitude as the terminal
voltage. The situation is complex and an easy way to represent it is
to produce a load chart.

GENERATION

4-7

Module 1

4.3 The load chart for a synchronous generator


feeding an infinite busbar
This is a fairly easy chart to develop and we will be working in
single-phase equivalent values throughout.
We begin with a general phasor diagram for a machine with an
excitation voltage E supplying a lagging current I to a supply of
voltage V through a synchronous reactance XS ; Figure 4.9 refers.

Figure 4.9
The current has an in-phase component I P and a quadrature
component I Q . If we multiply all phasors in Figure 4.9 by V=XS
we obtain Figure 4.10 in which IXS has become VI , the complex
power, and I P XS and I Q XS become the real power VI P and the
reactive power VI Q respectively. We now take the point O in
Figure 4.10 as a new origin and rotate the diagram by 90 to
obtain Figure 4.11.

4-8

SYSTEM FUNDAMENTALS

Section Four

Figure 4.10

Figure 4.11

To construct the chart we locate the point A at a distance V 2/XS


away from the origin O and draw a circle of radius AC = VE/XS
to define the locus of the operating point C for constant
excitation E. We will have a family of circles of different radii
corresponding to different values of E.
Exactly where the operating point lies will depend on the
magnitude of the input power. This will de.ne the vertical
intercept OD which equals VI. So, once we have de.ned the
excitation voltage E, which we do by setting the field current,
and the power input to the machine, which we do by setting the
governor, we have fixed the operating point. The other variables
such as power factor angle and load angle are then fixed.
Looking at Figure 4.11 we can deduce some interesting facts.
Firstly, if we keep the excitation constant and keep increasing the
input power we eventually reach a point where the model fails.
There is a point M, vertically above the point A, which is the

GENERATION

4-9

Module 1

point of maximum electrical power output. For E and V , as


defined, we cannot get more electrical power than this out of the
machine. If we put more in, then something has to give.
What happens is this. The input power in excess of M accelerates
the machine beyond synchronous speed and if it is not stopped it
will lose synchronism with the infinite busbar. Imagine watching
eight rowers in a boat, all working in perfect synchronism, until
one catches a crab! Synchronism is lost, chaos ensues and it takes
a while to sort the mess out. Now think what would happen if a
500 MW generator went out of synchronism and you can
understand why people are very reluctant to let a generator
operate near the point of maximum power.
We can place a limit on our chart to define a region in which
operation is not allowed. This is most easily defined as a straight
line that limits the load angle to some value. For convenience, let
us make it 70 and show the practical stability limit as the line
AT in Figure 4.12.

Figure 4.12

We can also show a line representing the maximum input power


that the drive can supply and we can define a maximum value of
E, limited by the maximum rating of the field winding. Finally
there has to be a maximum value to the VA that the generator can
supply and this is limited by the heating of the stator circuit. With
all these limits in place, we have the load chart of Figure 4.12.
It might be worth trying the first of the self-assessment questions
now, but remember that so far we have been working in singlephase values.

4-10

SYSTEM FUNDAMENTALS

Section Four

4.4 Power transfer between synchronous systems


Load charts help us to visualise what is happening as the various
variables in the system change, but they are of no use for making
fast calculations for an operational system. We have to look at
analytical models that are suitable for computation.

Figure 4.13

For the simple equivalent circuit of Figure 4.13 we have:

E = V + jXsI
and if we take V as the reference phasor we obtain:

I=

E V0
= Icos jIsin
Xs 90D

Equating real parts:

Therefore, the electrical power generated is:

This power relationship is illustrated in Figure 4.14

GENERATION

4-11

Module 1

Figure 4.14

We see immediately from Figure 4.14 that there is a maximum


value of electrical power given by VE/XS. Increase the
mechanical input power above this and we lose synchronism,
exactly as we predicted with the load chart.
Now let us generalise and instead of thinking about a
synchronous generator imagine two a.c. busbars linked by a series
reactance. Each busbar is 'infinite' i.e. it has zero internal
impedance and in.nite inertia so that the voltage and frequency
are .xed. We illustrate the system in Figure 4.15.

Figure 4.15

Which way does the power .ow in Figure 4.15?

Exercise
Calculate the powers in the two
single-phase sources of Figure
4.15 when:

V 1 = 2000 V
V 2 = 20411.3 V
XS = j40

4-12

SYSTEM FUNDAMENTALS

Section Four

Using our standard conventions:

V 1 V 2 IXS
V 1  V 2 200  200 j40

XS
4090
j40
1:00 A
I
4090

Therefore:

P1 20061:0 200 W
P2 20061:0 200 W
We have power flow from right to left in the diagram.
Some people would say that this is obvious simply because V 2 is
greater than V 1 . This is not a sufficient answer. If you doubt this,
then repeat the exercise with

V 2 204  11:3 V
You should now find that the current has reversed and that the
power flow is from left to right. This is interesting because it tells
us that the flow of power between two a.c. busbars is controlled
electrically both by the magnitudes and by the relative phases of
the voltages at each end of the link.
Phase angles are very important to us. Remember however that
there is a mechanical system behind all this. We can only move
power across an interconnection if the power is available on one
side and can be absorbed on the other. Remember also that there
is energy contained in the spinning masses of all the generators on
the system. They can speed up, or slow down, and so absorb or
release energy. This will be seen as a change in the frequency of
the generated voltage and explains why a power provider will run
the system at a frequency slighter higher than the nominal value
just before a major load increase is expected. And as the load
increases, so the frequency drops, because the system takes energy
from the spinning mass of the generators to meet the initial
demand for more power.

4.5 Transient operation


The previous section leads us nicely to consider what does happen
when the generator, or the system, is operating transiently rather
than in its steady state. The first thing to appreciate is that the

GENERATION

4-13

Module 1

reactances of the generator change. As the electrical conditions in


the machine change, so changes are imposed on the magnetic
field distributions. These changes occur relatively slowly and
therefore, for a period following a disturbance the steady-state
values of the reactances no longer apply. The effect is to reduce
the series reactance of the machine from the synchronous
reactance value to a lower value known as the transient reactance
1
and represented by the symbol Xd. A typical value will be about
0.36 p.u. The previous analyses still hold, but we must write Xd1
in place of XS for the period of change. A full explanation of why
this happens can be found in standard texts on a.c. machines.
Now let us look at what is happening on the mechanical side of
the alternator while conditions are changing. We already know
that to change conditions in the machine the load angle has to
change. Now this is not an imaginary angle. It is the effective
angular displacement that exists between the mmf wave set up by
the rotor and the mmf wave created by the currents in the stator.
Both rotate at the same angular frequency, but one is slightly
behind the other. When conditions change, this angle must
change and one of the fields has to accelerate, or decelerate, a
little with respect to the other so that a new value of can be
established.
Given an in.nite busbar, we can assume that the frequency of the
supply is fixed, so the rotor of the machine has to make the
adjustment. If more power has been pumped into the machine
then the rotor will accelerate for a short time so that can
increase. This leads to an increase in the electrical power generated

Pe = VE sin /XS
The excess power that has been added is, for a short time,
available to accelerate the rotor. We shall call this power the
accelerating power Pa which will provide an accelerating torque
Ta acting on the rotor which has a polar moment of inertia J.
Under dynamic conditions therefore, the input power
the mechanical system must provide:

Pm from

the electrical power output Pe

the power to accelerate the inertia of the machine Pa

the power to overcome the losses, Pl.

The accelerating torque Ta can be obtained from Newtons laws of


motion as Ta = J where J is the polar moment of inertia of the
rotor, is the rotor acceleration and, for some angular rotor
position , we have:

4-14

SYSTEM FUNDAMENTALS

Section Four

in general we can write:

so that:

then, since in general, P

= torque,

and with the assumption that the speed change is small,


so we have:

s,

This is the 'Swing Equation'.


Power engineers often prefer to perform dynamic calculations
using the so-called H constant, or inertia constant, such that:

H=

stored kinetic energy at rated speed


rated voltamperes

Js2
2S

where S is the rated voltamperes. H then has the dimensions of


time and with M = s J,

M=

2 HS
joule-second per radian
s

2 HS d 2
2 watt
with Pa =
dt
s
We now need a corresponding expression for the electrical output
and we will only consider the simple case of constant excitation
at this stage. Then:

Note that we have taken the three-phase value of the electrical


power because the mechanical system is accelerated by the

GENERATION

4-15

Module 1

accelerating power from each phase; a single-phase solution


would not be appropriate here.
The power balance equation then becomes:

or:

This is a mathematically non-linear differential equation because


of the presence of the sine term. Several standard software
packages exist which can be used to give the load angle as a
time function when M, Pm, Pmax and Pl are known.
Alternatively, a further approximation can be made using a
linearised solution for a small change, such that:

Pe = (dPe /d ) = (Pmax cos O ) = GO


from an initial condition = o.
Then assuming simple damping proportional to speed above
or below synchronous (due to induction machine action in the
generators damping windings), the loss term can be written
as:

P1 = Kd

d
( )
dt

and the power balance equation becomes:

d2
d
+ Kd
+ Go = Pm
2
dt
dt

This is a linear second order differential equation and the


standard (homogeneous) form of it is written as:

The damping ratio is , and the undamped natural frequency is


n, so that in this particular case:

2n =

Kd
G
2
and n = o
M
M

The motion of damped, elastic systems such as this is dealt with in

4-16

SYSTEM FUNDAMENTALS

Section Four

section 11.4.2 of Modern Engineering Mathematics by Glynn James.


For our present purpose it is suf.cient to note that the rotor will
move to a new position and will oscillate about the new position
with an angular frequency n. The oscillation will be damped by
the damping effect of losses in the machine.
Typically, for a modern machine rated at 1 000 MVA at a power
factor of 0.8, running at 3 000 r/min, the H constant will be
about three seconds and:

Then with operation at an initial load angle of 60 and a


maximum power of (say) 1 000 MW:

Go = Pmax coso = 500 MW/rad


and:

n =

Go
= 5.1 rad/s
M

giving an undamped periodic time of about


electromechanical rotor oscillations.

1.2 s for the

We can simplify the problem a little for the case of a solid threephase fault ocurring at the generator terminals. In this case the
electrical power output must go to zero if the fault has zero
resistance and if we ignore all losses then the swing equation
becomes:

or:

Integration gives us:

However, the initial value of

d/dt

GENERATION

4-17

Module 1

was zero so:

dd Pm
6t

M
dt
Integrating again gives:

Pm
6t 2 d0
2M

and we usually know d0 from the initial conditions.


Note that the angles here are measured in radians!

Exercise
A three-phase, 2-pole, 50 Hz
turbogenerator is rated at
1 100 MVA at a power factor
of 0:8 lagging. The polar
moment of inertia of the rotating
parts is 61 000 kg m2.
Calculate the value of the H
constant. If the peak on the P=d
curve occurs at a power of
1 020 MW, find the full load
operating load angle and the
undamped natural frequency of
the electromechanical rotor
oscillations.

Angular velocity:

2p
63 000 100p rad=s
60
Jo2 61 000100p2
H s
2:74 s
2S
261 1006106
os

Given:

Pe Pmax sind;
and
4-18

SYSTEM FUNDAMENTALS

with

Pmax 1 020 MW

Section Four

Therefore,

n =

= 59.6 The undamped frequency is:

Go
M

but:

and:

Go = Pmax cos0 = 516 106 W/rad


so:

n = 5.18 rad/s.

4.5 The equal area criterion for stability


To determine whether or not a power system is stable after a
disturbance it is necessary to solve the swing equation for the
system. In most cases, this clearly needs to be done by computer,
but there is a simple graphical method that we can use when we
have a single machine operating on an infinite busbar. The
method gives us further insight into what is actually happening
during the disturbance and also allows us to work out how fast
our protection equipment needs to be if it is to protect the system
from damaging transients.
In section 4.4 we showed that the electrical power produced by a
single generator is:

where Pmax is the maximum power that can be transferred for


given values of V and E. The relationship is shown in Figure 4.16.

GENERATION

4-19

Module 1

Figure 4.16
Let us assume that the generator is initially operating with a load
angle do and that the mechanical power input equals the electrical
power output and has a value PA . Losses are ignored.
Let us now increase the input power to a new value PB . The
machine cannot move to its new equilibrium position, d d1 ,
instantaneously, so initially we have excess power, PB  PA ,
which is available to accelerate the machine. The load angle
begins to increase.
As d d1 the accelerating power falls to zero because the
electrical power has now become PB , but the machine has inertia
and it will therefore run on past the position d d1 . Once d is
greater than d1 the electrical power will exceed PB and we will
now have a power deficit Pe  PB . This represents an energy
deficit and the only source that this can be taken from is the
kinetic energy of the machine. So the machine must slow down. It
slows until its excess kinetic energy has been lost. At this point it
must be running at synchronous speed again, at some angle
d d2 . However, we still have a decelerating force created because
Pe now equals PC which is greater than the mechanical input
power PB . The machine continues to slow down. With no
damping the machine will simply swing back to d do and the
whole cycle will begin again. We have set up an oscillation about
the new steady state position d d1 . In reality, losses in the
machine will cause it to settle down quickly to its new steady state
position.
Figure 4.16 contains two areas, ADB and BEC. It can be shown that
these areas represent energy. The area ADB represents kinetic
energy gained by the machine as it accelerates to increase its load
angle from do to d1 . The area BEC represents the kinetic energy
given back as the machine slows back down to synchronous speed.
It should be obvious that for stability these two areas should be
equal.
So, how could a machine become unstable?
4-20

SYSTEM FUNDAMENTALS

Section Four

Figure 4.17
Consider Figure 4.17. Here we have increased the input power by a
considerable amount and the area ADB represents the increase in
kinetic energy of the machine as its rotor accelerates past the
position d d1 . The area BEC is clearly less than ADB so when
d d2 the machine is still running beyond synchronous speed.
But, we now have a new problem because as soon as the load
angle increases beyond d2 the electrical power drops below the
new input power PB . and the machine starts accelerating again! It
cannot return to d d1 . If we allow this situation to continue
then the rotor of the machine will advance so far that the
electrical power output will go negative, i.e. the generator will
suck power in from the infinite busbar and try to act as a motor.
This is not a good thing to do with a 500 MW generating set. So
we have to control things. We need fast-acting controllers on the
power input and in the field excitation circuit and we need fast
protective gear to isolate the generator from the busbar before the
situation gets completely out of control.

GENERATION

4-21

Module 1

Exercise
A synchronous generator
operating with constant
excitation is connected to an
infinite busbar through a fixed
transfer impedance and has a
power/load angle curve of the
form P 100 sind. Draw this
curve to scale and use the equal
area criterion to confirm that,
after a sudden change in output
power from 40 to 80 units, the
machine returns to a stable
operating mode.

Figure 4.18
The P=d curve is drawn to scale and the two operating points,
P1 40 units of power and P2 80 units of power, can be
identified. The construction of Figure 4.18 can be used to identify
that the area A2 is greater than the area A1. The machine is
therefore stable.
We can put the equal area criterion on a firmer footing with a
little bit of mathematics. Starting with the swing equation, and
assuming no damping, we have:

4-22

SYSTEM FUNDAMENTALS

Section Four

d2 d
M 2 Pm  Pe Pa
dt
where P m is the mechanical power input;

Pe is the electrical power output; and


Pa is the accelerating power.
Multiplying both sides of the equation by

2 dd
M dt
gives:

d2 d dd
Pa dd
2
2
M dt
dt dt

We now use the identity

d  2
dy
y 2y
dt
dt
to convert

d2 d dd
2 2
dt dt
into

d
dt

"

dd
dt

2 #

so that

d
dt

"

dd
dt

2 #
2

Pa dd
M dt

so

"
d

2 #
dd
Pa
2 dd
M
dt

Integration gives:

GENERATION

4-23

Module 1

"

dd
dt

dd

dt

2 #

Z
Pa dd

s
Z
2
Pa dd
M

This is the angular velocity of the rotor relative to the


synchronous field. For the machine to be stable, this relative
motion must become zero. So, over the period of acceleration and
deceleration illustrated in Figure 4.16 between do and d2 ; dd=dt
must go to zero.
Hence:

Zd2

Pa dd 0

do

This is the mathematical formulation of the equal area criterion.


Expanding the equation, we obtain:

Zd2

Pm  Pe dd 0

do

Zd1

Pm  Pe dd

do

Zd2

Pm  Pe dd 0

d1

The two integrals represent the areas ADB and BEC in Figure 4.16
respectively.
Note that when using this formula, area ADB will be positive,
whereas area BEC will be negative. We can re-express the above
equation to make both areas positive i.e.

Zd1

Pm  Pe dd

do

4-24

SYSTEM FUNDAMENTALS

Zd2
d1

Pe  Pm dd

Section Four

Exercise
A three-phase generator is
operating under conditions that
define its power angle curve as
Pe 100sind MW . It supplies
a power of 50 MW.
The mechanical input to the
generator is suddenly increased
to 75 MW. Determine whether
or not the machine remains
stable.

Let us take a look at the nature of the problem.


Figure 4.18 illustrates the limiting condition. The initial operating
point is defined by the 50 MW load and since Pe 100sind, we
calculate do as 30 .
We now step up the mechanical input power. The load angle
increases and the limiting condition for stability is the condition
illustrated by point C in Figure 4.18. The machine must have
stopped accelerating by the time it reaches C, and preferably
before then.
Using the equal area criterion we can say that if area BEC is equal
to or greater than area ABD then the machine will remain stable.
We can solve the problem graphically, but that is a bit hit and
miss, so we will use the equations that we have just derived for the
two areas.
Firstly area ABD:

ABD

Zd1

Pm  Pe dd

do

where P m 75 MW is the new input power and


d0 30 0:524 radians.

GENERATION

4-25

Module 1

d1 is defined by 100sind1 75, so d1 48:6 0:848 radians

ABD Pm d1  do 

Z48:6

100sind dd

30

ABD 750:848  0:524 100cos48:6  cos30


ABD 3:83
Secondly area BEC:

BEC

Zd2

Pm  Pe dd

d1

If we look at the symmetry of Figure 4.18 we see that


d1 p  d2 ,
or, alternatively, area BEC 26 area BEF.

BEC 26

Zp=2

Pm  Pe dd

d1

h p


i
p
BEC 26 75  d1 100 cos  cosd1
2
2
BEC  23:81
The machine will remain stable because jarea BECj>jarea ABDj

4.6 Fault conditions and the equal area criterion


We can apply the equal area criterion to the analysis of faulted
systems in which transfer reactances change and where switchgear
is operating to alter the synchronous connections. In the real
world we would use a transient analyser to solve the problem and
this would take into account the full complexity of the system and
include the effects of automatic voltage regulators (AVRs) and
other controlling sub-systems. However, we are trying to keep
things relatively simple so that we can grasp some of the
principles involved. The equal area criterion is a good tool for this
purpose.

4-26

SYSTEM FUNDAMENTALS

Section Four

Figure 4.19
In Figure 4.19 we begin with a machine operating in a steady
condition and feeding an infinite busbar through a network. The
effective reactance between the generator emf E and the voltage of
the infinite busbar, V , determines the Psind curve (see Section 4.4
and Figure 4.15).
The Psind curve for the system is shown as the pre-fault curve in
Figure 4.19. The initial value of the load angle is do .
We now imagine a fault occurring away from the generator which
results in an increased value of reactance between the source emf
E and the busbar voltage V . This means that we must
immediately transfer to the new Psind curve shown in Figure 4.19
as the fault curve.
The load angle cannot change instantaneously so the operating
point for the machine drops from point a to point b in the figure.
We now have accelerating power available because the mechanical
input power is greater than the electrical output power. As the
machine accelerates, the load angle d increases along the fault
curve. Now, after a time t 1 , the protection equipment operates to
open a switch which removes the fault, but leaves us with a
network connection which is different from the original one. At
t t 1 the load angle has advanced to d1 so that as the switch
opens the electrical output characteristic changes from fault to
post-fault and the operating point moves abruptly from point c
to point e.

GENERATION

4-27

Module 1

The machine continues to swing because of its inertia, even


though the electrical output at point e is now greater than the
mechanical input. It does however begin to slow down and, in this
example, it comes back to the synchronous speed at point f, at a
load angle d2 . The area abcd equals the area defg and the machine
remains stable.

Exercise
What would be the condition of
critical stability on Figure 4.19?

We would reach critical stability if the machine were to swing as


far as point h on the post-fault curve. Any slight movement
beyond h would create an accelerating torque which would cause
d to increase beyond d p=2.
We could create this condition easily by delaying the operation of
the switch so that d1 moves to the right. This would increase the
area abcd and the necessary increase in the area defg could well
push the point f as far as point h and beyond.
To make useful operational sense of this we need to link the
angles d1 and d2 to time because the operation of protective gear
and of switchgear is defined in time and not in terms of load
angle. This takes us back to the swing equation and the solution
for load angle d in terms of time. It is too complex an issue for
this work book, but you can pursue the problem in later units
should you so wish.
Example
In the network shown in Figure 4.20 the emf behind the generator
transient reactance is E 1:25 p:u. The generator is connected by
two lines in parallel to an infinite busbar held at 1:0 p.u. volts. A
fault occurs at point F and is switched out at a time t 1
corresponding to a load angle d1 . After the operation of the
switch, the generator is connected to the busbar via the single
healthy line.

4-28

SYSTEM FUNDAMENTALS

Section Four

Determine the critical value of the switching angle for stability to


be maintained if the machine is originally supplying 1.0 p.u.
power.

Figure 4.20
The pre-fault equivalent circuit reduces to that of Figure 4.21. The
Psind curve is given by:

1:25
sind 3:125sind
0:4

Figure 4.21
The fault condition is represented in Figure 4.22(a). The star
network ABC is converted to a delta network ABC in Figure 4.22(b)
to give the transfer reactance between E and V as X 1:1 p:u.

GENERATION

4-29

Module 1

Figure 4.22
The fault Psind curve is given by:

1:25
sind 1:14sind
1:1
The post-fault condition is defined by Figure 4.23 and the postfault Psind curve is defined as:

1:25
sind 2:5sind
0:5

Figure 4.23

4-30

SYSTEM FUNDAMENTALS

Section Four

Referring back to Figure 4.19, the condition for stability would be:

area abcd area defh 0


or:

Zd1

Pm  1:14sinddd

do

Zd2

Pm  2:5sinddd 0

d1

Pm d1  do 1:14cosd1  cosdo Pm d2  d1 2:5cosd2  cosd1 0


We know that Pm 1:0.

do is obtained from the initial conditions; 1:0 3:125sindo , so


that do 18:7 or 0:326 radians.
d2 corresponds to the point h in Figure 4.19, so d2 is obtained
from the final condition that: 1:0 2:5sind2
where p=2 < d2 < p,
so that d2 156:4 or 2:73 radians.
Rearranging the above equation and substituting the known values
gives:

cosd1 0:755
so that

d1 139

? Self-assessment question 4.1


A synchronous generator with a transient reactance of j0:4 p.u. is
connected to an infinite busbar of voltage V b 1:0 p:u. via a
transmission reactance of j0:8 p.u. An ideal AVR maintains the
machine terminal voltage V t at 1.0 p.u.
Sketch a phasor diagram to show the relationship between V b ; V t
and the generator excitation e.m.f E when maximum power is
being transferred between the terminals at V t and the busbar at
V b.
Hence or otherwise determine the value of the load angle between
the excitation e.m.f. E and the busbar voltage V b .

GENERATION

4-31

Module 1

self-assessment question 4.2

A turbo-alternator is rated at 100 MVA, 11 kV and 0.8 power


factor lagging. It has a synchronous reactance of 1.5 p.u. and the
field current to obtain rated voltage at no load is 200 A.
Draw an operating chart for the machine when operating at 11 kV
and use the chart to determine the field current required if the
machine is delivering 60 MW at 0.95 leading power factor.
Assume that the field is not saturated.

self-assessment question 4.3

Construct an operating chart for a 100 MW turbo-alternator with


a synchronous reactance of 1.67 p.u. assuming that the machine
is rated to produce full load power at 0.9 lagging power factor
and operates at rated voltage on an infinite busbar.
Draw in operating limits given that:

2.4 p.u.

the excitation limit is

the armature current is limited to rated value

the maximum output is 100 MW

the stability margin is

0.1 p.u. power.

Hence determine:

the minimum leading power factor at which the generator


may operate at 50 MW output and the load angle for this
condition

the maximum permissible power output at a load angle of


20 and the MVAr at this point

the p.u. excitation for a power output of 75 MW at a lagging


power factor of 0.8.

self-assessment question 4.4

A three-phase, two-pole, 50 Hz turbogenerator supplies 400 MW


at a load angle of 10. The polar moment of inertia of the
rotating parts is 61 000 kg m2.

4-32

SYSTEM FUNDAMENTALS

Section Four

Calculate the H constant and the undamped natural frequency of


the electromechanical oscillations, when the machine supplies
rated power of 880 MW at a power factor of 0.8 lagging. The
excitation is unchanged.
The machine is initially in the steady state in its original
operating condition (400 MW at 10). A three-phase short circuit
then occurs at the machine terminals.
Determine the load angle of the machine three cycles after the
short circuit occurs.

self-assessment question 4.5

A 50 Hz, 400 MVA synchronous generator has an inertia


constant of 2 MJ/MVA and supplies a 220 kV infinite busbar over
a circuit with an effective series reactance of 0.6 p.u. referred to
400 MVA and 220 kV. The circuit includes an appropriate
transformer.
The excitation of the machine is set to provide an emf of 1.2 p.u.
Determine the load angle of the rotor relative to its no-load
position when the machine supplies 300 MW to the busbars.
A sudden change in circuit configuration increases the series
reactance to 0.9 p.u.
Determine the angular position of the generator rotor 0.05 s after
the change.

self-assessment question 4.6

A three-phase synchronous generator is connected to an infinite


busbar through a synchronous reactance X. The operating
conditions are:

E = V = 1.0 p.u.
PI = 0.8 p.u.
X = 0.5 p.u.
A fault occurs which increases the value of the series reactance to
1.2 p.u., but the emf, E, and the busbar voltage, V, remain at
1.0 p.u. By the time the fault has cleared, the load angle of the
generator has increased from its initial value to a value of 100.
Determine whether or not the generator will remain stable.

GENERATION

4-33

Module 1

4-34

SYSTEM FUNDAMENTALS

Section 5
LOAD FLOW ANALYSIS

5.0 Objectives
When you have completed this section you should be able to:

understand why load flow is important to power utilities

understand the nonlinear nature of the load flow equations

understand the Gauss iterative method for solving non-linear


equations

derive simple equations for load flow and formulate them in


a fashion suitable for Gauss iterative analysis

perform simple load flow analysis by hand and appreciate


why computers perform these tasks far better than humans

understand the limitations of the Gauss iterative method.

5.1 Introduction
In a large integrated power system, it is difficult to assess the
voltages and currents associated with each individual transmission
line, although the positions of power generation and consumption
are known. The difficulty lies in the fact that the system equations
are nonlinear and cannot be solved directly. Instead an iterative
approach is used where an initial guess is made to the problem
which is subsequently changed until the solution fits the problem.
This activity is referred to as load flow, since the solution will
account for all flows of power, both active and reactive, between
the generator and the load.
Load flow analysis is important to power utilities for many reasons
but the main two uses are concerned with day to day operations,
where the redistribution of power flows due to a line being
removed for maintenance are important to the economic
operation of the power system, and power system planning, where
LOAD FLOW ANALYSIS

5-1

Module 1

future expansion must be considered in the light of existing


capabilities. It is also important in ensuring that individual line
and plant loading is kept within safe and secure limits.

Exercise
Can you think of other reasons
why power utilities use load flow
analysis?

Specific reasons for performing load flow studies include:

to ensure that power system plant is not run above


nameplate rating

to keep voltage levels of certain buses within close tolerances


to ensure correct reactive power requirements

to assess if contingency fault conditions may potentially lead


to widescale system outages.

5.2 The load flow problem


Figure 5.1 shows, possibly, the simplest power system
configuration. A generator of fixed terminal voltage VA is
connected to a load S, via a transmission line which, to keep
matters simple, has a series resistance of R. The problem is to find
the amount of real power, PG , generated by the generator. This
seems, at first sight, to be a simple problem, but, it will be seen,
this is not quite the case.

Figure 5.1
It is common in power systems to specify the loads in terms of
their power, i.e. so many watts, kilowatts or megawatts. Thus, the
5-2

SYSTEM FUNDAMENTALS

Section Five

load of Figure 5.1 is related to the voltage at busbar B, VB , and the


current through the system, I :

S VB I

Equation 5:1

However, the voltage at busbar B, VB , is related to the voltage at


busbar A, VA and the voltage drop along the transmission line, IR,
which in turn is related to the current through the system, I :

VA  VB IR

Equation 5.2

We may solve Equations 5.1 and 5.2 for VB . Eliminating I :

VA  VB
S

R
VB

Equation 5.3

which can be written as:

VB2  VB VA RS 0

Equation 5.4

And so, to find VB involves a nonlinear equation. Having found


VB we may then proceed to find the power dissipated in the
transmission line, and hence find the power produced by the
generator at A.

Figure 5.2
This seemingly simple exercise has turned out to involve solving
quadratic equations. This is the fundamental problem in
calculating load flows: nonlinear equations. Consider now the still
relatively simple network of Figure 5.2. Here there are four busbars
to consider. If we wished to find the distribution of power flows
along each line, we would need to solve four simultaneous
nonlinear equations. This may no longer be solved by inspection.
Instead, we have to use iterative techniques which are far more
easily performed by computer than by hand. The simplest method
of iteration is called the Gauss method.

LOAD FLOW ANALYSIS

5-3

Module 1

Exercise
Write a solution to the nonlinear Equation 5.4. Can this
solution be used to solve for the
power system of Figure 5.2?

Equation 5.4 is a simple quadratic equation that may be solved by


the well-known formula:

VB

VA +

q
VA2  4RS
2

However this formula is only applicable to equations with one


unknown and hence cannot be used for solving the equations
relating to the system of Figure 5.2. To solve a set of nonlinear
equations, we must use an iterative method.

5.3 The Gauss iterative method

5.3.1 Theory
The operation of the Gauss iteration can be appreciated readily by
applying it to the solution of the simple quadratic equation 5.4.
Equation 5.4 is rearranged as:

VB 

RS
VA
VB

Equation 5.5

However, to make the expression iterative, we will calculate a new


value for VB on the left hand side of equation 5.5 by guessing an
initial value for VB on the right hand side of equation 5.5. Thus,
the equation becomes:

VBk1 

5-4

SYSTEM FUNDAMENTALS

RS
VA
VBk

Equation 5.6

Section Five

where the superscript k refers to the order in which the values of


VB are calculated. Now, by assuming an initial value for VB and
repetitively evaluating Equation 5.6, the exact solution of VB to
the equation may be found.
Example
Calculate the power generated by the generator at busbar A in
Figure 5.1 given the following values:
Load

400 MW

Resistance of transmission line; R


Voltage of busbar

15:5 O

A 231 kV

Calculate the answer using the Gauss iterative method and check
it by using the quadratic solution formula.
To proceed we use Equation 5.6. Taking an initial value of VB as
231 kV (i.e. the same as VA ), we get the following results by
iterating:
Table 5.1
k

VB kV

204.16

200.63

200.10

200.02

200.00

200.00

and so we see that, to two decimal places, no improvement in the


answer occurs for more than six iterations. To check the answer,
we use the quadratic formula:

VB

q
VA + VA2  4RS
2

which gives the answers

VB 200 kV
or

VB 31 kV
The above example has illustrated another important point
LOAD FLOW ANALYSIS

5-5

Module 1

regarding iterative equations: the possibility of multi-valued


solutions. When assessing the answers from the quadratic formula,
we intuitively choose the higher value solution since, in a power
system, we expect busbar voltages to be of approximately the same
magnitude. Hence,

VB 200 kV
is the correct answer. However, if the two answers are closer
together, this selection process becomes more difficult. For the
example given above, the initial choice of VB has little bearing on
the result.
Having found VB , we can calculate the power loss in the
transmission as:

PL

VA  VB 2 31 kV2

62 MW
R
15:5 O

Equation 5.7

Thus, the generator at A must export:

PG Load losses 400 62 462 MW

Exercise
Try performing the iteration of
the previous example using
different starting values.

You should find that the result is always 200 kV, even for starting
values close, but not equal, to 31 kV.

5.3.2 Application of the Gauss iterative method to


multi-terminal power systems
To be of any use to a power systems engineer, the Gauss iterative
method must be applicable to more realistic power system
5-6

SYSTEM FUNDAMENTALS

Section Five

configurations. We shall take as our example the system of Figure


5.2; in practice, computer programs used by power utilities are
capable of handling systems with two thousand or more busbars.
In our simple system of Figure 5.1 we assumed that the
transmission line and load were purely resistive. This allowed us
conveniently to ignore any mention of the flow of reactive power.
However this quantity is very important in power system analysis
and must be considered further. This implies that loads must be
specified in terms of their real and reactive powers, and that the
reactance of transmission lines must also be represented.
The equations are formulated as follows. Taking busbar 3 of Figure
5.2 as an example, the total real, P, and reactive, Q , power at
busbar 3 is related to the voltage of the bus, V3 , and the net
current passing through the bus, I3 , as:

V3 I 3 P3 jQ3

Equation 5.8

where the * superscript denotes the complex conjugate. P is


taken to be positive if there is net generation at a bus, in which
case I3 is flowing from the generator into the transmission system
as shown in Figure 5.2. I3 may therefore be expressed as:

P3 jQ3 
I3
V3
or

P3  jQ3
I3
V3

Equation 5.9

However, I3 is composed of the components of current entering or


leaving busbar 3 via the lines to other busbars. Thus, using
admittances to represent the transmission lines, Equation 5.9 may
be re-expressed as:

P3  jQ3
Y13 V1 Y23 V2 Y33 V3 Y43 V4
V3
Equation 5.10

Note that admittances refer only to the transmission lines and not
the busbar loads or generators. Equation 5.10 may be rearranged
into a form suitable for iteration:



1 P3  jQ3
V3
 Y13 V1 Y23 V2 Y43 V4
V3
Y33
Equation 5.11

LOAD FLOW ANALYSIS

5-7

Module 1

Similar to Equation 5.6 for the simple system of Figure 5.1,


iterative use of Equation 5.11 allows us to find the value of V3 . In
general, for a power system having N buses, the voltage at the kth
bus is given by:

"
#
N
1 Pk  jQk X
Vk

Ykn Vn n 6 k
Ykk
Vk
n1

Equation 5.12

Exercise
Explain the significance of
admittances Ykk and Ykn in
Equation 5.12.

Ykn the value of this term is minus the admittance measured


between busbars k and n in the network.
Ykk sum of all admittances connected to busbar k:
Iterative evaluation of Equation 5.12 for all buses will eventually
lead us to the final solution. As described so far, the complex
power,

S P jQ;
is specified for all buses. However, no allowance has been made for
the power losses, real or reactive, that occur in the transmission
lines; these losses are, of course, load dependent. If we specified S
for all buses, then clearly no solution could be found unless the
generation power exactly equalled the load plus the losses. To
allow for this, we make one bus within the system independent of
S; this bus is referred to as the slack or swing bus and is usually
denoted as being node 0. Furthermore, the voltage is specified at
the slack bus and, hence, Equation 5.12 is not applied to the slack
bus.

5-8

SYSTEM FUNDAMENTALS

Section Five

Exercise
Why is a slack bus necessary to
iterative load flow analysis?

In load flow analysis, both generation and load are specified.


However, the system losses will depend on the exact solution
found by the load flow study. The slack bus is used to ensure that
generation exactly equals load plus losses.

Figure 5.3
Example
Find the voltages at busbars 1 and 2 of Figure 5.3 after one
iteration of the Gauss method.
Table 5.2
Busbar

Voltage

Generation (p.u.)

Load (p.u.)

0.9908

1 j2

0.3  j0.1

0.2 j0.1

0.7 j0.1

First of all we will construct the admittance matrix from the


admittance values given in Figure 5.3. Remember that diagonal
elements are formed from the sum of the admittances connected
to the busbar in question, and that the off diagonal elements are
negative with respect to the values given in Figure 5.3.

LOAD FLOW ANALYSIS

5-9

Module 1

3 2
3  j6
I0
4 I1 5 4 2 j4
I2
1 j2

2 j4
3  j6
1 j2

32 3
1 j2
V0
5
4
1 j2
V1 5
V2
2  j4
Equation 5.13

We will begin by assuming that V1 and V2 are both set to 10 .


Hence, applying Equation 5.12 to this problem for V1 and V2
gives us:



1 P1  jQ1
V1
 Y01 V0 Y21 V2
V1
Y11

Equation 5.14



1 P2  jQ2
V2
 Y02 V0 Y12 V1
V2
Y22

Equation 5.15

and

Substituting in actual values results in the following equations:



1
0:1 j0:2
V1
 2 j40:99 j0 1 j21 j0
3  j6
1 j0
Equation 5.16

and



1
0:7  j0:1
 1 j20:99 j0 1 j21 j0
V2
2  j4
1 j0
Equation 5.17

Note, when calculating the P  jQ at buses 1 and 2, we take


generation to be positive, load to be negative and form the
complex conjugate as demanded by Equations 5.14 and 5.15.
Working out Equations 5.16 and 5.17 gives:

V1 0:973 j0:027

and

V2 1:085 j0:130:
Note, that from Equation 5.13:

I0 3  j6V0 2 j4V1 1 j2V2


Equation 5.18

and so:

5-10

SYSTEM FUNDAMENTALS

Section 5

I0 = (3 j6)(0.99 + j0) + (2 + j4)(0.973 + j0.027)


+ (1 + j2)(1.085 + j0.130) = (0.429 j0.062)
Equation 5.19
*

Hence, VA generated at slack bus 0 is V0 I 0 plus the connected


load, i.e.:

S = (0.99 + j0)(0.429 + j0.062) + (1 + j2)


= 0.575 + j2.06
Equation 5.20
Note that the solution as shown above is referred to as the Gauss
method. However, another slightly different method can be used
in which, after evaluating V1 from Equation 5.14, this new value
is then used to calculate V2 from Equation 5.15. This approach is
called the Gauss-Seidel method and is a slight improvement on
the Gauss method.

5.3.3 Limitations of the Gauss iterative method


A drawback to using the Gauss method is the, sometimes,
excessive number of iterations that are needed before a solution is
found. Part of the problem is that this method takes no account
of sign or magnitude of the error existing between iterations. This
problem can be met part way by applying an acceleration factor
which multiplies the difference between iterations by a constant
in the range 1 to 2. However, the best solution is to use a
technique which takes account of the error and uses it to modify
the next iterative cycle. The Newton-Raphson technique is an
example of this and takes far fewer iterative cycles to reach the
solution than the Gauss method.

Exercise
Why is the Newton-Raphson
method superior to the Gauss
method?

In the Gauss method, calculation of the values at a new iterative


step depends solely on previous values and the iterative formula.
However, in the Newton-Raphson method, the calculation of a
new iterative step also makes use of an estimate of the error
LOAD FLOW ANALYSIS

5-11

Module 1

existing from the previous step. This enables the NewtonRaphson


method to converge upon the solution using fewer iterative steps
than the Gauss method.

5.4 The NewtonRaphson method

5.4.1 Theory
The NewtonRaphson method differs from the GaussSeidel
method in that new iterative updates of the required busbar
voltages are based upon the rate of change of the solution.
Initially we will simplify the theory in order that the basic
principle of this method of iteration is fully understood. We will
begin by considering the d.c. system of Figure 5.1. We learned that
the relationship describing voltages, power and line resistance is
given by Equation 5.4 which is repeated here in a slightly different
form:

VB2  VA VB RS F1 VB 0

Equation 5.21

Equation 5.21 shows Equation 5.4 as a function, F1 , of VB . Of


course, when we have found the correct value of VB to fit our
parameters in VA , R and S, then F1 VB will be zero. Using, as
before, the index k to refer to successive iterations of VB , we can
write:

F1 VBk DVBk 0

Equation 5.22

where VBk is the error between the correct value of VB and its
estimate on the kth iteration. In order to update the value of
DVBk1 , we will try to estimate the value of DVBk by using the
Taylor series expansion of F1 :

F1 VB

F1 VBk

DVBk

dF1
dVB

Equation 5.23

Equation 5.23 shows the first two terms in the Taylor series
k
expansion where dF1 =dVB is the derivative of F1 with respect
to VB evaluated at VBk . Since Equation 5.23 equates to zero, we
may evaluate DVBk as:

F1 VBk
DVBk

dF1 k
dVB
5-12

SYSTEM FUNDAMENTALS

Equation 5.24

Section Five

Note that Equation 5.24 only approximates to DVBk since only the
first two terms in the Taylor series expansion of Equation 5.23
were considered.
Exercise
What is the expression for
dF1 / dVB ?

We can easily evaluate the gradient of F1 by differentiating


Equation 5.21 with respect to VB :

dF1
2VB  VA
dVB

Equation 5.25

Exercise
Using the same example of
Section 5.3.1, calculate the first
iterative value of VBk1 starting
with VBk VA initially.

For VBk VA 231 kV, S 400 MW and line resistance


15:5 O; F1 evaluates to:

F1 VBk VBk 2  VA VBk RS


2316103 2  2316103 2 15:564006106
6:26109
Equation 5.26

and

dF1
dVB

2VBk  VA 262316103  2316103


2316103
Equation 5.27

LOAD FLOW ANALYSIS

5-13

Module 1

Hence,

F1 VBk 6:26109


DVBk

2316103
dF1 k
dVB
26:84 kV

Equation 5.28

Therefore the updated value of VB is:

VBk1 VBk DVBk 231  26:84 204:16 kV


Equation 5.29

Further iterative cycles reveal:

Table 5.3
k

VB kV

204.16

200.10

200.00

These results prove the worth of the NewtonRaphson method.


Comparing the above table with the table for the Gauss method
shown in Section 5.3.1, it is clear that the correct solution is
reached with fewer iterative cycles using the NewtonRaphson
method (3 iterations compared to 5). Note, however, that there is
a price to be paid for quicker convergence: a more complicated
algorithm.

5.4.2 Application to a.c. systems


When applied to an a.c. power sytem, the equivalent of F1 is a
function of two direct variables. In the GaussSeidel case, we saw
that the governing equation, Equation 5.12, is solved for a real
and imaginary component of Vk , or alternatively, a magnitude
and an angle of Vk but in either case there are two variables to be
found.
We will proceed by deriving the equations necessary to solve
Figure 5.3, the 3-bus power system. We will take all complex
values in the equations to be in polar coordinate form:

5-14

SYSTEM FUNDAMENTALS

Section Five

Vk jVk jdk

Vn jVn jdn

Ykn jYkn jykn


Equation 5.30

From Equation 5.12 we may write:

Pk  jQk

N
X

jVk Vn Ykn jykn dn  dk

Equation 5.31

n1

Equation 5.31 may be separated into its real and imaginary


components:

Pk

N
X

jVk Vn Ykn jcosykv dn  dk

Equation 5:32

n1

Qk 

N
X

jVk Vn Ykn jsinykv dn  dk

Equation 5:33

n1

Similar to the GaussSeidel case, the slack bus (taken to be bus 0


in Figure 5.3) is not considered in the system equations. Beginning
with initial values for the busbar voltages, values for Pk and Qk are
evaluated from Equations 5.32 and 5.33. The calculated value for
real power, Pkcalc , corresponds to the term F1 VBk on the right
hand side of Equation 5.23 (note, of course, that we will have two
equations of the form of Equation 5.23 in the a.c. case, the extra
Equation being in Qk ). Similarly, the specified value of real power,
Pkspec , corresponds to the left hand side of Equation 5.23. Thus,
an equation in P of the form:

Pkspec Pkcalc

qPk
DVk
qVk

Equation 5.34

may be derived. Notice that the derivative of Equation 5.23


becomes a partial derivative in Equation 5.34 since Pk is a
function of all busbar voltages. An equation similar to Equation
5.34 may be derived for Qk . Denoting DPk as the difference
between the specified and calculated power:

DPk Pkspec  Pkcalc

Equation 5.35

and similarly:

DQk Qkspec  Qkcalc

Equation 5.36

We can write the final expression relating Pk , Qk and Vk for


busbars 1 and 2 of Figure 5.3:

LOAD FLOW ANALYSIS

5-15

Module 1

qP1
6 qd1
2
3 6
6 qP2
DP1
6
6 DP2 7 6
6
7 6 qd1
4 DQ1 5 6
6 qQ1
6
DQ2
6 qd1
6
4 qQ2
qd1

qP1
qd2
qP2
qd2
qQ1
qd2
qQ2
qd2

qP1
qjV1 j
qP2
qjV1 j
qQ1
qjV1 j
qQ2
qjV1 j

3
qP1
qjV2 j 7
72
3
qP2 7
7 Dd1
qjV2 j 7
Dd2 7
76
7Equation 5.37
76
qQ1 74 DV1 5
7
DV2
qjV2 j 7
7
qQ2 5
qjV2 j

The 464 matrix of partial derivatives is referred to as the Jacobian


matrix. By considering the derivatives of Equations 5.32 and 5.33
with respect to the relevant variables (in this case d1 , d2 , V1 and
V2 ), expressions for each element of the Jacobian may be derived.
Numerical values of the Jacobian can then be evaluated using the
initial values of the busbar voltages.
Thus far, initial values of busbar voltages have enabled us to
calculate DP, DQ and the Jacobian matrix. In order to solve for
Dd and DV , the Jacobian needs to be inverted and then
multiplied by the column vector in DP and DQ . When Dd and
DV are found, they are added to the initial values for Vd to form
a more accurate estimate. The iterative process may then be
repeated until the correct busbar voltages are found.
It will be apparent that load flow calculations using the Newton
Raphson method cannot easily be performed by hand calculation
and are better entrusted to a computer.
When written as a computer program, it is necessary to apply a
test which stops the iterative process when DV is less than some
preset value indicative of the accuracy required. Unlike the Gauss
Seidel method, the NewtonRaphson method is sensitive to initial
values that are far removed from the correct solution. However,
since in power systems busbar voltages are usually close to 1 p.u.
in magnitude, it is rare for the NewtonRaphson method to fail to
converge when 1 p.u. is taken as an initial guess.
Extra busbars may be incorporated into Equation 5.37 to take
account of power systems with greater than 3 busbars. Each extra
busbar will add an extra 2 elements to the column vectors and add
2 extra columns and 2 extra rows to the Jacobian. Thus, for an N
busbar system, the main burden of the computation will be in
evaluating the inverse of a 2N  16 2 N  1 Jacobian matrix.

? Self-assessment question 5.1


Perform the second iteration of the Gauss method example given
in subsection 5.3.2. and hence calculate the power generated at
the slack bus.
5-16

SYSTEM FUNDAMENTALS

Section 6
UNBALANCED SYSTEMS

6.0 Objectives
When you have finished this section you should be able to:

use symmetrical components to represent unbalanced threephase systems

carry out calculations to determine conditions in unbalanced


systems.

6.1 Introduction
In this section we develop the use of symmetrical components for
analysing three-phase unbalanced systems. The particular
unbalanced conditions created by faults on systems will be dealt
with in Section 7. Here we just want to concentrate on the
principles of the method and apply it to simple examples of
unbalanced loading.

6.2 Symmetrical components


We are already used to the concept of a balanced three-phase
system of phasors. If you wish you may refer back to Section 2.4.1
and Figure 2.11.
In Figure 6.1(a) we show such a system made up of voltages V a1 ,
V b1 and V c1 . The voltages are mutually displaced by 120 8 and
the sequence is defined as positive because a stationary observer
on the vertical axis will see the phasors pass by in the sequence a,
b, c. In Figure 6.1(b) we show a balanced system of negative
sequence voltages V a2 , V b2 and V c2 . In this case the phasors
rotate in the sequence a, c, b; hence the term negative sequence.

UNBALANCED SYSTEMS

6-1

Module 1

Figure 6.1
Finally in Figure 6.1(c) we show three phasors which are in phase
with each other. These are called zero sequence components.
For the purpose of our analysis these phasors are going to be
theoretical concepts, but we could produce them if we so wished.
The positive sequence would come from a standard three-phase
generator, the negative sequence we could obtain simply by
swapping over two of the output connections from the standard
generator and the three zero sequence components could be
obtained from three, separate, single-phase generators mounted on
one shaft with the windings so arranged that the three output
voltages were in phase at all times.
Now let us add together the a phase, the b phase and the c
phase components respectively of Figure 6.1. The result is shown
in Figure 6.2. We have created three new phasors, V a , V b and V c
and these are obviously unbalanced. Writing these as vector
equations, we have:

V a V a0 V a1 V a2
V b V b0 V b1 V b2
V c V c0 V c1 V c2

Figure 6.2
What we now need to recognise is that we can represent any
arrangement of three phasors by a unique set of three sequence
components. So that any unbalanced condition can be represented
6-2

SYSTEM FUNDAMENTALS

Section Six

in this way. This makes analysis easier because we are then dealing
with two sets of balanced three-phase systems the positive and
negative sequence components and one set of co-phasal
components. We solve the problem for each set of components
and simply add the results together.
Now comes the difficult part.
How do we know what the unique values of the sequence
components are?
We will return to this question shortly, but first we need the help
of the a operator.

6.3 The operator


We note firstly that the voltages V a1 , V b1 and V c1 are mutually
displaced by 120 8; as are V a2 , V b2 and V c2 , although the
sequence is different. To make life easier for us we need a way of
representing this 120 8 phase shift.
Let us remind ourselves of the j operator. We know that if we
multiply a phasor by j then we keep the magnitude of the phasor
constant, but rotate the phasor by 90 8:

i:e:

j 1:090

and, if V V0 
then; jV 1:090  6V0

V90

We now need to introduce a new operator a where


a 1:0120  . Multiplying a phasor by a rotates it anticlockwise by 120 8.
Since 0 a0 1:0120  we can represent it by:
0 0
a 0:5 j0:866, or 0 a0 1=2 jH3=2 (i.e.
1:0cos120  j1:0sin120  ). We can also show that:

UNBALANCED SYSTEMS

6-3

Module 1

These relationships may be useful to us later.


We can now write:

This follows because Vb1 = a2 Va1, Vc1


Vc2 = a2Va2; and Vao = Vbo = Vco.

= aVa1; Vb2 = aVa2,

This has simplified things a lot because we need only concern


ourselves with the values of the 'a' phase. We can simplify a bit
further by dropping the subscript 'a' in the symmetrical
components and writing:
Equation 6:1

or
Equation 6:2

(Before proceeding further, just make sure that you really know what
V0, V1 and V2 stand for.)
So far so good, but we still need to know the values of V0, V1
and V2 and we obtain these by using some mathematics.
Equation 6.2 is written in matrix form and can be inverted using
standard methods to obtain:

6-4

SYSTEM FUNDAMENTALS

Section Six

3
2
V0
1
7 16
6
4 V1 5 4 1
3
V2
1
2

1
a
a

3 2
3
Va
1
7 6
7
a 2 5 64 V b 5
a

Equation 6:3

Vc

or

1
V a V b V c
3

1
V 1 V a aV b a2 V c
3

1
V 2 V a a2 V b aV c
3
V0

Exercise
The following phase voltages are
measured at the output of a
three-phase, star-connected
generator. The star point is
earthed.

V a 1:00 
V b 1:2100
V c 0:7145

p:u:


p:u:

p:u:

Determine the zero, positive and


negative sequence voltages that
can represent this condition,
taking the a phase as reference.

This involves straightforward substitution.

UNBALANCED SYSTEMS

6-5

Module 1

1
1:0 1:2100 0:7145
3
1
1:0 1:20:174 j0:985 0:70:814 j0:574
3
1
0:218 j1:584 0:073 j0:528 0:53482:13 p:u:
3

1
V a aV b a2 V c
3
1
1:0 1:0120  61:2100  1:0240  60:7145 
3
1
1:0 1:2220  0:725 
3
1
1:0 1:20:766  j0:643 0:70:906 j0:423
3

V0
V0
V0
V1
V1
V1
V1
V1

1
0:715  j0:476 0:238  j0:159 0:287  33:75
3

1
V 2 V a a2 V b aV c
3
1
V 2 1:0 1:2340  0:7265 
3
V 2 0:689  j0:369 0:78  28:17  p:u:

p:u:

We now have three sets of component voltages. The next question


is, how do we use them?
It would seem to make sense to calculate three sets of component
currents, but what do the component circuits look like?

6.4 Sequence components of impedance


The previous exercise gave us three sets of sequence voltages that
we could use to replace the unbalanced three-phase set that was
measured. We can apply these in turn to the circuit that the
original generator was feeding.
Let us represent this by the equivalent circuit of Figure 6.3

6-6

SYSTEM FUNDAMENTALS

Section Six

Figure 6.3
If we let the generator provide a set of positive sequence voltages
V a1 , V b1 and V c1 then the solution for this set is straightforward.
The phase voltage V a1 creates a phase current

V a1 =Zc ZL
The neutral impedance Zn has no effect because the voltages are
symmetrical and the circuit is symmetrical so that the three phase
currents sum to zero at the star point and there is no positive
phase sequence current in the neutral wire.
We can therefore define a positive phase sequence impedance
Z1 for the circuit as:

Z1 ZC ZL
Now let the generator provide a set of negative sequence voltages
V a2 , V c2 and V b2
The phase voltage V a2 creates a phase current

I 2 V a2 =Zc ZL
and the negative phase sequence impedance Z2 is defined as:

Z2 Zc ZL
We might reasonably expect Z1 to equal Z2 in symmetrical, static
circuits. This is true provided that Z1 and Z2 are not sensitive to
phase sequence. In most cases this is true. A transmission line, a
cable or a transformer will not behave differently if we simply
swap over two of the supply connections. However, for electrical
machines things will be different because the reversal of the phase
sequence will reverse the direction of rotation of the magnetic
field in the airgap. We can therefore expect that for electrical
machines Z1 will not equal Z2 .

UNBALANCED SYSTEMS

6-7

Module 1

The zero sequence circuit presents us with problems and we need


to take care. The impedance of a three-phase line, or cable, for
example has a component due to mutual impedance caused by the
fluxes interlinking the conductors. If we excite the line with three
equal in-phase voltages (or zero sequence components) then the
coupling fields change and the mutual impedance will change. We
expect therefore to record a value of zero sequence impedance
which will be different from the value of the positive and negative
sequence impedances. Let us denote this as Z0 .
We have a further problem however, and that is the nature of the
circuit connection.

Exercise
If the zero sequence current in
line a of Figure 6.3 is I 0 , what
is the zero sequence current in
the neutral wire?

If the zero sequence current in line a is I 0 then the zero


sequence currents in lines b and c have the same value. This
follows from our definition of the sequence components in which
we define the zero sequence set as three identical co-phasal
components. Now, three co-phasal currents arriving at a star point
must add together and because they are co-phasal they will not
sum to zero. The current in the neutral wire is therefore the sum
of the three zero sequence components, i.e.

I N 3I 0
The voltage drop across the neutral impedance is then:

3I 0 ZN
We can now draw three equivalent single-phase circuits for the
three component systems. These are shown in Figure 6.4. Note
that we have to put 36ZN in the zero sequence equivalent circuit
to allow for the voltage drop 3I 0 6ZN that is produced across the
neutral impedance. Take note of this because it will be important
when we move on to fault calculations.

6-8

SYSTEM FUNDAMENTALS

Section Six

Figure 6.4

One last problem: suppose that the neutral of the load is isolated.
What happens to I0?
If we break the neutral circuit then I0 cannot flow! The balanced
positive and negative sequence components are not affected
because they sum to zero at the star point, but co-phasal currents
cannot sum to zero at a point and this means that if the neutral is
open circuit then no zero sequence current can flow.
This is an important conclusion for us because it means that we
need to know whether or not star points are earthed in any
system that is likely to be unbalanced. And what if there is no star
point? Suppose the supply system, or the load, were connected in
delta?
We need to examine this more closely, but first another exercise.

UNBALANCED SYSTEMS

6-9

Module 1

Exercise
A balanced, star-connected,
resistive load of 0:4 p.u. per
phase is supplied via a feeder
from the unbalanced supply
defined in the exercise in Section
6.3. The feeder has positive,
negative and zero sequence
reactances of j0:14, j0:14 and
j0:28 p.u. respectively. There is
a resistance in the neutral
connection of 0:25 p.u.
Calculate the current in the a
phase of the supply.

We have:

V 0 0:53482:13 p:u:
V 1 0:287  33:75 p:u:
V 2 0:780  28:17 p:u:
The total positive sequence impedance per phase is:

Z1 0:4 j0:14 0:42419:29

p:u:

The total negative sequence impedance equals the total positive


sequence impedance so that:

Z2 0:4 j0:14 0:42419:29

p:u:

The effective single-phase equivalent zero sequence impedance is:

Z0 0:4 j0:28 360:25 1:15 j0:28


1:18413:68

p:u:

Therefore:

0:534
68:45  p:u:
1:184
0:287
 53:04  p:u:
I1
0:424
0:780
 47:46  p:u:
I2
0:424
I0

6-10

SYSTEM FUNDAMENTALS

Section Six

We can now reconstitute the a phase current by combining I 0 ,


I 1 and I 2 using the form of Equation 6.1:

Ia I0 I1 I2
I a 0:4510:0:367 j0:930 0:6770:601  j0:799 1:8400:676  j0:737
I a 1:817  j1:478
I a 2:342  39:13 p:u:

6.5 Sequence impedances of transformer


connections
Positive and negative sequence impedances present us with no
problems. We treat each system of voltages or currents as we
would any balanced three-phase system and use an equivalent
single-line, single-phase equivalent circuit in our solution.
Zero sequence components have to be considered with care
because, as we have seen, the neutral connection is critically
important.
Firstly, then, a couple of simple rules:

If a neutral connection is made, then it offers a path to zero


sequence currents. Whether or not the path is complete will
depend on the connections in the remainder of the neutral
path;

If no neutral connection is made, then no zero sequence


current can flow.

This leads us to our first two zero sequence equivalent circuits for
two common transformer connections. These are shown in Figures
6.5(a) and 6.5(b).

UNBALANCED SYSTEMS

6-11

Module 1

Figure 6.5
In Figure 6.5(a), co-phasal currents can flow to earth on the
primary side and balancing currents can flow in the secondary
windings. These currents can also flow to earth via the earthed
neutral on the secondary side. The equivalent circuit shows a
straight through connection via the transformers zero sequence
impedance. Whether or not zero sequence currents will actually
flow depends on the neutral connections of the equipment that is
connected on either side.
In Figure 6.5(b), zero sequence currents cannot flow in the
secondary winding because the neutral connection is open circuit.
This means that there can be no zero sequence currents in the
primary winding. This is because there has to be a balance of
Ampere-turns in a transformer, i.e. N 1 I 1 N2 I2 . The equivalent
circuit shows the zero sequence impedance in place, but the circuit
is broken on the secondary side to prevent the flow of zero
sequence currents across the transformer.

6-12

SYSTEM FUNDAMENTALS

Section Six

Figure 6.6
Our second common transformer arrangement is the star/delta
connection shown in Figure 6.6. In Figure 6.6(a), the star point of
the primary is earthed. Zero sequence currents therefore have a
path to ground on the primary side. Balancing currents can flow
in the secondary windings as illustrated, but I a0 , I b0 and I c0 are
all equal and co-phasal. They therefore represent one continuous
zero sequence current, which is circulating around the delta
winding. There can be no zero sequence component of current in
the secondary lines. The delta winding effectively traps the zero
sequence currents. The equivalent circuit demonstrates this. A
path remains for the zero sequence currents to flow in the primary
network.
In Figure 6.6(b) there can be no zero sequence currents in the
primary and therefore none in the delta.
Finally we have the delta/delta winding. From what we have said
above, zero sequence currents may circulate around the delta but
cannot flow into the lines. The connection and its equivalent
circuit are shown in Figure 6.7.

Figure 6.7

UNBALANCED SYSTEMS

6-13

Module 1

The one helpful note is that the value of the zero sequence
impedance in a transformer is the same as the value of the positive
and negative sequence impedances.
Remember that any added impedance in the neutral circuit must
be represented in the zero sequence equivalent circuit by an
impedance of three times its value. This means that if ever we
have an earthing impedance connected between the star point and
earth then we must add three times its value to the zero sequence
impedance of the transformer. The connections of the equivalent
circuits shown above are not changed.

? Self-assessment question 6.1


A balanced three-phase supply feeds a balanced, unearthed, starconnected load. One line of the supply goes open circuit and the
current measured in the healthy a line is 20 A.
Using the current in the a line as reference, determine the
symmetrical components of the line currents if the open line is
line c. Assume that the phase sequence is a, b, c.

? Self-assessment question 6.2


A star-connected, three-phase supply provides balanced line
voltages of 200 V. It supplies a star-connected load which consists
of a 30 O resistor in the a phase, a capacitor of reactance  j40 O
in the b phase and an impedance of 30 j40 O in the c
phase. The neutral points of both supply and load are isolated. The
supply sequence is a, b, c.
Calculate the currents in the three lines and the voltage between
the two neutral points.

? Self-assessment question 6.3


The three phase voltages of an unbalanced three-phase supply are:

V A 220 j0 V
V B 0  j200 V
V C 100 j200 V

6-14

SYSTEM FUNDAMENTALS

Section Six

The supply feeds a balanced, star-connected load of 30 j40 O


per phase. The two neutral points are isolated.
Determine the symmetrical component currents that make up the
current in the a phase and hence the three line currents.

? Self-assessment question 6.4


A balanced delta-connected load is supplied from a star-connected
three-phase supply. The impedance of each branch of the load is
66037  O. Under fault conditions, the phase voltages of the starconnected supply become:

V a 110  kV
V b 10  110
V c 12115

kV

kV

Determine:
(a)

the sequence components of the supply voltages

(b)

the sequence components of the line currents

(c)

the actual line currents in the load

? Self-assessment question 6.5


Draw out the positive, negative and zero sequence networks for
the circuit shown in Figure 6.8. Assume that the generators are
operating at 1:0 p.u. excitation and provide balanced three-phase
voltages. All sequence reactances are given in Table 6.1 in per unit
values to a common voltage and VA base.

UNBALANCED SYSTEMS

6-15

Module 1

Figure 6.8
Hence obtain the value of the transfer reactance that exists for
each sequence component.

6-16

Positive sequence
reactance

Negative sequence
reactance

Zero sequence
reactance

Transformers

0.2

0.2

0.2

Generators

0.1

0.1

0.05

Tie line

0.2

0.2

0.3

SYSTEM FUNDAMENTALS

Section Seven

Section 7
FAULT CALCULATIONS

7.0 Objectives
When you have completed this section you should be able to:

distinguish between symmetric and asymmetric faults

be able to use symmetrical components to solve for


asymmetric faults

derive the sequence networks for the common asymmetric


faults: single phase to earth; phase to phase; double phase
to earth

be able to allow for different transformer connections when


solving for asymmetric faults.

7.1 Introduction
We shall be using the concepts gained in Section 6 to develop
techniques for determining the fault currents and fault voltages in
various commonly found fault conditions. We will assume that
the three-phase supply is always balanced and to distinguish
supply voltages from voltages elsewhere in the circuit we will
define the three phase supply voltages as Ea Eb and Ec. Since
these voltages are balanced, their symmetrical components are:

a positive sequence component of value E1

NO negative sequence component

NO zero sequence component.

The voltages on the network will be defined as Va, Vb and Vc. In


particular, at a point of fault these will be Vaf Vbf and Vcf. Since
these voltages are likely to be unbalanced, we will need to define
them in terms of the symmetrical component voltages V0, V1 and
V2. These are defined by Equation 6.3.

FAULT CALCULATIONS

7-1

Module 1

The sequence voltages are the driving voltages behind the


sequence currents. So, we can imagine the sequence networks as
Thevenin equivalent networks. For the balanced three-phase
system of Figure 7.1 the sequence networks are shown in Figure
7.2.

Figure 7.1

Figure 7.2
Our first exercise will be to short circuit the terminals a, b and c in
Figure 7.1. This is a simple, balanced fault, but it will help us to
see how the sequence networks are used.

7-2

SYSTEM FUNDAMENTALS

Section Seven

7.2 Fault analysis


Three-phase fault
We will now consider the case of the three-phase fault.

Figure 7.3
Three-phase fault

Since we have a balanced fault on a balanced system, we can say


that, at the fault point,

Vaf = Vbf = Vcf = 0


From Equation 6.3, we can deduce that V0 = V1 = V2 = 0.
Clearly, the only connections to the sequence networks which will
fit the constraint on the sequence voltages given above are:

Figure 7.4
Sequence network connections
for three-phase fault

From which we learn that under symmetrical, or balanced, threephase fault conditions we need only consider the pps network.
This is another justi.cation for describing three-phase power
networks with one line diagrams but only under balanced
conditions.

FAULT CALCULATIONS

7-3

Module 1

Exercise
A synchronous generator with
terminal voltage 11:8 kV,
rating 20 MVA and pps
transient reactance X1 0
0:08 p:u: is subjected to a
three-phase short-circuit.
Calculate the fault current.

First we calculate the value of the transient reactance in ohms.

X0 p:u:base voltage2
X O
base VA
0:08611 8002

0:557 O
206106
0

;Taking E1 to be

11:8 kV
p
3
(see Figure 7.5) the fault current I1 is

I1

E1 E1
11:86103
0 p
12:2 kA
Z1 X1
360:557

Note that an alternative method for this calculation involves the


three-phase short-circuit level, or just fault level FL which is given
by:

Base VA p
3VL Isc
Zp:u:
where Isc short-circuit current
and VL line voltage
FL

206106
VA
0:08
FL
206106
1
p
12:2 kA
And thus; I1 p
0:08
3611 800
3V
Hence

7-4

SYSTEM FUNDAMENTALS

FL

Section Seven

Single phase to earth fault


Now consider a short between the 'a' phase conductor and the
earth:
We can readily see that Vaf

=0

Figure 7.5

'a' phase to earth fault


From Equation 6.2, this leads us to:

V0 + V1 + V2 = 0

Equation 7.1

Furthermore:

Icf = Ibf = 0
and so, from Equation 6.2 formulated for currents rather than
voltages, we get:

I 0 = I1 = I 2 =

I af
3

Equation 7.2

Exercise
Can you arrange an
interconnection of the sequence
networks of Figure 7.2 to agree
with Equations 7.1 and 7.2?

FAULT CALCULATIONS

7-5

Module 1

Figure 7.6
Sequence network connection for
'a' to earth fault

Note that we could have readily chosen either the 'b' or 'c' phase
for the earth fault. However, the choice of 'a' to earth leads to a
sequence component diagram, Figure 7.6, which does not involve
a the 120 shift operator. This will simplify hand calculations.
Phase to phase fault
Now consider a 'b' to 'c' clear of earth fault.

Figure 7.7
'b' to 'c' fault

Exercise
By considering the fault point
currents, can you describe the
function of the zps network for
this fault?

Clearly Iaf = 0 and Ibf + Icf = 0. From the current version of


Equation 6.2 I0 = 0; since there is no zps source in the network,
this also implies that V0 = 0 and hence the zps network has no

7-6

SYSTEM FUNDAMENTALS

Section Seven

bearing on this fault. In fact, the zps network is only ever present
under earth-fault conditions.
To deduce the nature of the pps and nps networks connections
we proceed by realising that:

Vbf = Vcf
Thus from Equation 6.3:

(Vaf + aVbf + a2Vbf )

3
V1 = V2
1
2
V2 = (Vaf + a Vbf + aVbf )

3
V1 =

Furthermore

I1 = ( aI bf a2 I bf )

3
I1 = I 2
1 2
I 2 = ( a I bf aI bf )

3
Hence the connections of the networks are:

Figure 7.8
Sequence network connections
for 'b' to 'c' fault

Phase to phase to earth fault


In this case the fault will appear as:

FAULT CALCULATIONS

7-7

Module 1

Figure 7.9
'b' to 'c' earth fault

Exercise
Can you deduce the sequence
network connections to describe the
fault of Figure 7.9?

Since Iaf
that:

= 0, we see from Equation 7.2 formulated for currents,

I0 + I1 + I2 = 0
We also have it that Vbf

= Vcf = 0

Hence, from Equation 6.3:

V0 = V1 = V2 =

Vaf
3

Thus giving the connection of the networks as:

7-8

SYSTEM FUNDAMENTALS

Section Seven

Figure 7.10
Sequence network connections
for 'b' to 'c' to earth fault

Note that the choice of b and c phases for the last two faults
(b to c and b to c to earth) has resulted in sequence network
connection diagrams that do not involve a the 120 shift
operator.

7.3 More complicated networks


So far we have considered a power network comprising an ideal
three-phase voltage source and an overhead line. In practice, it is
normal to model the voltage source as being an ideal three-phase
supply behind a source (sub-transient) impedance. Also, we may
wish to consider the effect of resistance in the fault path
substantial fault resistance may occur in the case of earth faults.
As an example, consider the single line diagram of Figure 7.11
which shows an a phase to earth fault with fault resistance Rf.

Figure 7.11
Resistive single phase to earth
fault
Total line impedance between A and B = Z1.
In order to derive the sequence component networks,we will use
Figure 7.6 as a basis, but we must also consider separately Zs and
Z1 the proportion of the overhead line involved in the fault, as
well as the fault resistance.

FAULT CALCULATIONS

7-9

Module 1

Figure 7.12
Sequence network connections
for resistive earth fault

The correct sequence network connections for the fault of Figure


7.11, are shown in Figure 7.12. The source impedance Zs has its
own set of zero, negative and pps values. Notice how the busbar A
may be drawn into each sequence network. If we wished to find
the phase voltages existing at A,then we would have to calculate
Vao, Va1 and Va2 by solving the network of Figure 7.12,and then
transform these values into phase voltage form by using Equation
6.10.
In Figure 7.11, the only fault current flowing, Iaf, is from the a
phase conductor to earth via Rf

Ibf = Icf = 0
From Equation 6.3:

I0 =

I af
I af = 3I 0
3

Equation 7:3

Since all of Iaf flows into Rf we take account of this and the
relationship of Equation 7.3 by making Rf 3 times larger in the
sequence network diagram of Figure 7.12.

7-10

SYSTEM FUNDAMENTALS

Section Seven

Exercise
A 100 km, 400 kV
transmission line is subject to an
'a' phase to earth fault at 80
km. If the fault path resistance
is 10 , calculate the fault
current.

Figure 7.13

Data:

Referring to Figure 7.11 and assuming there is no load flow prior


to the fault:

Thus,

FAULT CALCULATIONS

7-11

Module 1

And hence the fault current

Ia = I0 + I1 + I2
= 3I0
= 1 092 j4 035 = 4 180 75 A

Exercise
Deduce the sequence network
connections of Figure 7.14 for a
resistive 'a' phase to earth fault.

Figure 7.14

Notice now that we have also included a two ended transmission


system. To proceed, it is helpful to consider just the pps network.
We will get:

Figure 7.15

If you are not sure about this, cover the right hand side of Figures
7.14 and 7.15. Do you notice the similarity between these and

7-12

SYSTEM FUNDAMENTALS

Section Seven

Figure 7.11 and the pps network of Figure 7.12? Thus, to deal with
2-ended systems, we need only consider the sequence networks to
be of the form of Figure 7.15, with the connections as shown.
To finish this problem, we need to concentrate on the zps
network, since the nps network will be similar to Figure 7.15 with
the sources shorted out. The transformer at B will cause us little
problem since, according to Section 6.5, it is simply a series nps
impedance connection.
However, owing to the presence of the delta winding of the
transformer at A, we will need to take account of the inhibited zps
current .ow into the generator at A. The entire sequence network
will, therefore, be:

Figure 7.16

Self-assessment questions 7.1

Figure 7.17
For the 400 kV system of Figure 7.17, calculate the following:

FAULT CALCULATIONS

7-13

Module 1

a) Fault = 'a' to earth, no fault resistance


Evaluate

Va; Vb; Vc; Ia; Ib; Ic at point 'V'.

b) Fault = 'b' to 'c', no fault resistance


Evaluate

Vb; Vc; Ib at point 'V'.

c) Fault = 'b' to 'c' to earth, fault resistance =


Evaluate

Vb; Vc; Ib; Ic at point 'V'.

[Assume no prefault load flow]

ZS1 = ZS2 = 1.0 + j 16


ZS0 = 0.5 + j 8
ZL1 = 1.8 + j 31 = ZL2
ZL0 = 10 + j 88

Self-assessment questions 7.2

Figure 7.18

The 132 kV system of Figure 7.18 is fed from end A only. The
switch at B is open.
The per unit sequence impedances of the circuit components are
as follows, to a base of 132 kV, 100 MVA:

Positive sequence
impedance

Negative sequence
impedance

Zero sequence
impedance

0 + j0.04

0 + j0.04

0 + j0.02

Transformers 0 + j0.20

0 + j0.2

0 + j0.2

Line

0 + j0.10

0 + j0.3

Source

0 + j0.10

A phase b to phase c to earth fault occurs at the mid-point of


the line and the earth fault resistance is 10 .
Determine:
(i)

7-14

the fault currents in the two faulted lines

SYSTEM FUNDAMENTALS

Section Seven

(ii)
(iii)

the voltage at the point of fault


the voltage of the unfaulted line at the mid-point of the
line.

? Self-assessment questions 7.3


(a)

Show that for a single phase-to-earth fault on phase a of a


three-phase system the fault current I a can be given as:

I a 3I 0 3I 1 3I 2
where I 0 ; I 1 and I 2 are the zero, positive and negative
sequence components respectively.
(b)

A 400 kV transmission line connects two 400 kV, threephase earthed systems.
At one end, the line is connected to the system via a delta/
earthed-star transformer of unity turns ratio. At the other
end the line is connected to the system via an earthed-star/
earthed-star transformer of unity turns ratio.
A resistive phase-to-earth fault occurs at the mid point of
the line. The value of the fault resistance is 5:0 O.
The per unit sequence impedances of the circuit
components are, to a base of 400 kV and 1 000 MVA, as
follows:

Positive sequence
impedance

Negative sequence
impedance

Zero sequence
impedance

Systems

0.006 j0.094

0.006 j0.094

0.003 j0.050

Line

0.013 j0.190

0.013 j0.190

0.060 j0.570

Transformer

0 j0.190

0 j0.190

0 j0.190

Calculate the fault current.

FAULT CALCULATIONS

7-15

Module 1

? Self-assessment questions 7.4

Figure 7.19
(a)

Draw out the sequence networks for the circuit shown in


Figure 7.19 assuming a fault at the middle of the tie line.
Assume that all generators are operating at 1:0 per unit
excitation. The per unit sequence reactances of the circuit
components are as follows, to a common voltage and VA
base.

(b)

Calculate the per unit fault current when the following


faults occur at the mid point of the tie line:

(i)

a line-to-earth fault on the a phase; and

(ii)

a line-to-line fault.

Positive sequence
reactances

Negative sequence
reactances

Zero sequence
reactances

Transformers

0.2

0.2

0.2

Generators

0.1

0.1

0.05

Tie line

0.2

0.2

0.3

? Self-assessment questions 7.5


The three busbars of a substation are linked by three, three-phase
reactors to form a ring. Each reactor has a reactance per phase of
0:09 per unit to a 10 MVA, 11 kV base. Each busbar is supplied
by one 11 kV, three-phase, 10 MVA alternator with a subtransient
reactance of 0:15 p.u. A 20 MVA transformer of 0:1 p.u.
7-16

SYSTEM FUNDAMENTALS

Section Seven

reactance connects one busbar to a 132 kV transmission line. The


line has a reactance of 25 per phase.
Resistance may be ignored.
A balanced three-phase fault occurs at the far end of the line.
(a)

Determine the magnitude of the fault current.

(b)

Determine the minimum kVA rating of the switchgear


needed at the point of fault to clear the fault safely.

(c)

Determine the current in each of the three alternators.

FAULT CALCULATIONS

7-17

Module 1

7-18

SYSTEM FUNDAMENTALS

Section 8
TRANSMISSION LINE
CHARACTERISTICS

8.0 Objectives
When you have completed this section you should be able to:

determine the capacitance and inductance of lines and cables


in simple regular arrangements

calculate the voltage induced between parallel arrangements


of conductors

set up a line representation using four-terminal networks to


calculate the conditions on lines

use characteristic impedances to calculate changes in


conditions on a transmission line

use P or > models of transmission lines to make simple


calculations of voltage, current, load flow and power factor
conditions on a line

construct a simple chart to show how the operating


condition of a line may be changed by changes in terminal
voltages

use the simple chart, or mathematical analysis, to determine


changes in power and var flow caused by changes in reactive
loads.

8.1 Introduction
The simplest view of a transmission line is that it is a length of
conductor used to connect a load to a source or to interconnect
two supply systems. As with any conductor it has inductance,
capacitance and resistance. The simple view would suggest that
the resistance is the most important factor because the I 2 R loss
and its associated heating will place the only limit on power
transfer.
TRANSMISSION LINE CHARACTERISTICS

8-1

Module 1

This limit exists, but for high voltage, long transmission lines
other factors are just as important. We have already seen that
there is a theoretical limit on the power that can be transferred
between a synchronous source E and a synchronous load V and
that the limit is determined by the load angle between the two
voltages and by the transfer impedance between them. The
inductance is a critical component of this transfer impedance.
Capacitance is also significant on long, high voltage lines. Reactive
current has to flow along the line to charge up the capacitance of
the line. This will affect the voltage distribution along the line. An
underground cable is far more capacitive than an overhead line
and will need proportionately more capacitive current to charge it.
The design and operation of long transmission lines is not
therefore a simple issue and lines are a very significant part of the
transmission and distribution network.
The inductance, capacitance and resistance of a transmission line
are distributed parameters. That is, each infinitesimal section of a
line has its own inductance, capacitance and resistance. An
accurate mathematical model of this is complex, but we only need
such accuracy when considering high frequency, or transient,
effects on a power line, or when dealing with exceptionally long
power lines. Fortunately, we can establish the man principles of
power line operation by using good approximate models that are
easy to set up and deal with.
In this section we will be examining how the physical dimensions
of a line determine its inductance and capacitance and we will use
these derived values to establish simple models to describe the
operation of the line. The models will be based on P and >
network approximations which are justified for line lengths of up
to 150 km. These models are then used to construct a simple line
chart from which we can determine the practical power transfer
capabilities of the line.

8.2 The series inductance of a two-conductor


line
Figure 8.1 represents a section through two, long, cylindrical,
parallel conductors, A and B. The effect of earth is ignored and the
current I is assumed to go up conductor A and return via
conductor B.

8-2

SYSTEM FUNDAMENTALS

Section Eight

Figure 8.1
The magnetic field set up by a long, cylindrical conductor has a
circular symmetry about the axis of the conductor and at a radius
x away from the axis the magnetising force H is given by:

I
2px

The magnetic flux density is given by: B m0 H , where m0 is the


permeability of free space and has a value m0 4p6107 Hm1 .
Thus the magnetic flux density at a radius x from the axis of
conductor A due to the current I in A is:

Bx m0 H

m0 I
2px

Imagine now a thin cylindrical shell of radius x, length l and wall


thickness dx, centred on the axis of A. The magnetic flux through
this shell will be:

dF Bx 6l6dx
The total magnetic flux existing between the surface of conductor
A and the surface of conductor B will be the sum of a series of
small fluxes dF as the variable x increases from x r to
x D  r . i.e.

ZDr



m0 I
m0 I
Dr
l dx
l ln
2px
2p
r

This flux links the conducting loop formed by the conductors A


and B.
We also have a flux created by the returning current in B. This will
add to the flux created by the current I in A and simple symmetry

TRANSMISSION LINE CHARACTERISTICS

8-3

Module 1

suggests that it will have the same magnitude. So, the total flux
linking the two conductors is:



m0 I
Dr
l ln
FT
p
r
Inductance is defined as flux-linkages per ampere, so we can define
the self inductance of a two-conductor line as L, where:



FT m0
Dr
l ln
L
HenriesH
I
p
r
or, writing this as a per unit length value:



FT m0
Dr
ln
L
Hm1
I
p
r
The calculation for a three conductor line is a little more involved
and becomes difficult if the line is not symmetrically spaced or if
the conductors are transposed. In a transposed line the conductors
are crossed over every so often, so that the a conductor moves to
the position of the b conductor, the b conductor moves to the
position of the c conductor and the c conductor moves to the
position of the a conductor. This tends to happen only on very
long lines.
The effect of earth wires and of the ground also need to be taken
into account for exact solutions, particularly when transient
conditions are being analysed. You can imagine how difficult this
can be when you consider that the resistivity of the ground will
vary with ground and weather conditions along the length of the
line.
However, we have developed a simple equation for the inductance
of a simple line which shows the way that inductance varies with
line separation. We also have a straightforward method of analysis
which we can use to calculate the electromagnetic interference
between adjacent lines.

8-4

SYSTEM FUNDAMENTALS

Section Eight

8.3 Mutual inductance and electromagnetic


interference

Figure 8.2
In Figure 8.2 we represent a two-conductor power line, AB, and a
parallel two-conductor communication circuit, CD. We will define
the distances separating the conductors as dAB ; dAC . etc and
assume that the radius of all conductors is small with respect to
the distances between conductors.
Following the procedure of the previous section, we can define the
flux passing between conductors C and D due to current I in
conductor A at a distance x from the axis of A as:

dF

m0 I
ldx
2px

The total flux linking the circuit CD due to the current in A is:

FA

xd
Z AD



m0 I
m0 I
dAD
l dx
l ln
dAC
2px
2p

xdAC

The flux linking the circuit CD due to the current I in conductor B


is:



m0 I
dBD
l ln
FB
dBC
2p
However, FB is opposing FA so the total effective flux linkage
with CD is:
TRANSMISSION LINE CHARACTERISTICS

8-5

Module 1

 



m0 I
dBD
dAD
ln
l ln
FT
dBC
dAC
2p
 

m0 I
dBD dAC
FT
l ln
dBC dAD
2p
The mutual inductance, M , between the two circuits is defined as
the flux-linkages per ampere, so that:

 

m0
dBD dAC
l ln
M
H
2p
dBC dAD
or

 

m0
dBD dAC
ln
M
Hm1
2p
dBC dAD
Calculating the induced emf, e, is now straightforward because:

eM

dI
dt

and, if

I isinot
then:

d
e Mi sinot Miocosot per metre length
dt

Exercise
Four long, straight conductors lie
in the same horizontal plane
with 1 cm between adjacent
conductors. The two left-hand
conductors supply a circuit with
500 mA r.m.s current at
12:5 kHz. Determine the e.m.f.
per metre run induced around
the loop formed by the two
right-hand conductors and the
mutual inductance between the
two loops per unit length.

8-6

SYSTEM FUNDAMENTALS

Section Eight

Assume a steady current in the left-hand conductors of Figure 8.2.


The flux linking the loop CD as a result of current I in A is:

A =

x = dAD

0 I
I
3
l dx = 0 l ln
2 x
2
2
x = dAC

The flux linking CD as a result of current I in B is:

B =

0 I
l ln 2
2

The total flux linking the loop CD in an anticlockwise direction


about A is:

T =

0 I
4
l ln
2
3

The mutual inductance per unit length, M, is the flux linkage per
ampere which is calculated as 0.58 107 Hm1.
The induced e.m.f. per metre run of CD is:

e=M

dI
= M 0.5 0.2 0.5 2 f cos 2 ft
dt

The r.m.s. value is calculated as 2.3 mVm1.


This example of mutual coupling should seem obvious and it
clearly has relevance to situations where communication circuits
run parallel to power lines. But, mutual coupling of this sort also
exists between the various phases of a multi-phase conductor
system; between adjacent circuits, where for example two threephase supplies run in parallel on one set of overhead towers; and
between phase conductors and the earth wire. The series
impedance of a long transmission conductor is therefore quite a
complex thing. It depends not only on the self impedance of the
conductor, but also on the effects of currents in adjacent
conductors and, possibly, the ground.

TRANSMISSION LINE CHARACTERISTICS

8-7

Module 1

8.4 The self inductance of a co-axial cable

Figure 8.3
In this case we assume that the current I is supplied by the central
conductor and that it returns via a second conductor or, in the
case of low level signals, along the co-axial sheath. Either way, the
conducting sheath of a co-axial cable protects the inner from the
effects of externally generated electromagnetic fields. The only
magnetic flux within the cable is produced by the current I in the
main conductor. Following our earlier examples we have:

m0 I
l dx
2p
 
m0 I
R
l ln
FT
2p
r
 
FT m0
R

l ln
L
I
2p
r
 
m
R
Hm1
L 0 ln
2p
r
dF

If the outer sheath is an effective screen and is earthed correctly at


each end, then there should be no mutual coupling between the
conductor and any other circuit.
High voltage cables are usually of a co-axial construction with
solid metal screening around each phase conductor. Each phase
conductor is laid separately. Medium voltage, three-phase cables
may have all three phase conductors bound within one cable
structure, but each phase conductor will be individually screened
to control the electric fields within the structure and this screening
will minimise mutual coupling and hence mutual inductance.

8-8

SYSTEM FUNDAMENTALS

Section Eight

Low voltage wiring will not have screening about the conductors
and mutual electromagnetic coupling will be maximised. This will
be a major problem when there are high frequency fields or where
transient conditions occur. Wiring in any control or data network
which may be susceptible to unwanted interference should always
be screened.

8.5 The shunt capacitance of a two-conductor


line

Figure 8.4
Figure 8.4 represents a section through two, long, cylindrical
conductors. The effect of earth is ignored and the charge
displacement is balanced with Q coulombs on conductor A and
Q coulombs on conductor B.
Electric field strength E is defined as acting positively along the
line of increasing x. We can then define the potential of a point at
x a with respect to a point x b as:

Vab 

Za
Edx
b

At a point a general distance x away from the axis of conductor A,


as defined in Figure 8.5, the electric field strength has two
components: one due to the charge on A and one due to the
charge on B. We assume that QA QB so that the total charge
on the system is zero.

TRANSMISSION LINE CHARACTERISTICS

8-9

Module 1

Figure 8.5
Theoretically it does not matter which path we take when we
determine the potential difference A and B, but for ease of
calculation we choose the one which gives the easiest solution. In
this case the choice is the shortest line between the two axes. At
the point x along this line the two components of E are:

E at x due to charge on A,
EA

Q
2pe0 xl

and E at x due to charge at B,

EB

Q
2pe0 d  xl

[If you are unsure about this then you need to refer to a basic text
on electrical theory and look up Gauss Law]
The constant Eo is the permittivity of free space and has a value
109 =36p Farads per metre (Fm-1). If the material between the
conductors is an insulator other than free space (or air) then we
have to multiply Eo by the value of relative permittivity for the
material.
The resultant electrical field strength ER is:



Q
Q

ER
2pe0 xl
2pe0 d  xl
so that the voltage between A and B is:

VAB
VAB



Q 1
1

dx

2pe0 l x dx
dr


Q
dr
ln

pe0 l
r
Zr

This gives the capacitance C as:


8-10

SYSTEM FUNDAMENTALS

Section Eight

Q
pe0 l

 Farads F
VAB ln dr
r

or, alternatively, the capacitance per unit length is:

pe0

 Fm1
dr
ln r

Exercise
Two long, straight, cylindrical
conductors run in parallel to
form a go and return circuit.
Each conductor has an outside
diameter of 2 cm and they are
separated by a distance of 1 m
between centres.
Determine the capacitance
between the conductors per
metre run.

The answer is obtained by straight substitution and should be


C 6:04 pF=m

TRANSMISSION LINE CHARACTERISTICS

8-11

Module 1

8.6 The concentric cable


A section through such a cable is represented in Figure 8.6.

Figure 8.6
Assume a charge Q on the inner conductor so that the electric
field E acts radially outwards from the centre in the direction of
increasing x.
The potential of the inner conductor with respect to the outer is
then:

V 

Zr
Edx
a

But, the electric field strength E at x is:

Q
2pe0 xl

so that

V 

Zr
a

a
Q
Q
dx
ln
2pe0 xl
2pe0 l
r

which gives:

8-12

SYSTEM FUNDAMENTALS

2pe0
  Farads per metre length:
a
ln r

Section Eight

Exercise
A concentric cable consists of an
inner conductor of diameter 2
cm and an outer conductor of
internal diameter 6 cm.
Polythene tape is wrapped
around the inner conductor to a
thickness of 1 cm and then a
layer of Nylon tape is laid on
top of the polythene to fill
completely the remaining space
between the two conductors.
Calculate the capacitance of the
cable per metre run. Take the
relative permittivity for
polythene as 2.4 and the
relative permittivity of Nylon as
3.6.

Figure 8.7

A cross section through the cable is shown in Figure 8.7. The


voltage between the inner and outer conductors is given by:
r3

r2

r2

r1

V = E1dx E2 dx
E1 at x for r2 < x < r3 =

Q
2 0 1 xl

E2 at x for r1 < x < r2 =

Q
2 0 2 xl
TRANSMISSION LINE CHARACTERISTICS

8-13

Module 1

so that:


 
 
Q
1
r3
1
r2
ln
V
ln
r2
r1
2pe0 l e1
e2
Substitution gives:

Q
140 pF=m
Vl

8.7 Simple line models


From what we have done so far we can see that the simplest
representation of a two conductor line will show a series reactance
XL 2pfL per unit length and a shunt reactance

XC

1
2pfC

per unit length, where L and C have the values already defined.
If we allow for the possibility of losses along the line then the
series impedance Z R joL should be used and it will be
easier to work with a shunt admittance Y joC rather than a
shunt reactance.
For lines less than about 150 km long we can choose one of two
approximate representations: either the P equivalent circuit or the
> equivalent circuit. These are illustrated in Figures 8.8 and 8.9
respectively.

Figure 8.8

8-14

SYSTEM FUNDAMENTALS

Section Eight

Figure 8.9
In practice, there is little to choose between the two models
although for some applications one may be better suited than the
other.
Now, what we need to do is to develop the models a little bit
further so that we end up with the standard four-terminal (or twoport) representation of a transmission line. This will enable us to
represent the line by the equivalent circuit model shown in Figure
8.10 and by the two following equations. Unfortunately this
means a bit of tedious manipulation, but it is worth doing because
it will show us exactly how the A B C D constants are related to
the basic parameters of the overhead line (or even underground
cable!).

Figure 8.10

VS AVR BIR
IS CVR DIR

Exercise
Refer back to Figure 8.8 and try
to show that the following
equations are valid:


ZY
VS 1
V R ZI R
2
"
#


ZY 2
ZY
IS Y
VR 1
IR
4
2

TRANSMISSION LINE CHARACTERISTICS

8-15

Module 1

From Figure 8.8, we have:

V S ZI B

IC
Y=2

IB IR IC
V RY
IC
2
;
;
or

V RY
IB IR
 2

V RY
VR
V S Z IR
2



ZY
VS 1
V R ZI R
2

We also have:

IS IA IB
and

IA
 ZI B
VR
Y=2

Y
2
Y
I S V R ZI B I B
2

V RY
ZY
IB
1
IS
2
2
I A V R ZI B

or

;
or

Substituting for IB gives us:




V RY
V R Y ZY
IR
1
IS
2
2
2
or

"

ZY
IS Y
4


ZY
VR 1
IR
2

We have now shown that the P circuit can be represented by the


following two port matrix equation:

8-16

VS
IS

SYSTEM FUNDAMENTALS

ZY
1 2
6
4
ZY 2
Y 4

3


7 VR
5
IR
ZY
1 2
Z

Section Eight

or

VS
IS

B
D



VR
IR

The values of A B C D can be read directly from the matrix.


Exercise
If you are still feeling strong,
show that AD  BC 1:0

The > equivalent circuit gives rise to a similar matrix


representation as follows:

VS
IS

ZY
61 2
4
Y

3
Z2 Y 
Z 4 7 V 
R
5
I
R
ZY
1 2

Whichever model we use, we can see that the B constant is


dominated by the series impedance Z and the C constant is
dominated by the shunt admittance Y . The A and D constants
are equal in both models and will have a value close to unity.
Note 1: the A B C D constants are complex numbers
Note 2: the equations for V s and for I s refer to phase quantities.
Exercise
An overhead line has a total
series impedance Z
4 j12 O and a shunt
admittance of

Y 0 j0:756102 S.
Determine the A B C D
constants for a P representation
of the line.

TRANSMISSION LINE CHARACTERISTICS

8-17

Module 1

For the network:

( 4 + j12 ) ( 0 + j0.75 10
ZY
A = 1+
= 1+
2
2
B = Z = 4 + j12
C =Y +

) = 0.955 + j0.015

ZY 2
= 0.0056 + j 0.733S
4

D=A

Exercise
A balanced three-phase overhead
line has constants A =
0.910 and
B = 10064 . The
receiving-end line voltage is held
at 400 kV. Determine the line
voltage at the sending end
needed to deliver 900 MW at
0.9 power factor leading at the
receiving end.

900
= 1000 MVA
0.9
1000 106
= 144326D A
IR =
3
3 400 10

S=

VS = AVR + BI R
40010D
D
+ 1443 100 ( 26 + 64 ) V
3
VS = 204.64 + j36.1 + J 144.3 kV
VS = 0.9

VS = 272.841.4D kV
VS(Line) = 472.541.4D kV
[Note the large value of the load angle which would be
unacceptable in practice].

8-18

SYSTEM FUNDAMENTALS

Section Eight

8.8

Line performance chart

As shown earlier:

From which:

Taking the receiving-end voltage VR as the reference phasor we


can draw the phasor diagram of Figure 8.11 to represent this
equation.

Figure 8.11

In this figure we have split the receiving-end current IR into two


components: IRP which is in phase with VR and IRQ which is at a
right angle to VR. IRQ is the quadrature component of the load
current and in this case it is taken as a leading component i.e. the
receiving-end power factor is assumed to be leading.
The active and reactive power at the receiving end are given by:

PR = 3VRIRP
QR = 3VRIRQ
[Note the negative sign to represent the leading vars].
Remember from the first section that lagging vars are associated
with an inductive load in which the current lags the voltage and
by convention a positive sign is used. Leading vars are used for
capacitive loads in which the current leads the voltage and in this

TRANSMISSION LINE CHARACTERISTICS

8-19

Module 1

case the sign for vars is negative. It can be said that an inductive
load absorbs vars and a capacitive load generates vars.
From the phasor diagram of Figure 8.11 it is apparent that because
the B parameter of the line is effectively a constant for any given
application, the following relationships apply:

jBjIRP ! PR
jBjIRQ ! QR
We can use this to scale the phasor diagram in units of power. In
particular, any voltages, V 0 say, along the BIRP axis and V 00 along
the BIRQ axis are related to the received power by:

PR 3jV R j

V0
jB j
Q R 3 jV R j

jB j
V 00

Exercise
A three-phase transmission line
has a two-port B parameter of
5070  O=phase. The sending
and receiving end voltages are
held constant at 400 kV. If a
line performance chart is now
constructed, what will every 10
kV along the V 0 and V 00 axes
actually represent in terms of
real power (MW) and reactive
power (Mvar)?

400
jV R j jV S j p 231 kV
3

8-20

SYSTEM FUNDAMENTALS

Section Eight

Now, for every 10 kV along the BIRP axis on the chart,

I RP =

10 103
= 200 A
B

Every 10 kV along this axis therefore corresponds to:

3 231 103 200 = 138.6 MW


Along the BIRQ axis every

10 kV corresponds to 138.6 Mvar.

It can be seen therefore that we can scale the BIRP and BIRQ axes to
represent real and reactive power at the receiving end of the line.
This forms the basis of the line performance chart.
A positive value of PR clearly represents power arriving at the
receiving end whereas a negative value of PR means that power is
being sent from the receiving end to the sending end of the line.
A negative value of QR means that the reactive power received at
the receiving end of the line is leading and conversely when BIRQ
is positive the reactive power received is lagging. These two cases
correspond to a capacitive load and an inductive load respectively.
We can now generalise the line chart as shown in Figure 8.12
which shows the axes for real power and reactive power with the
origin for power measurement at the tip of the AVR phasor.

Figure 8.12
Several important features of the active and reactive power transfer
characteristics of a transmission line are readily apparent from

TRANSMISSION LINE CHARACTERISTICS

8-21

Module 1

Figure 8.12. In practice we must arrange that the magnitude of the


voltages at sending and receiving ends do not deviate significantly
from normal designed levels, since any significant deviation from
normal levels will be reflected down through the network to the
power distribution system and will result in a poor quality of
supply from the point of view of voltage stability. Normally, if the
receiving end voltage is maintained at its nominal value then the
magnitude of the sending end voltage should be such that:

 
V S 
0:95h h1:05
VR
There is also a requirement to limit the angle between the voltages
at each end of the line. This is the load angle of the line and for
stability reasons this is limited to within +308.
It is thus clear that in practice one is not entirely free to specify
any arbitrary power PR or reactive power QR to be transmitted to
the receiving end of a transmission line. In fact it is very evident
that, within the typical range of voltages possible, only certain
reactive powers can actually be transmitted to the receiving end.
In a practical system, these constraints are met by employing
special equipment on the system essentially to control the reactive
and active power flows.
It should be noted that the reactive power associated with power
systems is very closely bound up with the whole matter of voltage
control whereas the active power transmitted is largely dictated by
the load angle of the line.
An overhead earth wire can help to improve the capacity of a line
to transfer power. It can be shown that the earth wire reduces the
effective series impedance of the line, which of course means a
reduction in the B parameter. If you think back to the work we
did on synchronous sources and synchronous loads you may
remember that the maximum power that could be transmitted
between two voltages depends inversely on the transfer
impedance. Thus a reduction in B will mean an increase in the
maximum possible power transfer.
Surge impedance loading
This is an interesting case where the load on the line is resistive,
i.e. at unity power factor, and equal in value to the characteristic
impedance of the line.

r

 r
 
Z
L=
Zo
=Y
C
With this loading, there is no voltage drop along the line
8-22

SYSTEM FUNDAMENTALS

Section Eight

(assuming that the line is lossless) and there is no reactive power


flow along the line.
Example
A balanced, three-phase, overhead line has constants
A 0:9510  and B 11065  O. The receiving-end and
sending-end phase voltages are maintained at 325 kV and 385 kV
respectively to deliver a three-phase load of 1 500 MW at 0:9
power factor leading.
(a)

Draw a line chart to represent this condition;

(b)

Given that the real power is now reduced to 750 MW and


that the terminal voltages remain unchanged, determine
the reactive power now received at the receiving end of the
line.

(c)

If the total VA at the receiving end were to be reduced to


zero and the receiving end voltage were to be maintained at
325 kV, what value would the sending-end voltage need to
be?

We will adopt a slightly different approach to line charts here and


construct it as a current diagram rather than a power diagram.
The basic equation is:

V S AV R BI R
so

IR

V S AV R

B
B

Allowing for the phase angles, and taking V R as the reference


phasor, we have:

IR f

jV S jd jAja

6jV R j0
jBjb jBjb

AV R
2:816103  55 A
B
VS
3:56103 d  65 A
B
5006106
I
1:716103 25:84 A
3
325610
If we choose a scale of 1 cm 0:66103 A, then we can draw
the three phasors as in Figure 8.13.

TRANSMISSION LINE CHARACTERISTICS

8-23

Module 1

Line

OS = 4.68 cm, represents the phasor

AVR
55D

Line

OT = 2.85 cm, represents the phasor I.

The line ST then represents the phasor

VS
B
The horizontal intercept OW represents the in-phase current.
We now reduce the real power to one-half the original value, so
the new in-phase current is represented by OX where
OX = OW/2.
The terminal voltage remains unchanged, so the lengths of
phasors OS and ST cannot change. The angle of

AVR
B
cannot change, so the only possible variable is the angle between
the phasors, OST. The point T can therefore move on a circle
of radius ST centred on S. The new operating point must
therefore be the point T which defines the new current OT.
Thus IQ = XT = 0.5 cm = 0.5 0.6 103 = 300 A
and Q = 3 300 325 103 = 292.5 Mvar.
The power factor is now lagging
Finally, if we reduce the receiving-end
current phasor must be zero, so that:

AVR VS
=
B
B
or

VS = AVR = 308.7510 kV

8-24

SYSTEM FUNDAMENTALS

VA to zero, then the

Section Eight

Figure 8.13

? Self-assessment questions 8.1


An overhead line has a total series impedance Z of 5 j13 O
and a shunt admittance of j0:756102 S.
Determine:
(a)

the approximate A; B; C; D constants of the line based


upon P and > representations; and

(b)

the A; B; C; D constants of the line when its capacitance


is neglected.

Determine also the magnitude and phase of the sending-end


voltage relative to the receiving-end voltage when a load of 30
MW at 0:85 power factor lagging is supplied. The receiving-end
line voltage is 132 kV. Compare the results obtained by using the
approximate equivalent circuits of the line and comment on the
comparison. The given parameters are consistent with a line
length of approximately 30 km.

? Self-assessment questions 8.2


Outline the basis of a line performance chart for an overhead line.
A balanced three-phase overhead line has constants
A 0:9510 and B 11065 O. If the receiving-end phase
voltage is held constant at 325 kV, determine graphically or
otherwise:
TRANSMISSION LINE CHARACTERISTICS

8-25

Module 1

(a)

the sending-end phase voltage necessary to deliver a load of


1 500 MW at 0.9 power factor leading at the receiving
end; and

(b)

the maximum power that can be transmitted if the


sending-end voltage is held constant at the value
determined in part (a).

?
(a)

Self-assessment questions 8.3


An overhead transmission line consists of two long, parallel
conductors each of radius r separated by a distance D. The
conductors form a go-and-return circuit. Starting with the
magnetic circuital law, show that the self-inductance of the
line is given by:

where
(b)

Figure 8.14

8-26

0 = 4 107 Hm1 is the permittivity of free space.

A four-wire, three-phase, 50 Hz overhead line ABCN is


represented in Figure 8.14 with separations as shown. A
communications line DE runs parallel to the line with its
conductors in the same horizontal plane as the three phase
conductors, A, B and C.
A third harmonic current of 100 A r.m.s. flows in the transmission line.
Calculate the voltage induced in the communication line per
kilometre length.

DAB = DBC = DBN = 1.2 m


DCD = 1 m
DDE = 0.5 m

SYSTEM FUNDAMENTALS

Section Eight

? Self-assessment questions 8.4


Conditions on a long, balanced, three-phase overhead
transmission line are described by the following equation:

V 1 AV 2 BI 2
where A 0:9510

and

B 11065 O.

The phase voltage at end 2 is held constant at 232 kV and the


following operating conditions are applied:
(a)

the phase voltage at the sending end, V 1 , is constrained so


that

 
V 1 
0:95h h1:05
V2
and
(b)

the phase angle, d, between V 1 and V 2 is constrained so


that

30  hdh30

Determine graphically or otherwise the limiting values of reactive


power that can be transferred by the line to the receiving end
under all allowed operating conditions.

TRANSMISSION LINE CHARACTERISTICS

8-27

Module 1

8-28

SYSTEM FUNDAMENTALS

Section 9
PRINCIPLES OF PROTECTION

9.0 Objectives
When you have completed this section you should be able to:

calculate the operating conditions for an overcurrent


protection system used, for example, for the protection of
radial feeders

make simple calculations for the settings of directional


overcurrent relays in a ring-distribution system

calculate conditions in a Distance Protection System using


stepped characteristics

calculate conditions in a Current Differential Protection


Network used for unit protection.

9.1 Introduction
In power systems all plant is subject to occasional failure and,
when this happens, has to be removed from the system. To detect
such failures we employ power system protection measuring
equipments. Protection is a measuring technique which monitors
the state of the plant and initiates its removal from the system if
and when faults occur. The need for protection is obvious in that:
(a) it provides for the removal of faulted plant so as to aid the
stability of the rest of the system and (b) it limits damage to plant
due to excessive fault current. In broad terms the main properties
of protection are as follows:
(i)

It must only initiate tripping when the item of plant is


faulted and must ideally initiate tripping of the circuit
breaker necessary to remove only the faulted plant a
property often referred to as discrimination.

(ii)

Ideally it must trip the circuit breaker quickly this


property aids system stability.
PRINCIPLES OF PROTECTION

9-1

Module 1

(iii)

It must be very reliable.

This course module deals with some of the basic principles of


different types of protection used on power system transmission
and distribution systems. It also shows simple setting calculations
for various protection relays employed on a particular part of the
system. In practice correct relay settings are germane to attaining
the required protection discrimination.

9.2 Types of protection


Protection devices may be classified into two basic types nonunit systems and unit systems.
Non-unit Protection
These are time-graded systems which utilise information (currents
and voltages) derived from a particular point in the system.
Overcurrent and Distance protection are prime examples. Time
grading is applied through a sequence of equipments so that when
a fault occurs a number of protection devices respond, but ideally
only those closest to a fault complete the tripping function. The
others make incomplete operations and then reset.
Overcurrent Protection
The most common non-unit scheme which is widely applied in
both distribution and transmission systems is the overcurrent
device.

Exercise
What is the basic difference
between a transmission and
distribution system, in terms of
protection requirements?

A transmission system is a high voltage interconnected system in


which generators are connected via transmission lines for the
purposes of bulk power transfer. Voltage levels are very high
(275 kV and 400 kV in the UK; 500 kV and above used in other
parts of the world). Very fast, expensive and highly sophisticated
9-2

SYSTEM FUNDEMENTALS

Section Nine

protection is employed on such systems in order to minimise the


risk of losing a lot of power. A distribution system is used for
distributing power to the consumers. Normally, it comprises lines
and cables which are fed from one end only with load taps at
various points along the lines. Voltage levels are relatively low
(33 kV and 11 kV) for large industrial consumers and 415 V (line
voltage) and 230 V (phase voltage) for the domestic consumers.
Relatively slow and less expensive protection is normally
employed on such systems since a loss of line would affect only a
very small number of consumers.
Overcurrent protection is applied as the primary and often the only
protection scheme at distribution voltages and is also applied in
transmission systems as back-up protection.
Exercise
What is the fundamental
difference between primary and
back-up protection?

Primary (or main) protection has the fastest measuring speed and
is designed to be absolutely discriminative. Back-up protection
often operates at a relatively slow speed and has relatively low
discriminative properties. It is provided as a back-up in case of
primary protection failure.
The protection device relies upon a comparison of the current
measured via the Current Transformers (CTs) at a particular point
in the system. This comparison is often arranged to produce
particular current versus tripping time response curves according
to the requirements of the system. In the simplest case, tripping is
initiated when the measured current exceeds a pre-set value.

PRINCIPLES OF PROTECTION

9-3

Module 1

Exercise
Why is it essential to employ a
Current Transformer for current
measurements? In a particular
overcurrent protection system a
CT with a 500 : 1 ratio is
employed. If the level of fault
current is 2:5 kA, what will be
the current passing through the
overcurrent relay?

Under fault conditions, the level of fault current flowing through


the distribution line becomes very high and is therefore not
suitable for the protection device. A CT is thus employed to step
down this current to a more suitable level.
With a CT ratio 500:1
and Ifault 2.5 kA
then current through the device is

Ifault/CT ratio 2 500/500 5 A


These relays have an inverse operating-time/current characteristic
and a definite minimum operating time. These can be arranged to
provide for discriminative tripping by suitably adjusting the
individual relays so that their operating times versus current
curves bear a definite relationship with respect to one another.
In terms of relay settings there are essentially two control
parameters:
(i)
(ii)

plug setting which defines the relay sensitivity


time multiplier setting for co-ordinating time gradings of
relays on a system to attain discrimination.

Relay Plug Setting


Plug setting, i.e. current in the relay, depends upon the relay
current setting which in turn is given as percentage plug setting.
Typical plug settings for minimum current operation are 40% of
full load current for earth faults and 125% of full load current for
other types of fault. The current in the relay is calculated as a
multiple of plug setting and is given by:

Plug setting multiplier 100 I/[CT ratio 6 %plug


setting 6 nominal rating]
9-4

SYSTEM FUNDEMENTALS

Section Nine

where I is the actual fault current.


Exercise
A current transformer with a CT
ratio of 500 : 1 feeds an
overcurrent relay with the
following setting:
100%
(i)
(ii)
40%
(iii)
125%
Determine the current in the
relay (as a multiple of plug
settings) if the fault current is
2 kA, assuming nominal relay
rating of 1 A.
(i)
(ii)
(iii)

I1 100 6 2 000 / 500 6 100 4 plug settings


I2 100 6 2 000 / 500 6 40 10 plug settings
I3 100 6 2 000 / 500 6 125 3:2 plug settings

It is apparent from the above that a lower percentage setting makes


the relay more sensitive. In practice this facility is very useful (and
desirable) for making the relay operating times comparable for
earth and other types of fault. Levels of fault currents for earth
faults are generally lower than for other types such as three-phase
and phase-phase. A typical arrangement for a three-phase system
involving the three elements of the relay is as shown in Figure 9.1
from which it can be seen that the aforementioned differing
sensitivities for different types of fault can be achieved by making
only one relay element responsive to earth faults.

Figure 9.1
A typical overcurrent relay
arrangement

PRINCIPLES OF PROTECTION

9-5

Module 1

Exercise
In practice why is it desirable to
have nearly the same relay
operating times for all types of
fault?

In a feeder arrangement of the type shown in Figure 9.2 (a) in


which a number of overcurrent relays are involved, an abnormally
long operating time of the faulted section relay for a particular
type of fault could very easily upset the relay co-ordination, thus
affecting relay discrimination.
Relay time-multiplier setting
Time multipliers enable the co-ordination of the time grading so
that sufficient discrimination may be provided to isolate a faulty
feeder correctly. In the simple radial feeder system of Figure 9.2(a)
we would have to grade the time/current characteristics with
differing time-multiplier settings typically as shown in Figure
9.2(b). It should be noted that all overcurrent relays of the type
discussed above are, in a sense, universal relays with a universal
relationship between operating time and relay current in multiples
of plug setting typically as shown in Figure 9.3.

9-6

SYSTEM FUNDEMENTALS

Section Nine

Figure 9.2
Protection of a radial feeder
system (a) System arrangement
(b) Relative time-multiplier
settings

Figure 9.3
Universal relay characteristic
This characteristic is better
described by its formula

0:14
psm0:02  1

where psm is the plug setting


multiplier.

Furthermore, in any application, the problem of deciding suitable


time-multiplier settings is governed by breaker operating time.

PRINCIPLES OF PROTECTION

9-7

Module 1

Exercise
An overcurrent relay with an
inverse characteristic shown in
Figure 9.3 sees a fault current on
a feeder of 4 plug settings. If the
breaker operating time is 0:2 s
and an overall fault clearance
time of 1 s is required,
determine the relay timemultiplier setting.

For a plug setting of 4, the operating time from Figure 9.3 using
the charactistic equation is 4:98 s. The relay must operate in
1  0:2 0:8 s. Therefore time-multiplier
setting 0:8/4:98 0:16

9.3 Protection settings for a ring distribution


system
You will see from the foregoing that radially fed power systems
have a very low integrity, as a high fault [say in the first section of
Figure 9.2(a)] will result in the loss of supply to sections 2 and 3.
A better alternative power supply is to use a further
interconnection between the remote substation and the supply
substation as shown in Figure 9.4.

Figure 9.4
Ring distribution system
Unfortunately this means that a simple time-graded overcurrent
system of the type described before cannot be used as power and
fault current can flow in both directions. To provide
discrimination, directional relays must be used. These are arranged
to operate when fault current flows out of the feeder. The contacts
of the relay are arranged in conjunction with the over-current
relay to allow measurement when closed and prevent
measurement when open.
9-8

SYSTEM FUNDEMENTALS

Section Nine

At the supply substation there is no necessity to use directional


relays as the only possible way that fault current can flow is into
the feeder.
The arrows on the system diagram in Figure 9.4 indicate the
direction of current flow which will operate the relay at that
location. Examination of the system shows that, for protection
purposes, the system can be regarded as two radial systems. One
concerned with relays A1, B1, C1 and D1 and the other involving
relays A2, D2, C2 and B2.
The fault level at the end of each feeder would be calculated by
assuming the circuit breaker at A2 is open when calculating the
fault level for relays A1, B1, C1, and D1. And the circuit breaker at
A1 is open when calculating the fault level for relays A2, B2, C2
and D2.
In terms of relay co-ordination for the system shown in Figure 9.4,
this can best be achieved by treating the system as two radial
systems, one comprising relays A1, B1, C1 and D1 and the other
comprising relays A2, B2, C2 and D2. For this aspect of radial
systems, relays may conveniently be co-ordinated by starting at
the farthest load point and working towards the source. Each relay
is successfully set such that downstream relays operate before
upstream relays.
With reference to Figure 9.4, for a particular fault level at the
incoming busbar, a typical relay co-ordination in terms of time
multiplier settings (TMS) will be:

9.4 Distance protection


The distance or impedance measuring principle represents the
most widely applied non-unit protection as primary (or main)
protection to high voltage transmission lines. The basic measuring
principle is simple in that from a knowledge of the various
voltages and currents at one end of the line, it is possible from the
ratio of the measured voltage to current, to determine the
impedance between the relay and a particular fault point on the
line. Figure 9.5 illustrates the basic principle.
PRINCIPLES OF PROTECTION

9-9

Module 1

Figure 9.5
Basic distance relay principle
With reference to Figure 9.5, if the distance from the relaying
point to the fault is x, then the impedance measured for a zeroresistance fault is given as:

V=I xZL
where ZL positive phase line impedance/metre.
The measured impedance follows the typical locus shown in Figure
9.6.

Figure 9.6
Typical impedance locus
It is important that the distance relay, say at end A in Figure 9.5,
sees faults only on the feeder AB, i.e. forward faults, and does not
operate for reverse faults behind A. Furthermore, in practice there
is very often a small resistance present in the fault. The Distance
protection is thus arranged to compare the circuit impedance with
a defined area of the measured impedance plane so that any fault
within the defined area causes tripping of the circuit breaker. A
very common form of arranging the Distance protection
characteristic is shown in Figure 9.7a (also known as the MHO
characteristic) which very adequately meets the two
forementioned requirements.
The primary circle shown in Figure 9.7a is constructed on the
diameter AB of Figure 9.6. AB is the impedance of the protected
length of line. If the value of line impedance to the point of fault
plus any fault resistance lies inside this circle, then the protection
will operate.
9-10

SYSTEM FUNDEMENTALS

Section Nine

Figure 9.7
Relay MHO characteristics
It is normal practice to arrange the protection characteristics in
zones as shown in Figure 9.7b.
Exercise
Why is it necessary to cater for
a small fault resistance? What
will happen if the fault
resistance is very large?

Consider the following simple feeder arrangement on which a


fault involving a small fault resistance Rf has occurred.

Figure 9.8
If Rf 0 the measured impedance

ZM

V
xZL
I

Otherwise

ZM

V
xZL Rf
I
PRINCIPLES OF PROTECTION

9-11

Module 1

This means that if the relay characteristic is confined only to the


solid fault locus as shown in Figure 9.6 and not extended to that
shown in Figure 9.7a, then the relay will fail to see this as a fault.
For very large fault resistances, the measured impedance can fall
well outside the extended characteristic shown in Figure 9.7a.
The first zone relay is set to protect about 80% of the line AB and
trips for all faults that present an impedance inside this range. The
zone-1 operating time corresponds to the fastest measuring speed
that the particular Distance relay will achieve and is typically
between 1030 ms in modern equipments.
The zone-2 characteristic operates after a delay and is seen to have
a reach in excess of the remote line terminal (this reach is
typically 120%). The delay is typically 0:30:4 s and is arranged
to give satisfactory tripping at A in an acceptable time for faults
beyond B which, due to local failure, are not cleared by the
switchgear beyond end B. This is the previously referred to remote
back-up function.
The third zone is seen to encompass the relay location. This zone
operates in a time greater than zone-2 and thus provides some
further remote back-up extending further into the system. It is
seen also to provide some local reverse back-up that is useful in
the event of the circuit breaker behind relay location A failing to
clear the fault. Figure 9.9 shows how, ideally, the operating time
of the protection would vary with fault position relative to end A.

Figure 9.9
Time distance characteristics of
a practical distance protection
scheme

9-12

SYSTEM FUNDEMENTALS

Section Nine

Exercise
Why is the zone-1 relay set to
only 80% of the line?

In practice, there is always an element of uncertainty in the


protection relay measuring process near the boundary to which it
is set. The relay accuracy is affected by prevailing system and fault
conditions and this means that if it is set to cover the full 100%
of the feeder then it could see faults in the next feeder section
clearly a situation to be avoided.

9.5 Unit protection


These are schemes which respond only to faults inside a clearly
defined zone. They utilise information derived from two (or
occasionally more) points in a system. A comparison of quantities
on the boundaries of a protected zone is the usual technique.
Examples of unit protection are differential current relays where
the current entering a zone is compared with that which leaves it.
Exercise
What are the fundamental
differences between unit and
non-unit protection?

A non-unit scheme makes measurements from information derived


only at one end of the protected zone, whereas a unit scheme
utilises information from both ends of the zone. Secondly, the
former provides back-up protection through time grading, whereas
the latter has no such property.

PRINCIPLES OF PROTECTION

9-13

Module 1

All unit schemes derive from the Merz and Price circulating
system. In its simplest form, the CTs at each end of the line are
connected as shown in Figure 9.9a so that in theory no current
flows in the relay for faults outside the zone.

Figure 9.9a
Circulating current system

Figure 9.9b
Balanced voltage system
An alternative, but similar, technique hinges upon connecting the
CTs in opposition so that under external fault conditions no
current flows in the series connected relays. This technique, often
referred to as the balanced voltage principle, is shown in Figure
9.9b.
Circulating current schemes are rather limited in feeder
applications because the voltage developed across the CT
secondaries increases in proportion to the length of the feeder.
Prohibitively large CTs are required when applying the principle to
feeders of longer than about 2 km and the balanced voltage
principle finds more widespread use. The series connected relays
are of relatively high impedance and the system may be
satisfactorily applied in principle for the protection of feeders up
to about 50 km. It should be noted that in such applications pilot
wire circuits (which are normally rented telephone cables) have to
be employed for transmitting currents from one end to the other
for the purposes of comparison.
Some modification to the basic scheme is required for two main
reasons:
(i)

(ii)

9-14

Under heavy through fault conditions, the secondary CT


currents are not exactly equal due to the inevitable
difference (however slight) in the CT ratios.
The pilot wire circuit has shunt capacitance so that, even if
the voltages at either end are identical, some current will
flow in the relays. This will tend to cause tripping under
healthy conditions.

SYSTEM FUNDEMENTALS

Section Nine

There are various methods of overcoming these foregoing


problems, the most common of which is the use of bias. Biasing
of relays consists of restraining the operation of a relay if the
magnitude of the summed local and remote currents exceeds a
certain percentage (which can be set according to the application
requirements) of the sum of the magnitudes of the individual
local and remote currents. Figure 9.10 shows a typical bias curve
in which:

Idiff=IA + IB (assuming both currents flow into the


protected zone)
and

Ibias= {kIA+}

Figure 9.10
Typical bias characteristic

Summation Devices
So far the differential principle has been discussed as though it
were applied to single-phase systems. In dealing with three-phase
systems, the principles could be applied to each phase, but this is
not economically justified because three pilot wire pairs would be
required. It is therefore common practice to sum the currents at
the feeder ends. A common arrangement using a summation
transformer is as shown in Figure 9.11.

PRINCIPLES OF PROTECTION

9-15

Module 1

Figure 9.11
A typical summation
arrangement
Here the output is proportional to:

n 2Ia n 1Ib nIc


where n is typically 3:
Note that in practice there are a number of different types of
current differential protection schemes in use and their usage
depends upon the type of application. For example, static relays
employing metallic pilot wires as a means of sending power
frequency analogue signals between the two ends of the protected
zone are suited for circuit lengths of only up to about 50 km,
principally because of the limitation imposed by the resistance and
capacitance of the pilot wire. An alternative scheme is the phase
comparison carrier protection in which a high frequency carrier
signal, modulated by a power frequency signal, is sent over the
protected line iteself. This type of scheme is suitable for the
protection of long lines up to about 500 km in length.
More recently, the development of high capacity, long-haul digital
communication systems (such as fibre optical links) has made
possible the development of digital current differential relays. Here
the power frequency analogue signals are firstly coded into binary
data and because of the speed and high capacity of the
communications link, it is then possible to send the current
signals of each phase (rather than a single summated quantity) to
the other end of the protected zone. More details of these various
types of practical schemes can be found in module three section
five on differential protection.

? Self-assessment question 9.1


Figure 9.12 shows a feeder arrangement on which overcurrent
protection relays R1 and R2 are employed with the CT ratios as
shown.
9-16

SYSTEM FUNDEMENTALS

Section Nine

Figure 9.12
Each breaker has an operating time of 0:2 s and the plug setting
on both relays is 125% for normal overcurrents. If each relay has
the inverse characteristic as shown in Figure 9.3, determine the
following:
(a)

The time multiplier setting for relay R2 if the three-phase


fault current is 2 kA and the corresponding fault clearance
time is to be 1 s.

(b)

The time multiplier setting for relay R1 necessary for


discrimination.

(c)

The fault clearance time for a 4 kA three-phase fault


occurring beyond breaker B2 and also for a similar fault
occurring between the two breakers.

(d)

Will the same time multiplier settings be suitable when


dealing with earth fault currents of 1 kA if the plug setting
for the earth fault element is 40%?

? Self-assessment question 9.2


(a)

Outline the principle of distance protection and explain


why it is useful for performing remote back-up protection
functions.

(b)

A distance relay with a MHO characteristic is situated at


end A of a 400 kV single circuit transmission line AB and
has a first zone setting of 80%. The positive phase
sequence impedance of the line AB is 2:0 j36:0 and
the relay is set to protect 80% of this line. If a singlephase- earth fault of resistance 20 O occurs three-quarters
of the way along the line from end A, show that the relay
at end A under-reaches to such an extent that it will not
see this as a fault in zone 1 (the remote-end infeed from
end B can be neglected).

PRINCIPLES OF PROTECTION

9-17

Module 1

Self-assessment question 9.3

(a)

Outline the principle of a simple differential protection


scheme and explain why it is necessary to use a bias type
characteristic.

(b)

A low voltage distribution line is protected by a bias current


differential protection as shown in Figure 9.13.

Figure 9.13

An internal fault occurs on the feeder such that:

IA = (40 j30)A
IB = (20 + j15)A
If the tripping criterion for the relay is such that:

deduce a value of the constant k for which the relay would just
operate.

Self-assessment question 9.4

A 30 MVA transformer feeds several feeders through an 11 kV


busbar. The transformer circuit breaker is provided with a 250:1
current transformer and each feeder is provided with a 100:1
current transformer. Each set of current transformers feeds an
induction-type overcurrent relay and each relay has a timecurrent characteristic given by:

where psm is the operating current in multiples of plug setting.


All relays have plug settings of

125%.

A fault on one feeder results in a phase current of

9-18

SYSTEM FUNDAMENTALS

1 000 A.

Section Nine

Determine:
(i)

(ii)

The operating time of the feeder relay if its time multiplier


setting is 0:5
The time setting required on the transformer relay to
provide a 0:5 s discrimination time margin.

? Self-assessment question 9.5


A distance relay with a MHO characteristic is positioned at the
sending end of a 400 kV single-circuit transmission line and set to
protect 80% of the line. The line has a positive phase sequence
impedance of 2 j36 O.
A single-phase-to-earth fault of impedance 10 j3:75 O occurs
at a point 60% of the way along the line from the sending end.
(i)

(ii)

Construct the MHO characteristic for the relay and use it to


show that the relay will not operate for this fault.
Determine the maximum value for a resistive fault at the
point concerned for the relay just to detect the fault.

80%

This booklet is printed


on 80% recycled paper

PRINCIPLES OF PROTECTION

9-19

You might also like